Você está na página 1de 278

2003 OITE Questions Divided by Content Domain

Foot and Ankle (20 questions)


11, 18, 28, 42, 54, 66, 77, 82, 92, 105, 121, 137, 155, 174, 188, 217, 242, 253, 265, 270
Hand (18 questions)
10, 21, 32, 49, 61, 73, 81, 91, 95, 107, 118, 126, 143, 163, 208, 238, 250, 262
Hip & Knee Reconstruction (23 questions)
2, 15, 25, 37, 44, 47, 62, 72, 88, 113, 119, 131, 142, 151, 154, 167, 179, 194, 210, 239, 251, 263,
272
Medically Related Issues (7 questions)
31, 75, 116, 160, 203, 235, 259
Musculoskeletal Trauma (52 questions)
1, 8, 14, 23, 33, 43, 50, 57, 59, 68, 74, 79, 87, 90, 94, 98, 103, 10o6, 109, 110, 112, 117, 124,
128, 132, 136, 147, 152, 158, 164, 172, 173, 177, 182, 187, 192, 196, 198, 200, 206, 214, 219,
221, 225, 228, 230, 233, 237, 241, 249, 260, 271
Orthopaedic Diseases (26 questions)
7, 13, 29, 38, 46, 63, 67, 93, 100, 111, 122, 134, 141, 149, 159, 165, 170, 193, 209, 220, 224,
229, 246
Orthopaedic Science (29 questions)
3, 20, 30, 39, 48, 60, 69, 78, 84, 102, 120, 135, 146, 153, 157, 168, 176, 181, 190, 195, 202, 205,
215, 223, 227, 231, 245, 256, 268
Pediatric Orthopaedics (35 questions)
4, 12, 17, 26, 34, 41, 51, 56, 65, 71, 80, 85, 89, 96, 101, 115, 123, 129, 138, 144, 148, 156, 162,
171, 175, 183, 186, 191, 201, 207, 216, 222, 234, 247, 258
Rehabilitation (9 questions)
22, 55, 70, 104, 133, 150, 185, 236, 261
Shoulder and Elbow (16 questions)
6, 24, 35, 52, 86, 108, 114, 125, 130, 145, 199, 213, 232, 244, 255, 267
Spine (18 questions)
5, 16, 27, 40, 53, 64, 76, 99, 139, 161, 169, 178, 189, 212, 240, 252, 264, 273
Sports Medicine (19 questions)
9, 19, 36, 45, 58, 83, 97, 140, 166, 180, 184, 197, 204, 211, 226, 243, 254, 266, 274

1. What is the most common complication associated with surgical fixation of a transverse
midshaft humeral fracture using an antegrade statically locked medullary nail?
1. infection
2. radial nerve palsy
3. brachial artery injury during distal interlocking
4. shoulder pain
5. nonunion

Chapman, et al compared clinical and radiographic results for locked IM nails and plates used
in the treatment of humeral diaphyseal fractures. The most common complication associated
with antegrade IM nails was shoulder pain and decreased ROM followed by symptomatic
hardware requiring removal. Complications varied in each group, with the highest incidence of
shoulder pain and stiffness in those patients treated with antegrade IM nails. This finding may be
due to the surgical trauma of nail insertion and proximal locking (injury of the rotator cuff,
subdeltoid bursa, and deltoid muscle). Plating was associated with a higher incidence of elbow
pain and stiffness as well as a slightly higher incidence of nonunion and infection. These
findings may be attributed to the greater amount of soft tissue disruption associated with the
plating procedure.
This study concluded that for the subset of patients requiring surgical treatment of diaphyseal
humerus fractures, both IM nailing and compression plating both provide predictable means of
achieving fracture stabilization and ultimate healing. Neither method was shown to be markedly
superior to the other. Either method, properly selected and performed, should lead to successful
results in a large majority of patients.
ANSWER

2. A 62-year-old woman with primary osteoarthritis is scheduled to undergo cementless total hip
arthroplasty. History reveals that she underwent pelvic irradiaton for uterine cancer 10 years
ago. The patient should be considered at highest risk for which of the following complications of
total hip arthroplasty?
1. Deep venous thrombosis
2. Osteolysis
3. Heterotopic ossification
4. Hip instability
5. Aseptic loosening
The correct response to this question comes out of a paper by Jacobs et al., JBJS, 77-A,
12, Dec. 1995, 1829-1835. In this paper, the authors describe their review of 1319 total hip
arthroplasty cases with insertion of a hemispherical porous-coated acetabular component without
cement. The authors identified 12 hips in eleven patients that had been previously irradiated
prior to THA. The pre-operative diagnosis was radiation osteonecrosis in five of the hips and
osteoarthritis in four of those patients who survived for the completion of the follow-up period
(average 37 months).
All of the acetablular components were of hemispherical design and were coated with an
ingrowth surface of titanium fiber-metal (Harris-Galante I or II; Zimmer). The hips were
evaluated radiographically with an initial post-op and follow-up AP radiographs with the method
of Martell et al. Definite loosening of the acetabular component was defined as a vertical or
horizontal change in the position of the component of more than two millimeters when the most
recent radiographs were compared to the six-week post-op radiographs. A component was
defined as failed if it radiographically had migrated.
In two of the three hips with clinical and radiographic failure, the acetabular component
had migrated more than one centimeter superiorly and medially. The third component had
migrated more than two millimeters. Two hips were revised at the completion of follow-up
secondary to this migration. One other hip was seen radiographically to have progressive
circumfrential radiolucency and progressive condensation of bone at the bone-implant interface
hence, it was probably loose. In total, four of nine hips evaluated throughout the follow-up
period had acetabular components which failed. The other five components were stable.
The study was not large enough to statistically determine any significance in regards to
the dosage of radiation between those hips which failed and those which do not. Similarly, no
significant conclusions can be drawn as to factors such as component size, the inclination angle,
the number of transfixation screws, the use of bone graft, or the type of femoral stem employed.
It is believed that the high failure rate observed in these hips with previous irradiation stems from
injury to the osteoblasts themselves. Histiological evidence shows generalized demineralization,
empty lacunae, and coarse disorganization of trabaculae.
ANSWER

3. Which of the following factors contributes to intervertebral disk degeneration?


1) Increase in cell numbers
2) Increase in local pH
3) Increase in aggregating proteoglycans
4) Decrease in collagen cross linking
5) Decrease in nutritional transport

From Orthopaedic Basic Science: A variety of mechanisms including declining nutrient and
waste product transport mechanisms, decreasing concentration of viable cells, cell senescence,
apoptotic debris, loss of aggregating proteoglycans, modification of matrix proteins, degradative
enzyme activity, accumulation of degraded matrix macromolecules, and fatigue failure of the
matrix, may contribute to disk degeneration. Although each of these mechanisms may alter disk
composition and microstructure, their relative importance and the interactions among them have
not been established.
ANSWER

4: Compared with other classifications of Legg-Calve-Perthes disease, the lateral pillar


classiciation has the advantage of
1.) exhibiting greater interobserver agreement
2.) remaining unchanged throughout the course of the disease
3.) quantifying the osteonecrosis visible on the frog lateral hip radiograph
4.) predicting proximal femoral growth arrest
5.) describing the degree of metaphyseal osteonecrosis

We evaluated the inter-observer agreement of radiographic methods when evaluating patients


with Perthes' disease. The radiographs were assessed at the time of diagnosis and at the 1-year
follow-up by local orthopaedic surgeons (O) and 2 experienced pediatric orthopedic surgeons
(TT and SS). The Catterall, Salter-Thompson, and Herring lateral pillar classifications were
compared, and the femoral head coverage (FHC), center-edge angle (CE-angle), and articulotrochanteric distance (ATD) were measured in the affected and normal hips. On the primary
evaluation, the lateral pillar and Salter-Thompson classifications had a higher level of agreement
among the observers than the Catterall classification, but none of the classifications showed good
agreement (weighted kappa values between O and SS 0.56, 0.54, 0.49, respectively). Combining
Catterall groups 1 and 2 into one group, and groups 3 and 4 into another resulted in better
agreement (kappa 0.55) than with the original 4-group system. The agreement was also better
(kappa 0.62-0.70) between experienced than between less experienced examiners for all
classifications. The femoral head coverage was a more reliable and accurate measure than the
CE-angle for quantifying the acetabular covering of the femoral head, as indicated by higher
intraclass correlation coefficients (ICC) and smaller inter-observer differences. The ATD showed
good agreement in all comparisons and had low interobserver differences. We conclude that all
classifications of femoral head involvement are adequate in clinical work if the radiographic
assessment is done by experienced examiners. When they are less experienced examiners, a 2group classification or the lateral pillar classification is more reliable. For evaluation of
containment of the femoral head, FHC is more appropriate than the CE-angle.
Wiig O: Interobserver reliability of radiographic classifications and measurements in the
assessment of Perthes disease. Acta Orthop Scand, 2002;73:523-530.
ANSWER

5. In a patient with cervical spondylotic myelopathy who requires surgical decompression,


which of the following is a relative contraindication to a posterior approach such as laminaplasty
or laminectomy?
1 - Multilevel stenosis and spinal cord compression
2 Fixed Cervical Kyphosis
3 Obesity
4 Instability or unwillingness to comply with postoperative bracing
5 Severe (AP canal diameter of less than 12mm) stenosis

The correct answer is 2. According to the article Cervical Spondylitic Myelopathy: Diagnosis
and Treatment, written by Sanford Emery, posterior decompression techniques are ideal for
patients with diffuse canal stenosis or dorsal cord compression due to buckling of posterior
ligamentum flavum. Posterior decompression of the cord for anterior pathology, most patients
with cervical spondylosis and OPLL, is an indirect maneuver which requires the cord to
subsequently shift posteriorly, away from the pathology, within the thecal sac in order to release
the cause of anterior compression. In order for this to take place the curvature of the cervical
spine must be setup within the saggital plane alignment. The more lordotic the cervical spine
is, the more freedom the cord will have to translate within the sac following decompression. A
straight or kyphotic spine is less likely to allow adequate movement and therefore be less likely
to relieve symptoms caused by anterior disease. Instability is also a concern with regard to
choice of approach, however, this preexisting factor only necessitates the need for posterior
fixation following decompression, and it does not exclude a posterior approach. Therefore, a
fixed cervical kyphosis is a relative contraindication if the goal is to decompress due to anterior
cervical spine pathology.
ANSWER

6: A mini open approach for a rotator cuff repair has what primary advantage over a standard
open repair?
1.
2.
3.
4.
5.

Decreased infection rate


Decreased risk of injury to the maxillary nerve
Decreased risk of deltoid avulsion
Decreased risk of injury to the biceps tendon complex
Increased rotator cuff healing rate

There are 4 principles described by Neer in rotator cuff repair.


1) Anterior-Inferior acromioplasty or reshaping or reshaping rather than acromiectomy.
2) Meticulous repair of the deltoid origin and avoidance of those procedures that may place
this area at risk for injury.
3) Releasing, mobilizing, and repairing the torn rotator cuff tendons.
4) Early restoration of passive motion through surgeon directed individualized
rehabilitation.
Experienced surgeons report .5% incidence of deltoid avulsion. In formal open repair deltoid
avulsion is rare. However, there are 2 disadvantages to formal open repair. Deltoid takedown
and repair requires a period of protection in order to avoid any inadvertent avulsion. This may
preclude early rehab/mobilization. 2nd, open repair appears to be associated with more
postoperative pain than mini-open or all arthroscopic methods.
The mini approach described in 1994 uses arthroscopic decompression (acromioplasty)
anterior deltoid detachment is no longer necessary. Therefore there is less perioperative
morbidity since the deltoid is not taken down.
ANSWER

7. What finding at initial presentation is most predictive of survival in patients with osteogenic
sarcoma?
12345-

Alkaline phosphatase level


Pathologic fracture
P53 status
Size of the tumor
Stage of the tumor

Osteosarcoma is the most common primary malignant bone tumor, excluding myeloma, and is
the third most common malignant disease in adolescence after leukemia and lymphoma. It
occurs most commonly during the second decade with a second peak during the 7th decade.
Although it can occur in all bones it is usually found within the metaphyses of long bones. The
distal femoral metaphysis and proximal tibial metaphysis are the most common sites. Patients
usually present with pain that is exacerbated with activity and 50% of the time the onset of pain
is associated with minor trauma. There are multiple forms of osteosarcoma, including highgrade central, low-grade central, telangiectatic, parosteal, periosteal and high-grade surface
osteosarcoma.
Treatment of classic high grade osteosarcoma consists of multiagent preoperative chemotherapy
combined with wide resection or amputation followed by postoperative chemotherapy. Although
the use of noeadjuvant chemotherapy is thought to prevent the development of resistant tumor
clones, destroy microscopic metastases and shrink the tumor, it has not been proven to increase
patient survival.
The single most predictive factor in osteosarcoma is the presence or absence of detectable
metastatic disease at presentation. Patients without clinically apparent metastatic disease at
presentation have a long-term survival of 60-80% while those with known metastases at
presentation have a 5-year survival of 20-40% with an overall survival rate between 10-20%.
Other prognostic factors include the degree of tumor necrosis after noeadjuvant therapy, tumor
size and elevated serum lactate dehydrogenase and alkaline phosphatase levels. Axial tumors
also have a worse prognosis due to difficulty in obtaining a wide surgical margin.
P53 is a tumor suppressor gene and functions as a cell-cycle regulator by inducing cell apoptosis.
It is one of the most commonly defective genes in osteosarcoma, but it has not been associated
with disease progression or prognosis.
The Bielack article demonstrated primary metastases, axial site and poor response to
chemotherapy as poor prognostic factors.
ANSWER

References: Bielack SS, et al: Prognostic factors in high-grade osteosarcoma of the extremities
or trunk: An analysis of 1,702 patients treated on neoadjuvant cooperative osteosarcoma study
group protocols. J. Clin Oncol 2002;20:776-790.
Gibbs, CP Jr, Weber k, Scarborough MT: malignant bone tumors. ICL 2002;51:413-428.

8.
Initial management of a hypotensive adult trauma patient in the ED includes insertion of
at least 2 large bore IV catheters and administration of :
12345-

1L bolus of hypertonic saline


2L bolus of hypteronic saline solution
1L bolus of LR solution
2L bolus of LR solution
2 units of uncrossmatched blood

Hypovolemia is the most common cause of shock in a trauma patient. Initial treatment for
hypovolemia is to insert two large bore IV needles (16-gauge or larger) and administer 2 L of
Lactated Ringers or Normal Saline. IV placement should be in the antecubital vein of each arm
to allow for fluid resuscitation if there is a unilateral vessel injury. If the patient remains unstable
after fluid resuscitation, blood transfusion is indicated. Typed and crossed blood is preferred but
this process can take an hour. Typed but not crossed blood is the next choice but typing takes
around 20 minutes. In an emergency, give O positive or O negative (especially for females in
their childbearing years).
-

Class I: loss of 15% of blood volume (~750ml). Minimal symptoms. Body recovers
on its own within 24 hours.
Class II: loss of 15-30% of blood volume (~750-1500ml). Tachycardia, tachypnea,
decrease pulse pressure, mild mental status changes.
Class III: 30-40% blood loss (~1500-2000ml). Significant tachycardia, tachypnea,
mental status changes, and decrease in blood pressure.
Class IV: more than 40% blood loss (>2000ml). Severe tachycardia, tachypnea
decrease pulse pressure, obtundation, or coma.

Resuscitation with 2L of LR solution or normal saline is the recommended intial management,


followed by blood if indicated. Type and crossmatched blood is the product of choice (takes a
hour). Type specific blood is the second choice (takes 20min). Universal donor blood O+ or O(for female pts) is usually well tolerated when given to trauma victims in shock.
Browner BD, Jupiter JB, Levine AM, Trafton PG: Skeletal Trauma, ed 2. Philadelphia, PA, WB
Saunders, 1998 pp141-143
ANSWER

9. A 30-year-old man who sustained a knee dislocation in a football game underwent an


arthroscopically assisted anterior and posterior cruciate ligament reconstruction 1 week after
injury. Examination 3 months after injury reveals a small effusion, decreased mobility of the
patella and passive range of motion from 20-60 degrees despite physical therapy. What is the
nest most appropriate step in management.
12345-

Aggressive physical therapy


Corticosteroid injection
Revision anterior cruciate ligament reconstruction
Manipulation
Arthroscopy

Many authors have commented on the acute surgical management of ACL tears. The
general conclusions are that acute surgical management (<2-3 weeks) should be avoided in order
to avoid motion problems and arthrofibrosis. In a study out of HSS (Marzo et al, Arthroscopy 8,
10-18; 1992), a review of 642 patients who underwent ACL repair, 18 developed motion
problems. Out of these, 9 patients had reconstructions within 2 weeks. A knee dislocation with a
combined ACL/PCL injury may not afford the surgeon the luxury of waiting and may require
acute treatment secondary to instability of the knee or associated injuries (especially vascular).
The first reference discusses the arthroscopic management of combined ACL/PCL injuries. The
authors discuss timing of surgery and advocate waiting about 4-6 weeks.
This patient presents with significantly restricted ROM as well as patellofemoral
restriction of motion at 3months. The second reference discusses the management of such
patients. They state that limitation of extensions is usually secondary to pathology within the
intercondylar notch region whereas limitation in knee flexion are secondary to scar in the
medial/lateral gutters or within the suprapatellar notch. They recommend arthroscopic
evaluation with debridement of scar in the notch or fat pad as well as any Cyclops lesion
(although this patient did not have a clunk).
Explanation of wrong answers: 1- This patient is already undergoing therapy. 2- Corticosteroid
injections would not help in that the scar has already formed 3- there is no evidence that the ACL
reconstruction has failed and would require revision. 4- Manipulation may be helpful but does
not directly address the pathology as well as arthroscopy.
References
1- Fanelli GC, Giannotti BF et al Arthroscopically-assisted combined ACL/PCL
reconstruction. Arthroscopy 1996 12:5-14
2- Bach et al: Reflex Sympathetic dystrophy, Patella infera contracture syndrome and loss of
motion following ACL surgery. ICL 1997 46:251-60
ANSWER

10. Active mobilization following flexor tendon repair is best accomplished with the wrist in:
1. flexion and the MCP joints in flexion
2. flexion and the MCP joints in extension
3. neutral and the MCP joints in extension
4. neutral and the MCP joints in flexion
5. extension and the MCP joints in flexion

Following flexor tendon repair, 2 post-operative mobilization techniques are commonly


employed. The first, advocated by Kleinert, active finger extension is used with passive flexion
allowed by means of a rubber band attached to the fingernail and at the wrist. The wrist and MCP
joints are held in flexion. The rubber band maintains the PIP in 40-60 degrees of flexion,
allowing extension of the PIP against the rubber band. This avoids stretch on the repair, but the
allowed motion facilitates healing. The second technique, advocated by Duran, involves a
controlled passive motion technique with dorsal blocking of the fingers.
Typically, dynamic extension splinting is started at 6-8 weeks, with strengthening permitted at 810 weeks.
Horii et al investigated flexor tendon excursions in various loading conditions. They also
investigated different wrist splinting techniques following flexor tendon repair. Comparing of the
Kleinert splint, P-splint (Kleinert splint with a palmar bar), and S-splint (synergistic splint) was
performed. The S-splint allows for synergistic motion of the wrist and fingers. Increased tendon
excursion was noted with use of the S-splint.
Though the references cited do not specifically address it, the results presented indicate that
tendon excursion and IP motion are improved with the wrist in extension and the MCP joints in
flexion. This suggests that wrist extension and MCP flexion may be the best position for active
mobilization following flexor tendon repair.
References:
Horii E et al: Comparative flexor tendon excursions after passive mobilization: An in
vitro study. J Hand Surg Am 1992; 17:559-66.
Manske PR (ed): Hand Surgery Update. Englewood, CO, ASSH, 1994, 44-49.
ANSWER: 5

11.

In planning a surgical approach on a 23 year-old ballet dancer with os trigonum


syndrome, it is important to remember that the os trigonum is in what position relative to
the flexor hallucis longus tendon?
1. Anterior
2. Posterior
3. Medial
4. Lateral
5. Within the tendon

Explanation:
Relatively straight forward answer. In the article by Marotta et. al., the approach to the
accessory ossicle was as follows: A 3cm straight incision was made at the posterior lateral ankle,
posterior to the peroneal tendons as they approach the tip of the lateral malleolus. Care is taken
to protect the sural nerve and all of its branches. Deeper dissection is directed toward the
posterior ankle joint and the capsule is identified. The capsule is opened in line with the skin
incision over the os trigonum, which is palpable at the posterior lateral margin of the talus.
Sharp dissection is then used to excise the ossicle in its entirety. They point out that care must be
taken to avoid injury to the flexor hallucis longus tendon, the tibial nerve, the posterior tibial
artery, and the posterior tibial tendon on the medial side of the ossicle.
The incidence or radiographically detectable os trigonum has been reported between 313% in general population. It is a distinct but infrequent ossicle, separate from the posterior
border of the talus and containing a groove for the flexor hallucis longus tendon. Normally the
talus contains two posterior tubercles, a medial and lateral tubercle. It is thought that os
trigonum is a separation of the second ossification center of the lateral tubercle from the
remainder of the talus.
Symptomatic os trigonum impingement presents with recurrent pain, stiffness,
tenderness, and swelling behind the ankle in the space anterior to the Achilles tendon. This is
especially noticeable during pointe work in dancing (i.e. the demi-pointe position in ballet with
the dancers foot in maximum plantar flexion) and extreme plantar flexion in sports like javelin
and soccer.
ANSWER

Wredmark, T., Carlstedt, C.A., Bauer, H., Saartok, T. Os Trigonum Syndrome: A clinical Entity
in Ballet Dancers. Foot Ankle 1991;11:404-406.

12. Which of the following approaches is most appropriate to correct a projected limb-length
discrepancy of greater than 6 cm at skeletal maturity?
1. Epiphysiodesis (contralateral) alone
2. Acute shortening alone
3. Shoe lift alone
4. Gradual distraction lengthening
5. Physeal bar resection

Radiologic assessment of leg length discrepancy include determination of leg lengths


(scanogram), determination of skeletal age using the Greulich-Pyle atlas (standard radiographs of
left hand/wrist for boys and girls at various ages), and then determination of remaining growth
using the Green-Anderson growth remaining method or the Moseley straight-line graph.
General guidelines for treatment based on predicted LLD are:
0 to 2 cm:
2 to 6 cm:
6 to 20 cm:
> 20 cm:

no treatment
shoe lift, epiphysiodesis, shortening
lengthening, with or without other procedures
prosthetic fitting

Shoe lifts greater than 5 cm are not well tolerated. Epiphysiodesis has low morbidity and low
complication rate. It does make the normal leg abnormal and result in a decrease in the patients
stature, however. Shortening has the same indications as epiphysiodesis but is generally offered
to patients who are skeletally mature. Generally, the femur can be shortened up to 5 cm and the
tibia 3 cm with little loss of function. The risk of nuerovascular complications with tibia
shortening are higher though. Physeal injury can lead to the development of physeal bars, or
bone bridge across the physis that lead to growth arrest. Physeal bar resection may be indicated if
less than 50% of the physis is damages and more than 2 cm of growth remain in the affected
growth plate.
ANSWER

Moseley CF: Assessment and prediction in leg-length discrepancy. Instr Course Lect 1980; 38:
325-30.
Morrissy RT, Weinstein SL (eds): Lovells and Winters Pediatric Orthopaedics, ed 5.
Phioladephia, PA. Lippincott Williams and Wilkins, 2001, pp. 1105-50.

13. A 43 year old woman has a proximal humerral lesion that was found incedentally on the
chest radiograph shown in Figure 1a. She reports no pain and has full shoulder range of motion.
An MRI scan is shown in Figure 1b. What is the next most appropriate step in management?
12345-

ANSWER

observation
biopsy
Serum protein electrophoresis
Ct of the chest, abdomen, and pelvis
administration of bisphosphonates

observation (an incedentaloma)

Marco, et.al. Cartilage tumors: Evaluation and treatment. JAAOS 2000, 8, p 292-304.

14.
Formation of heterotopic bone in the hip abductor musculature after antegrade
intramedullary femoral nailing is most closely associated with
1.
2.
3.
4.
5.

ipsilateral tibia fracture


the age of the patient
timing of fixation
reaming
wound irrigation

ANSWER

reaming

15. A patient who received low-molecular weight heparin after under going total hip arthroplasty
1 day ago had normal neurologic function and a hematocrit level of 30.8. On the third postoperative day, she reports severe hip pain, is unable to dorsiflex her foot, and has a hct of 21.6.
Radiographs show the implant in good position. These developments are most likely caused by
1.
2.
3.
4.
5.

limb overlengthening
hematoma
infection
DVT
heparin induced thrombosis syndrome

Sciatic nerve palsy induced by post-op hematoma was reported by Sorenson and Christensen in
the Journal of Arthroplasty 1992; 7:551.
ANSWER

16. Respiratory Compromise following anterior cervical spine procedures is most closely
associated with
1.
2.
3.
4.
5.

Multilevel approaches above C4


Surgical time less than 3 hours
Obesity
Smoking
Diabetes Mellitus

Ref: Sagi HC, Beutler W, Carroll E, Connoly PJ. Airway complications associated with surgery
on the anterior cervical spine. Spine 2002; 27:949-953.
Retrospective study: 311 anterior cervical spine procedures. Risks for respiratory distress
include 1) prolonged procedures (>5 hours), 2) more than 3 levels C2, C3, C4, 3) more than
300cc EBL. Smoking was not a risk factor, fat was not a risk factor, they dont comment on DM,
but intuitively it doesnt seem to make much sense.
ANSWER

17. Figures 2a and 2b show the radiographs of a patient who has disproportionately short stature.
There is a defect in the gene coding for
1
2
3
4
5

fibroblast growth factor receptor -3


fibrillin
cystathionine synthase
adrenocorticotropin
parathyroid hormone

FGF- 3: Achondroplasia is caused by a defect in FGF-3


- Autosomal dominant disorder;
- Arises due to a point mutation for the gene encoding a fibroblast growth factor-3
- Over 80% of all persons with this disorder are born to parents who are not achondroplastic, &
such parents rarely have second achondroplastic child;
- It is presumed in these instances that disorder is consequence of mutation and may be related to a
mutation in a fibroblast growth factor gene;
- Growth of endochondral bones is abnormal in achondroplasia and causes disproportionate and
rhizomelic shortening of limbs;
- Since intramembranous ossification is unaffected head is relatively large and the forehead is
prominent;
- Long bones are wide with flaring of the metaphyses
- Stenotic deformity in the lumbar spine of pts w/ achondroplasia tends to woresn w/ increasing age
Fibrillin: defect causes Marfan syndrome. Typically the patient is unusually tall with exceptionally long extremities
and long, tapering fingers and toes. Typically the patient is unusually tall with exceptionally long extremities and
long, tapering fingers and toes. A variety of spinal deformities may appear, including kyphosis, scoliosis, or rotation
or slipping of the dorsal or lumbar vertebrae
Cystathionine synthase: related to homocysteinuriathats about all I could find on it.
Adrenocorticotropin: aka - adrenocorticotropic hormone (ACTH). Deficiency causes secondary hypocortisolism
(Addisons Disease). Symptoms include nausea, vomiting, weakness, fatigue, fever, and hypotension. Cushing's
disease results from a pituitary adenoma that causes excess production of ACTH
Parathyroid hormone: aka PTH is an 84-amino acid peptide synthesized in and secreted from the chief cells of the
(four) parathyroid glands. PTH helps regulate plasma calcium, directly activates osteoblasts, and modulates renal
phosphate filtration. Decreased calcium levels in the extracellular fluid stimulate release of PTH, which acts at the
intestine, kidney, and bone to increase serum calcium

ANSWER

Sponseller PD: The skeletal dysplasias, in Morrissy RT, Weinstein SL (eds): Lovell and Winter's Pediatric
Orthopaedics, ed 5. Philadelphia, PA, Lippincott Williams & Wilkins, 2000, pp 243-285.
Koval KJ (ed): Orthopaedic Knowledge Update 7. Rosemont,IL, American Academy of Orthopaedic Surgeons,
2002, pp 209-218.
Baitner AC, Maurer SG, Gruen MB, DiCesare PE: The genetic basis of the osteochondrodysplasias. J Pediatr Orthop
2000;20:594-605.

18. While open reduction and internal fixation as a treatment of calcaneal fractures has gained
greater acceptance in the last two decades, closed reduction and pinning (Essex Lporesti type
technique) is felt to yield satisfactory outcomes in which of the following fracture patterns?
12345-

ANSWER

lateral process
anterior process
tongue-type
comminuted posterior facet
joint depression type

tongue type

Tornetta, P, The Essex Lopresti reduction for calcaneal fractures revisited. JOT 1998: 12: 469473.
Koval, et.al. OKU 7 pp 547-564.

19. A 9-yo boy sustains a lateral blow to the knee while playing soccer. Examination reveals
valgus knee laxity. Knee radiographs are normal. What is the next most appropriate step in
management?
1.
2.
3.
4.
5.

Casting
Bracing
CT
MRI
Stress radiographs

--------------------------------------------------------Sponseller PD, Staniski CL: Rockwood & Wilkins Fx in Children, 5th ed., p 992 in chapter 23
Fx & Disloc about the knee.
Stress views are indicated when the initial radiographic appearance is negative but the history
and physical signs suggest epiphyseal separation. Stress radiographs may be falsely negative if
there is associated muscle spasm. Adequate analgesia relaxes muscle spasm and helps protect the
physis from further injury during examination. Some traction should be applied to the lower leg
as the knee is angulated for radiographic examination. Smith (JBJS 1962, 44:1659-1660)
reported two 15 yo boys injured on the football field for whom the diagnosis of distal femoral
epiphyseal separation could be made only on stress views.
ANSWER

: 5

20. What is the effect of tobacco smoking on musculoskeletal tissues?


1- Increases the relative risk of a hip or wrist fracture
2- Increases ligamentous laxity
3- Causes osteomalacia
4- Causes articular cartilage atrophy
5- Causes skeletal muscle hypertrophy

Smoking has been shown to adversely affect bone mineral density, lumbar disk disease, the rate
of hip and wrist fractures, and the dynamics of bone and wound healing.
Smoking increases the concentrations of free radicals, which have been suggested to be involved
in bone resorption. Melhus et al examined whether the dietary intake of antioxidant vitamins
may modify the increased hip fracture risk associated with smoking. After adjustment for major
osteoporosis risk factors, the odds ratio (OR) for hip fracture among current smokers with a low
intake of vitamin E was 3.0 (95% confidence interval 1.6-5.4) and of vitamin C 3.0 (1.6-5.6). In
contrast, the OR decreased to 1.1 (0.5-2.4) and 1.4 (0.7-3.0) with high intakes of vitamin E and
C, respectively. This effect was not seen for beta-carotene, selenium, calcium, or vitamin B6. In
current smokers with a low intake of both vitamins E and C, the OR increased to 4.9 (2.2-11.0).
Porter SE. Hanley EN Jr: The musculoskeletal effects of smoking. J Am Acad Orthop Surg
2001 ;9:9-17.
Melhus H, Michaelsson K. Holmberg L. Wolk A, Ljunghall S: Smoking, antioxidant vitamins.
and the risk of hip fracture. J Bone Miner Res 1999;14:129-135.
ANSWER

21.

Figures 3a and 3b show the radiographs of a 29-year-old surgeon who reports persistent
pain in the dominant wrist after a fall 3 months ago. Examination reveals that range of
motion in the affected wrist is nearly symmetric with the opposite side, although he
reports pain at the extremes of dorsiflexion and radial deviation. Management at this
time should consist of
12345-

scaphoid bone graft and screw fixation.


scaphoid bone graft, screw fixation, and radial styloidectomy.
scaphoid excision and capitolunate-triquetral-hamate fusion.
proximal row carpectomy.
vascularized bone graft from the distal radius to the scaphoid.

The radiographs show a scaphoid fracture with nonunion. The other significant piece of
information is in the history, which describes the patient having pain with radial deviation and
dorsiflexion. These motions bring the scaphoid into close contact with the radial styloid.
Radiographs also show the radial styloid to already have a few osteophytes. However there are
no signs of Scaphoid Nonunion Advanced Collapse, which usually takes years to develop. Thus
both proximal row carpectomy or scaphoidectomy with four corner fusion can be eliminated as
answer choices since they are used for SNAC wrist. Additionally the literature is unclear as to
which is a better operation for Scaphoid Nonunion Advanced Collapse. In fact, this point is so
controversial that it cannot really be tested. So eliminate both answers. Vascularized bone graft
is also utilized, but too controversial to be correct.
Scaphoid fractures can be divided into acute (<6 weeks), early (<6 months), and late (>6
months). The generally accepted treatment for acute fractures is casting, although many will
treat with internal fixation for displacement or waist and proximal pole fractures given the very
high rates of nonunion. Acute fractures do not require bone grafting. Early fractures such as this
one are best treated by screw fixation and bone grafting. Both cited articles recommend radial
styloidectomy if any amount of arthritis is present at the radial styloid. Late fractures with
minimal arthrosis can be treated with bone grafting as well, but arthrosis mandates either a PRC
or scaphoidectomy and four corner fusion.

ANSWER

scaphoid bone graft, screw fixation, and radial styloidectomy

Stark HH, Rickard TA, Zemel NP, Ashworth CR: Treatment of ununited fractures of the
scaphoid by iliac bone grafts and Kirschner wire fixation. JBJS 1988; 70:982-991.
Smith BS, Cooney WP: Revision of failed bone grafting of nonunion of the scaphoid:
Treatment options and results. CORR 1996; 327: 98-109.

22. A 22 year old transtibial amputee is an avid runner but finds it difficult to run on uneven
ground. What prosthetic foot modification would most likely address this problem?
1 Stationary ankle flexible endoskeleton
2 articulated dynamic response
3 solid ankle cushioned heel
4 solid ankle split keel
5 energy storing

Patients with a below-knee amputation can be expected to have good return of function.
It is important to maintain knee motion. The long posterior myocutaneous flap is standard, but
good results have also been obtained with a posterolateral skew flap. Advances in prosthetic
wear include flexible sockets, silicone liner suction for suspension, and dynamic response feet.
The stationary ankle flexible endoskeleton is the most basic advance over the solid ankle
cushioned heel foot. A small piece of wood is attached to the base of the keel, which is
completed by a semirigid foam and belting materials that allow a smooth roll over the foot. The
flexibility in the keel also allows for better accommodation to uneven surfaces by permitting
small amounts of inversion and eversion. The stationary ankle flexible endoskeleton foot,
however, does not provide enough stiffness or lever arm for push-off during higher level
athletics, making a dynamic response foot necessary. In this design, the keel deforms under load,
but its inherent memory allows for a return of energy upon removal of the load, similar to a
spring. A split in the keel can allow inversion and eversion of the foot. The dynamic keel can be
long or short and is fitted to each patient based on activity level, height, and weight. The most
advanced type of foot is the articulated dynamic response foot. As opposed to the single keel
design of the dynamic foot, the articulated dynamic foot has an articulation at the ankle with
bumpers that permit specified amounts of motion. These feet are superior in accommodating
uneven surfaces. The ideal foot prosthesis depends on the need for dynamic response versus the
need to accommodate to uneven ground. For instance, running requires a greater dynamic
response and golfing a greater need for inversion and eversion. The individual must be involved
in the choice of feet. Some active individuals may wish to have several options for different
activities.
ANSWER:

OKU 7 Chapter 15: Amputation and Prosthetics 127-137

23. A 60-year-old patient undergoes a cementless hemiarthroplasty for a displaced femoral


neck fracture via a direct lateral (Hardinge) approach to the hip. What gait abnormality is
most likely to be present at the 3-month follow-up examination?
12345-

Gluteus maximus lurch


Gluteus medius lurch
Vaulting gait
Circumducted gait
Stiff-legged gait

The direct lateral approach to the hip (the Hardinge Approach) is performed with the patient in a
supine position. The incision is centered over the greater trochanter and extends distally (8cm)
along the femoral shaft and proximally it curves slightly posteriorly and ends at the level of the
ASIS. The gluteal fascia and the ITB are then divided the entire length of the incision, and in
line with it. The greater trochanter is thus exposed, the insertion of the gluteus medius is then
incised, in line with its fibres, from the origin of the vastus lateralis, curving around the greater
trochanter and splitting the muscle proximally (not to exceed 4 cm from the tip of the greater
trochanter). The hip is then adducted and the capsule exposed. The repair is done in layers,
reattaching the anterior gluteus medius and vastus lateralis tendons to their posterior, undisturbed
tendon counterparts.
The danger in this approach, as initially reported by Hardinge, and by Ramesh et al (JBJS 1996),
is damage to the superior gluteal nerve (usually by over-zealous retraction). Inadequate repair of
the gluteus medius can also lead to adductor dysfuntion. The superior gluteal nerve exits the
pelvis at the greater sciatic foramen above the piriformis and rus horizontally forward beneath
the posterior gluteus medius. It is the motor nerve to gluteus medius, gluteus minimus, and TFL.
It can be damaged by stretch (retraction, or if the g. medius split extends too proximally (i.e. 4-5
cm proximal to the tip of the greater trochanter), it may be damaged directly.

ANSWER

24. Which of the following are considered the most characteristic features of shoulder arthritis following
anterior stabilization procedures?
1. Internal rotation contracture and inferior glenoid wear
2. Internal rotation contracture and posterior glenoid wear
3. Internal rotation contracture and superior glenoid wear
4. External rotation contracture and posterior glenoid wear
5. External rotation contracture and superior glenoid wear

The primary goal of anterior repairs for glenohumeral instability is to restore normal function by
eliminating recurrent instability. Repair procedures include Bankart, and capsular shift (anatomic) Putti-Platt,
Magnuson-Stack, and Bristow (anatomic altering). There has been an association between glenohumeral
instability repairs and glenohumeral arthritis. This arthritis is thought to be more common after anterior
instability repairs that result in excessive anterior capsular tightening and internal rotation contracture.
Capsulorrhaphy arthropathy causes severe glenohumeral arthristis that presents in pts at a much younger age
than primary OA. The gross pathology has been shown to be this severe internal rotation contracture and
eccenstric posterior glenoid erosion and wear.
Subscapularis lengthening and capsular release has been recommended to treat shoulders with pain and
internal rotation contracture after anterior instability repair in which the articular surfaces are not destroyed.
When the articular surface is destroyed, prosthetic arthroplasty is preferred. With posterior wear, , eccentric
reaming or bone grafting is sometimes needed.
ANSWER

2.

Internal rotation contracture and posterior glenoid wear

References:
Green: Shoulder arthroplasty for advanced glenohumeral arthritis after anterior instability repair. J Shoulder
Elbow Surg2001; 10:539-45

25. Figures 4a and 4b show the AP and lateral radiographs of a patient who has knee pain after a
fall. History is significant for femoral revision of a total hip arthroplasty 2 years ago. Treatment
should consist of
1.
2.
3.
4.
5.

open reduction and internal fixation with a cable-plate


open reduction with an allograft strut and cerclage wire fixation
closed reduction and retrograde intramedullary nailing
closed reduction with medial and lateral flexible retrograde rods
revision to a longer stem femoral component

The rate of periprosthetic fracture is between 0.6% and 2.5% in primary total hips, and in one
study, is slightly higher in cemented implants. In revisions, the rate is higher, 2.8%. The
Vancouver system classifies periprosthetic fractures:
Type A: involves the lesser or greater trochanter
-- if secondary to osteolysis, revise
-- if stem well fixed, conservative treatment
Type B: well fixed stem, fx involves the region of the stem
-- internal fixation
Type C: fx distal to stem
--internal fixation
Options for fixation include:
Plates/screws
Cerclage wiring/cables, screws with or without plate
Allo/autograft
Revision stem
This fracture is too distal for a revision stem and there is no indication that it is loose anyway.
Anything retrograde will just create a stress riser at the interface between the two implants. That
leaves you with answers 1 and 2, but this is a supracondylar femur fracture which you would not
fix with an allograft strut, you would plate it with a plate that extends above the level of the stem
by 2 cortical diameters.
Lewallen DG, Berry DJ: Periprosthetic fracture of the femur after total hip arthroplasty:
treatment and results to date. ICL 1998; 47: 243-249
ANSWER

26. A rotational periacetabular osteotomy offers what advantage over a medial displacement
pelvic osteotomy for treating adolescent hip dysplasia?
12345-

Can be used for an incongruent hip.


Provides intrinsic stability.
Provides only lateral acetabular coverage.
Provides hyaline cartilage coverage.
Internal fixation not required.

Gillingham BL, Sanchez AA, Wenger DR: Pelvic osteotomies for the treatment of hip
dysplasia in children and young adults. J Am Acad Orthop Surg 1999;7:325-337.
Takashi S, Hattori T, Kanishi N, Imata H: Acetabular development after Salters
innominate osteotomy for congenital dislocation of the hip: Evaluation by three dimensional
quantitative method. J Pediatr Orthop 1998;18:802-806.
Salter RB: The classic: Innomiate osteotomy in the treatment of congenital dislocation
and subluxation of the hip. J bone Joint Surg Br 1961;43:518.
Explanation:
Although the true incidence of persistent acetabular dysplasia in adults is unknown, its
role as a significant cause of premature hip osteoarthritis is well established. It is estimated that
between 20% and 50% of cases of degenerative disease of the hip are secondary to subluxation
or acetabular dysplasia. In a classic study of 130 patients with degenerative arthritis of the hip,
Stulberg and Harris found that 63 patients (48%) had underlying acetabular dysplasia.
Ideally, patients with acetabular dysplacia and subluxation are identified and treated in
infancy. Failing this, treatment instituted as early as possible in childhood, preferably before age
4, will take maximum advantage of the inherent remodeling capabilities of the hip joint.
Redirectional (complete) osteotomies improve the coverage of the femoral head by
shifting the position of the acetabulum and the acetabulum itself remains unchanged: single
innominate osteotomy (Salter), triple innominate osteotomy (Steele), periacetabular osteotomy
(Ganz). Redirectional (incomplete) osteotomies restore acetabular morphology by changing
the shape of the acetabulum: Pemberton osteotomy, Dega osteotomy. Salvage/augmentation
osteotomties may be indicated when a congruent reduction between the femoral head and the
acetabulum cannot be obtained: Chiari osteotomy, slotted-shelf procedure.
The above question asks what is the primary difference between a Ganz rotational
periacetabular osteotomy and a Chiari medial displacement osteotomy. The Chiari (not the
Ganz) is used for an incongruent hip (#1 is wrong). Both require internal fixation (#5 is wrong).
The benefit of a Ganz osteotomy is that it can provide multiplanar (#3 is wrong) correction of the
acetabulum (and its hyaline cartilage) in a congruent joint. The Chiari osteotomy, on the other
hand, actually makes a new roof in an incongruent joint by cutting above the acetabulum through
the ilium. It, along with the slotted-shelf procedure, requires periarticular soft tissue metaplasia
(since there is no hyaline cartilage) for success. Thus the preferred response is 4.
ANSWER

27. Lumbar spine disk herniations are most likely to show the greatest degree of spontaneous
resorption if they are:
1. contained
2. recurrent
3. sequestered
4. far lateral
5. in the upper lumbar segments

Saal, et al studied the natural history of lumbar intervertebral disc extrusions with nonoperative
management. This study clearly demonstrates that lumber disc extrusions morphologically
change in a manner consistent with resorption when treated nonsurgically. This occurs to the
greatest degree in sequestered disc herniations. The maximum shrinkage of the extrusions
occurred in the cephalocaudal dimension. Lesser degrees of shrinkage were noted in the AP
dimension of the disc material. The largest extrusions decreased the most in size and had the
greatest likelihood of total resolution.
Theoretically, when the nuclear material is exposed to the vascular supply of the epidural space
and is separated from the nutrient supply of the disc, the process of resorption begins. The
separation of the extruded fragment from the disc may preclude the production and
replenishment of the hydrophilic proteoglycan in the disc, thereby leading to progressive
desiccation. Additionally, cellular elements in the epidural space stimulated by the inflammatory
response would promote phagocytosis of the offending nuclear material. Inflammation and
neovascularization of the extrusion may maintain the hydration level initially, however, as the
inflammation subsides, the process of neovascularization would lead to phagocytosis and
resorption.
The distance by which the extruded fragment is separated from the disc facet level may also
play an important role in the loss of nutrition and subsequent phagocytosis and resorption.
Furthermore, in this study, no follow-up scans of the fragments demonstrated fibrosis resulting
from the disc extrusion. Conservative management did not lead to the delayed sequela of
perithecal or perineural fibrosis. Prompt resolution of inflammation plays a key role in limiting
the extent of fibrotic reaction.
ANSWER

28. A claw toe is distinguished from a hammer toe by which of the following features?
1.
2.
3.
4.
5.

dorsiflexion of the metatarsophalangeal joint


dorsiflexion of the proximal metatarsophalangeal joint
dorsiflexion of the distal interphalangeal joint
plantar flexion of the metatarsophalangeal joint
plantar flexion of the proximal interphalangeal joint

Hammer toe is a deformity of the proximal interphalangeal joint in which there is a


flexion deformity, either fixed or flexible. The fixed deformity is one in which the proximal
interphalangeal joint cannot be straightened; and a flexible deformity in one in which the joint
can be brought back into anatomic alignment.
The patient typically complains of pain over the tip of the toe and over the proximal
interphalangeal joint. In addition, the patient may develop callus underneath the tip of the toe or
the proximal interphalangeal joint region.
Initially, conservative management is indicated. This consists of adequately padding the
area with lambs wool or felt and changing the patients shoe wear to accommodate the padding.
If conservative management fails, then operative treatment can be carried out. The operative
treatment consists of excising the distal portion of the proximal phalanx and then holding the toe
in correct alignment for approximately 6 weeks, until satisfactory fibrous union has occurred.
Athrodesis of the PIP joint can be attempted, but fusion rate is typically low (50%).
Claw toe, however, is a deformity in which there is a hammer toe deformity as well as
dorsiflexion of at the MTP joint. Just as in hammer toes, these claw toe deformities can be either
fixed or flexible. The etiology is usually idiopathic, although it may be associated with
neurologic disorders. Again, the patients chief complaint is typically pain over the PIP joint
where the toes hit the top of the shoes. Callus may form beneath the metatarsal heads secondary
to depression of the metatarsal heads by dorsiflexion of the proximal phalanges.
Conservative treatment involves fitting the patient with shoes with an adequate toe box to
accommodate dorsiflexion of the toes, adding a soft insole, and using soft metatarsal support. If
this fails to alleviate the patients discomfort, operative treatment can be carried out. With a
fixed deformity, a release of the contracted structures on the dorsal aspect of the MTP joint,
which consists of both extensor tendons, the joint capsule, and the collateral ligaments, can be
undertaken to straighten the contracture. If there is also a fixed hammertoe, then a condylectomy
with excision of the distal portion of the proximal phalanx can be done. A Girdlestone flexor
tendon transfer, in which the FDL tendon is transposed to the dorsal aspect of the proximal
phalanx, is carried out to provide increased plantar flexion pull. Pin fixation is often used to
maintain alignment. This form of immobilization can be used for 4-6 weeks. If the deformity is
a flexible one, a Girdlestone transfer may produce a satisfactory result alone.
Turek. Chap. 19, p. 667-668.
ANSWER

29. A 50-year-old man has had left groin pain for the past 3 months. A radiograph, MRI scan,
abdominal CT scan, and photomicrograph are shown in Fisgures 5a through 5d. Which of the
following problems is considered a postoperative complication for this histologic type of bone
lesion?
1.

Infection

2.

Deep Venous Thrombosis

3.

Excessive Bleeding

4.

Hypercalcemia

5.

Malignant Hyperthermia

The history of groin pain combined with the abnormal radiograph and MRI demonstrates a lesion
in the left femoral neck. The CT abdomen shows a large mass in the left kidney. The histologic
sample shown is indicative of renal tissue, more specifically renal cell carcinoma. The lesion in
the femur is a renal cell metastasis.
Renal cell carcinoma tumors are well vascularized and massive blood loss is a risk of surgical
excision of such tumors. A study in Acta Orthopaedica Scandinavica showed that preoperative
embolization of spinal metastases reduced perioperative blood loss by 33%. This study
recommende that preoperative embolization should precede surgical treatment of spinal
metastases of renal cell carcinoma where it is anticipated that the operation will extend into the
pathological tissue.
ANSWER

30: Which of the following arteries provides the primary blood supply to the posterior cruciate
ligament?
1.) Superior medial geniculate
2.) Superior lateral geniculate
3.) Middle geniculate
4.) Inferior medial geniculate
5.) Inferior lateral geniculate

The vascular supply of the knee arises from the popliteal artery. This artery gives rise to the
superior and inferior geniculate arteries which each further divide into medial and lateral
branches. A middle geniculate artery also comes off the popliteal. This artery enters the knee by
passing through the posterior capsule. It supplies the posterior capsule, synovial membrane and
the cruciates. The PCL is also covered by a synovial sleeve that is well vascularized and
augment the blood supply of the ligament. Finally, the distal portion of the PCL also receives
vascular supply from capsular branches of the inferior geniculate and popliteal arteries.
The PCL and its synovial sleeve are innervated by the popliteal plexus, which receives
contributions from the articular nerve (branch of the posterior tibial nerve) and from terminal
portionof the obturator nerve.
DeLee: DeLee and Drez's Orthopaedic Sports Medicine, 2nd ed., Copyright 2003 Elsevier, p.2085.

ANSWER

31. Which of the following risk factors for venous thromboembolism is associated with a
primary hypercoagulable state?
1 Inflammatory Bowel Disease
2 Obesity
3 Varicose Veins
4 Malignancy
5 Antithrombin III deficiency

The correct answer is 5. A primary hypercoagulable state is defined as hypercoagulability


diagnosed by DVT, presence of Antiphospholipid antibodies, or a number of other lab
evaluations without the presence of known malignancy, autoimmune disease or vascular injury.
Secondary hypercoagulable states are those acquired due to a disease state or condtion
Obesity, pregnancy, vascular injury, autoimmune diseases, and malignancies being most
common. The important distinction in this question is the difference between primary and
secondary. A person with Antithrombin III deficiency will be hypercoagulable due to the
condition of his blood regardless of other disease processes or conditions making it the
primary cause.
ANSWER

32.
A carpenter sustains a traumatic amputation through the distal interphalangeal
joint of his dominant ring finger from a table saw. The wound is allowed to heal by
secondary intention. One year after the injury, he reports extension of the finger when he
tightly grips a hammer. Treatment should consist of?
1) step-cut lengthening of the central slip.
2) spiral oblique retinacular ligament reconstruction
3) fourth and fifth dorsal interossei slides
4) release of the lumbrical tendon
5) tenodesis of the lacerated profundus tendon to the terminal tendon.

ANSWER

4:

Release of the lumbrical tendon.

From Miller pg 307: paradoxic extension of the IPJs while attempting to flex the finger. It
is caused by the retracted FDP tendon and lumbrical origin by laceration of the FDP tendon,
distal digital amputation, reconstruction of the FDP with a loose tendon graft, or adhesion of
the lumbrical to the FDP tendon distal to the MPJ. The lumbrical becomes a continuation of
the FDP tendon. When FDP contracts, it pulls on the extensor hood causing IP extension.
Treatment: lysis of the lumbrical tendon in the finger or by repair of the FDP to its insertion
under correct tension.

33. Which of the following finding is the best indication for the use of temporary external
fixation of a femoral shaft fracture?
12345-

Type IIIA open fracture


Hemodynamic instability
Segmental fracture
Distal one third fracture
Ipsilateral tibial shaft fracture

There are very few indications for the temporary use of an external fixation for a femoral shaft
fracture. The main exception is in the severely injured patient usually with an injury severity
score greater than 40. Although an intramedullary nail can be placed quickly, the average time to
place an Ex Fix has been reported to be only 30 minutes in one study (Nowotarski et al. 2000).
In this question, a patient with hemodynamic instability needs to be stabilized as quickly as
possible and an Ex Fix usually takes less time to apply.
Mohr and colleagues (1995) demonstrated osteomyelitis in 11% of patients with a Gustilo type II
or III fracture treated with Ex Fix and concluded that there was no advantage of using an external
fixator over an intramedullary nail for open fractures. Segmental and distal fractures, as well as
ipsilateral tibial shaft fracture, are not indication for placing an external fixator. Because the
femur has many muscle attachments upon which an external fixator can become tethered, placing
an Ex Fix is not without complications. Due to the good results of intramedullary nail fixation
for both closed and open femur fractures, it is usually the treatment method of choice.
ANSWER

-2

Browner BD, et Al. Skeletal Trauma, ed 3. 1898-1899.


OKU 2. 2000, 127-137.

34. A 12 month old girl has left congential fibular hemimelia type 1. The ipsilateral foot and the
contralateral lower extremity are normal. A scanogram shows a limb-length discrepancy of 2 cm,
with all the shortening in the left tibia. Assuming no treatment is rendered, the limb-length
discrepancy at skeletal maturity will most likely:
12345-

remain 2 cm
gradually increase to 5 cm
increase at a constant rate of 2cm with each year of growth
increase markedly because of the complete absence of growth in the left tibia
increase, with the left lower extremity remaining in similar proportion to the right lower
extremity

Fibular hemimelia is partial or total absence of fibula. It is most common long bone deficiency and the most
common skeletal deformity in the leg. Can be a complete absence of the fibula or a terminal absence of the fibula.
With absence of the terminal fibula, there may be an absence of the lateral rays of the foot. The involved limb is
usually shortened and the foot is in equinovarus position. It is usually associated with anteromedial bowing of the
tibia. Classically the skin has dimpling seen over the tibia. Also associated with ankle instability, tarsal coalition,
and femoral shortening. Limb length inequality and foot/ankle instability are the primary problems.
Treatment of the shortened limb is base on the severity of the shortening. For severe shortening or a nonfunctioning foot, amputation is the treatment of choice. For less severe cases, lengthening or reconstructive
procedure may be helpful.
Rules for limb-length discrepancy:

Rules for treatment

For Boys (up to age 16)


and Girls (up to age)
Distal femur 3/8 in/year
Proximal tibia - in/year

<2 cm observe with shoe lifts


2-5cm epiphysiodesis long side +
shortening or lengthening
>5 cm - lengthening

Proximal femur -

1/8 in/year

Congenital deficiency of the fibula is not an isolated malformation but rather a part of a spectrum
of dysplasia of the entire lower limb. The fibular deficiency ranges from partial absence to
complete absence. Type 1 fibular hypoplasia, Type 2- partial absence of the upper third, Type
3- complete absence of the fibular.
There are five patterns of progression of limb-lengthening discrepancy in children. Most lower
limb length discrepancies with congenital causes (ie fibular hemimelia) follow a type 1
proportionate progression pattern, they can be predicted. The current methods of predicting
limb- length discrepancy at skeletal maturity are applicable to only to type-1 proportionate
pattern.
Paley D, et al. Mutiplier method for predicting limb-lenth discrepancy. JBJS 2000: 82 ; 14321446
Morrey CF: A straight line graph for leg length discrepancies. Clin Ortho 1978; 136; 33-40
ANSWER

35. Figures 6a and 6b show the radiographs of a 43yo man who injured his right elbow in a
fall. Which of the following surgical approaches will provide the best exposure for open
reduction and internal fixation of the fracture
12345-

Lateral Kocher
Transolecranon osteotomy
Triceps Splitting
Triceps Reflection
Medial extensile

The radiograph demonstrates a distal humerus fracture with articular extension. This fracture
is fairly severe and requires exact reduction and fixation of the articular surface to avoid future
joint motion problems. The approach with afford the best opportunity of addressing both the
fracture as well as looking at the articular surface is the transolecranon osteotomy. With this
approach, a posterior incision is made in the midline proximal to the olecranon and curved lateral
to the olecranon process. Identify the ulnar nerve in the groove of the back of the medial
epycondile and protect it. A screw hole may be drilled at this point in the olecranon (if screw
fixation is going to be used for fixation of the osteotomy) prior to the osteotomy. The extensor
mechanism of the elbow is lifted along with the olecranon following the osteotomy to allow
visualization of the distal humerus articular surface. Dangers in this approach include the radial
nerve in the proximal portion of the incision, the ulnar nerve (if not properly protected). The
median nerve and brachial artery lie anteriorly to the distal humerus.
Explanation of Wrong Answers: 1- the posterolateral approach (kocher) is between the anconeus
and the ECU and is used as an approach to the radial head. It can be extended proximally into an
anterolateral approach to the distal humerus. However, it would not afford the same time of
visualization and opportunity for fixation as 2, especially with the medial extension of the
fracture. 3- Triceps Splitting: again, does not allow visualization of the articular extension. 4Triceps Reflection: Same as 3. 5- medial Extensile: This approach is performed proximally b/w
the brachialis and triceps and distally b/w the brachiallis and the pronator teres. It is used for
ORIF of cornoid process fractures and may be used for ORIF of fractures of the humeral condyle
and epycondyle but does not provide good visualization of the medial side of the joint.
Reference:
1- Morrey BF: Surgical exposure of the elbow; in Morrey BF (Ed) The Elbow and its
Disorders.
2- Hoppenfeld: Surgical exposures in Orthopaedics- The anatomic approach.
ANSWER

36. An 18-year-old female Olympic basketball player is seeking advice on how to prevent future
ACL injury. What recommendations can be made based on available scientific data?
1. use of custom-fitted derotational bracing
2. use of off-the-shelf derotational bracing
3. arthroscopic prophylactic notchplasty
4. plyometrics and neuromuscular training for 6 weeks
5. oral contraceptive use
Wojtys et al investigated the association between ACL tears and menstrual cycle phase. 69
females with ACL tears were evaluated within 24 hours of injury. Exam, menstrual cycle details,
OCP use, and urine samples were investigated. A higher than expected % of ACL tears occurred
during the midcycle (ovulatory phase), while a less than expected % occurred during the luteal
phase of the cycle. Use of oral contraceptive agents decreased the association between ACL tears
and the ovulatory phase.
Hewett et al studied the effects of a jump training program on landing mechanics and lower
extremity strength in female athletes involved in jumping sports. The program is designed to
degrease landing forces and increase vertical height by teaching neuromuscular control.
Following completion of the program, peak landing forces were found to decrease by 22%, while
knee adduction/abduction moments which are predictive of landing forces were found to
decrease by approximately 50%. Female athletes demonstrated lower landing forces vs. male
athletes following completion of the program. Hamstring-to-quadriceps torque ratios and
hamstring power increased with training and vertical jump height increased approximately 10%.
These results suggest that plyometric and neuromuscular training may beneficially effect knee
stabilization and help to prevent knee injury in female athletes.

References:
Wojtys EM et al: The effect of the menstrual cycle on ACL injuries on women as
determined by hormone levels. Am J Sports Med 2002; 30:182-88.
Hewett TF et al: Plyometric training in female athletes: decreased impact forces and
increased hamstring torques. Am J Sports Med 1996; 24:765-73.
ANSWER: 4

37. Figure 7 shows the clinical photograph of the knee of a chronic cigarette smoker 2 weeks
after revision total knee arthroplasty. Treatment should now consist of
12345-

ANSWER

irrigation and debridement with a polyehtylene insert exchange


removal of components and insertion of antibiotic cement spacer
debridement and a medial gastrocnemius flap.
debridement and a free flap.
knee arthrodesis.

Ries, Skin Necrosis after total knee arthroplasty. J Arthroplasty 2002: 17, 74-47.

38. A 20-year-old woman has progressive low back pain. An AP pelvic radiograph, CT scan,
posterior pelvic bone scan, and biopsy specimen are shown in Figure 8a through 8d.
Immunohistochemistry reveals CD99 reactivity but no staining for leukocyte common antigen.
What is the most likely diagnosis?
1. Ewings sarcoma
2. Lymphoma
3. Multiple myeloma
4. Langerhans cell histiocytosis
5. Osteomyelitis
The AP pelvis and the CT pelvis show a large, purely lytic geographic lesion in the right iliac
crest. This is a young female with progressive low back pain. Myeloma is the most common
primary bone tumor (35% of all bone tumors, 45% of malignant bone tumors). 70% of patients
with multiple myeloma are men but the average age of patients at presentation is 62 years.
Radiographically, the lesions are sharply punched-out holes or bubbly in appearance.
Ewings sarcoma is a highly malignant primary bone sarcoma that usually affects children and
young adults. Any bone can be involved, but it is usually in the long bones (55%), pelvis, and
ribs (25%). Permeative destruction is a major feature of Ewings sarcoma but can range from
motheaten permeative to geographic and purely lytic. Periosteal reaction is also quite common
(50%) with onion skinning appearance. Histologically, sheets of monotous, malignant round
cell are seen. Compared to lymphomas, they stain less intensely. Osteomyelitis typically
presents as an acute, sharp pain.

ANSWER

McCarthy EF, Frassica FJ: Primary bone tumors, in McCarthy EF, Frassica FJ (eds): Pathology
of Bone and Joint Disorders with Clinical and Radiographic Correlation. Philadephia, PA, WB
Saunders, 1998, pp 258-60.
Weis L: Common malignant bone tumors: Osteosarcoma, in Simon MA, Springfield D (eds):
Surgery for Bone and Soft Tissue Tumors. Philadelphia, PA, Lippincott-Raven, 1998, pp. 265-74.
Mirra JM: Bone tumors. Clinical, radiographic, and pathologic correlations. Philadelphia, PA,
Lea and Febiger, 1989.

39. Anatomic studies of the proximal tibia have shown that the capsular reflection of the knee
joint extends the farthest distal to subchondral bone at what location?
12345-

ANSWER

anterior to the fibula


posterior to the fibula
directly medial to the tibia
directly posterior to the tibia
anteromedial to the tibia

posterior to the tibia

Reid, et.al. Safe placement of proximal tibia transfixian wires with respect to the intracapsular
penetration. JOT 2001: 15: 10-17.
OKU 7 pp479-88.

40. Which of the following is considered a normal age-related change in the intervertebral disk
in the elderly?
12345-

ANSWER

Increased proteoglycon concentration


increased biosynthetic function
increased viable cell concentration in the central region
decreased water content
decreased stiffness

Increeased water

Buckwalter J, maintaining and restoring mobility in the middle and old age: the importance of
the soft tissues. ICL 1997, 46: 459-469.

41. A tall, thin 13 yr old girl with idiopathic scoliosis undergoes instrumented posterior spinal
fusion from T4 to L3. Postop recovery is uneventful, and a clear liquid diet is begun on the third
day. The patient responds with vomiting and is unable to tolerate any oral intake; however,
bowel sounds are normal. Nasograstric suction reveals large amounts of persistent bilious fluid
drainage. The patient is afebrile, with a normal peripheral WBC count and normal serum
amylase and lipase levels. What is the next most appropriate step in management?
1.
2.
3.
4.
5.

request psychological consultation and increase the patients sedative medications


aspirate the spinal wound and institute broad-sprectrum antibiotic therapy
obtain a myelogram and revise the implants to lessen the correction
obtain an ultrasound of the kidneys and perform an exploratory laparotomy
obtain an upper gastrointestinal study and institute parenteral feeding

Shapiro, et al, give a nice review of medical complications of scoliosis surgery in Current
Opinions in Pediatrics 2001;13:36-41. The complications described include SIADH,
pancreatitis, superior mesenteric artery syndrome, pneumo- hemo- chylo- thorax, fat embolism,
and ileus. The above clinical picture is most consistent with ileus, making answer 5 an
appropriate choice.
ANSWER

42. Management of Charcot Neuroarthropathy in the foot may include the use of
1) Aspirin
2) Bisphosphonates
3) Cephalosporins
4) Estrogen
5) Vitamin D

Ref: Jude EB, Selby PL, Burgess J, Lilleystone P, Mawer EB, Page SR, Donohoe M, Foster AV, Edmonds ME,
Boulton AJ.

Bisphosphonates in the treatment of Charcot neuroarthropathy: a double-blind randomised


controlled trial.
Diabetologia. 2001 Nov;44(11):2032-7.

This is one of my favorite journals. A prospective study in the UK comparing bone turnover in
Charcot feet in patient taking pamidronate vs placebo showed that pts with bisphosponates on
board did better.
ANSWER

43. A 23-year-old patient sustains an isolated right knee dislocation in a motor vehicle accident.
A closed reduction is performed and confirmed with radiographs. What is the next appropriate
study?
1 - CT of the knee
2 - MRI of the knee
3 - Radiographs of the femur and tibia
4 - Angiography of the leg
5 - Noninvasive assessment of lower extremity perfusion
Seems pretty intuitive. Knee dislocations are associated with injury to the popliteal artery. So reduce the knee and
make sure that there is good blood flow distally. Noninvasive assessment of lower extremity perfusion can be as
simple as feeling for pulses or using a doppler. If neither of these is able to appreciate a pulse then move onto
angiography.
mechanism of injury:
distinguish between high velocity injuries versus low velocity injuries (as this reflects
incidence of vascular and nerve injuries);
w/ low-velocity knee dislocations occurrence of vascular injury is about 5% and nerve
injury is about 20%
classification: 5 types: described w/ tibia in relation to femur
- anterior (31%)
occurs from hyperextension of knee (may need > 30 deg of hyperextension to produce
this injury);
often the PCL, & ACL will both be torn;
either the MCL or LCL or both will usually be injured;
alternatively, hyper-extension injuries may cause disruption of the ACL and posterior
capsule while the PCL is spared
popliteal artery is tethered proximally at adductor hiatus & distally by arch of soleus
injury to the popliteal artery may initially manifest as an intimal tear or intraluminal
thrombus (damage is over a longer segment of the artery);
- posterior (25%)
there is disruption of both cruciate ligaments
possible extensor mechanism disruption;
avulsion of or complete disruption of popliteal artery depending on magnitude of injury
more likely to produce localized injury (isolated transection)
- lateral (13%), medial ( 3%), rotary ( 4% - usually posterolateral)
Clinical Findings:
- popliteal artery & vein injury is common
- knee dislocations that have spontaneously reduced may look benign but may lead to thrombosis of
the popliteal artery
- peroneal nerve injury occurs in 20% to 40% of knee dislocations & approximately half of these
palsies are permanent
- note that apparent neurologic injury may in many cases be due to ischemia
- typically both cruciates and least one collateral ligament are disrupted
- w/ peroneal nerve injury, be highly suspect for vascular injury
- even if pulse returns following reduction, consider need for arteriogram, since incidence of intimal
injury is high w/ concomitant nerve injury

ANSWER

Abou-Sayed H, Beulger DL: Blunt lower extremity trauma and popliteal artery injuries. Arch Surg
2002;137:585589.
Miranda FE, Dennis JW. Veldenz HC, Dovgan PS. Frykberg ER: Confirmation of the safety and accuracy of
physical examination in the evaluation of knee dislocation for injury of the popliteal artery: A prospective study. J

Trauma 2002;52:247-252.

44. Figure 9 shows the radiograph of a patient who has anterior knee pain and range of motion
from 0-60 degrees after total knee arthroplasty. The implants are well fixed. A CT scan of the
femoral component shows 7 degrees of internal rotation, and the tibial component rotation is
centered over the medial third of the tibial tubercle. Treatement should consist of
12345-

ANSWER

lateral retinacular release


tibial tubercle transfer
revision of the femoral component
revision of the tibial component
proximal quadriceps realignment

revision of the femur

Kelly MA, Patellofemoral complications following total knee arthroplsty. ICL 2001; 50; 403-7.

45. Intramedullary screw fixation of a Jones fracture has a statistically higher failure rate in
1.
2.
3.
4.
5.

Elite athletes
Female patients
Patients who did not undergo bone grafting
Patients younger than age 40 years
Fracture that have been fixed with screws larger than 4.5 mm in diameter

----------------------------------------------------------------------Larson CM et al. Intramedullary screw fixation of Jones fx: analysis of failures. AJSM 2002;
30: 55-60.
15 Jones fx with cannulated screw fixation from 1993-1999. 6 failures: 4 refractures and
2 nonunions. There was a higher proportion of elite athletes (division 1 or professional level)
among the failure groups (83%) compared with those without complications (11%). Return to
full activity, especially amonth elite athletes, before complete radiographic union was predictive
of failure.
Glasgow MT, Torg JS et al. Analysis of failed surgical management of fractures of the base of
the fifth metatarsal distal to the tuberosity: The Jones fx. F&A Int 1996; 17: 449-457.
Paper presents 11 patients with failure of surgically managed Jones fx. Surgical
management complicated by delayed union in 3, refracture in 7, nonunion in 1. 6 procedures
were treated with intramedullary screw fixation and 5 with inlaid corticocancellous bone graft. . .
. 9 patients were male, 2 were female. 7 collegiate athletes, 3 professional athletes, and 1
recreational athlete.
ANSWER

:1

46. Figure l0a shows the radiograph of a 16-year-old boy who is being evaluated for an upper
respiratory infection. Further evaluation includes the CT scan shown in Figure l0b. A bone scan
does not reveal any other sites of skeletal involvement. A biopsy specimen is shown in Figure
l0c. What is the next most appropriate step in management?
1- Systemic antibiotics
2- Chemotherapy
3- Radiation therapy
4- Wide resection
5- Curettage

Chondrosarcoma is a lesion arising from the medullary cavity with an irregular calcification
matrix. It has been described as punctate, or popcorn, or comma shaped calcification. The
primary treatment for this is a primary wide resection with no adjuvant therapy. If pulmonary
mets are present, they should be resected as well. Chemotherapy has no role in the treatment of
this tumor.
ANSWER

Springfield OS, Gebhardt MC. McGuire MH: Chondrosarcoma: A review. Instr Course Lect
1996;45:417-424.
Fiorenza F. Abudu A, Grimer RJ, et al: Risk factors for survival and local control in
chondrosarcoma of bone. J Bone Joint Surg Br 2002;84:93-99.
Menedez LR (ed): Orthopaedic Knowledge Update: Musculoskeletal Tumors. Rosemont. IL,
American Academy of Orthopaedic Surgeons. 2002, pp 187-194.

47.

Figure 11 shows the clinical photograph of a patient who underwent knee arthroplasty 12
weeks ago and now has wound complications. Management consisting of serial
debridements, bacteria specific-antibiotics, and local dressing changes has failed to
resolve the problem. Examiniation reveals that the patient is afebrile and has a normal
WBC count. In addition to continued use of antibiotics, management should include
12345-

retention of the total knee components and split-thickness skin grafts.


polyethylene exchange and split-thickness skin grafts.
removal of the total knee components, a local rotation flap, and split-thickness
skin grafts.
primary knee fusion and split-thickness skin grafts.
amputation.

The photograph demonstrates full thickness soft tissue necrosis. Recall that split thickness skin
grafts require an adequate bed, which is not present here. Thus this knee will require some sort
of rotational flap, onto which the split thickness skin graft can be placed. Additionally the
review by Dennis recommends removal of the arthroplasty components since significant skin
infections most often affect the components as well. This is well past the 4 week deadline for an
acute postoperative arthroplasty infection and should be treated as such.

ANSWER

removal of the total knee, rotation flap, and STSG

Nelson CL: Primary and delayed exchange for infected total knee arthroplasty. Am. J. Knee
Surg 2001: 14: 60-64.
Dennis DA: Wound complications in total knee arthroplasty. ICL 1997; 46: 165-69.

48. Collagen orientation in the deep layer of normal articular cartilage is aligned
1 randomly
2 parallel to the surface, not crossing the tidemark
3 parallel to the surface, crossing the tidemark
4 perpendicular to the surface, not crossing the tidemark
5 perpendicular to the surface, crossing the tidemark
Articular cartilage consists primarily of a large extracellular matrix (ECM) with a sparse
population of highly specialized cells (chondrocytes) distributed throughout the tissue. The
primary components of the ECM are water, proteoglycans, and collagens, with other proteins and
glycoproteins present in lower amounts. These all combine to provide the tissue with its unique
and complex structure and mechanical properties.
The structure and composition of the articular cartilage vary throughout its depth, from
the articular surface to the subchondral bone. These differences include cell shape and
volume, collagen fibril diameter and orientation, proteoglycan concentration, and water
content. The cartilage can be divided into 4 zones: the superficial zone, the middle or
transitional zone, the deep zone, and the zone of calcified cartilage.
The superficial zone is the uppermost zone of the cartilage and forms the gliding surface.
The thin collagen fibrils are arranged parallel to the surface, the chondrocytes are elongated
with the long axis parallel to the surface, the proteoglycan content is at its lowest level, and the
water content is at its highest level. The middle, or transition, zone contains collagen fibers
with a larger diameter and less apparent organization, and the chondrocytes have a more
rounded appearance. The deep zone contains the highest concentration of proteoglycans and the
lowest water content; the collagen fibers have a large diameter and are organized
perpendicular to the joint surface. The chondrocytes are spherical and often are arranged in a
columnar fashion. The deepest layer, the zone of calcified cartilage, separates the hyaline
cartilage from the subchondral bone. It is char-acterized by small cells distributed in a
cartilaginous matrix encrusted with apatitic salts. Histologic staining with hematoxylin and eosin
shows a wavy bluish line, called the tidemark, which separates the deep zone from the calcified
zone. The perpendicular collagen fibers in the deep zone cross the tide mark and are secured in
the calcified cartilage layer thus imparting resistance to shear stress at the bone-cartilage
interface.
able 1

ANSWER

Orthopaedic Basic Science,


AAOS 2nd Edition
Browne JE: Surgical alternatives
for treatment of articular
cartilage lesions.
JAAOS 2000;8:180-189.

49. A patient with neutral ulnar variance sustains an extra-articular fracture of the distal
radius that heals with normal palmar tilt but with loss of radial height. Resultant ulnar
variance is measured at +3mm. What percent of load transmission across the wrist will
now be borne by the ulna?
6- 10%
7- 25%
8- 40%
9- 75%
10- 90%

The kinematics of the normal wrist (Palmer and Werner, CORR 187: 26-35, 1984) as measured
with an experimental model show that the radius (through its articulation with the lateral carpus)
carries approximately 82% of the axial load of the forearm, and the ulna (through its articulation
with the medial carpus) carries approximately 18%. Further studies were done to evaluate the
change in the load borne by the ulna with +/- variance (whether surgical or the result of
shortening/relative lengthening after fracture). Changes in the ulna length dramatically altered
the amount of force borne by the distal ulna. Shortening of the ulna 2.5 cm resulted in a decrease
from 18 % to just 4%. Lengthening by 2.5 cm resulted in an increase from 18% to 42%. Thus, I
quote, When a Colles fracture settles 2.5 mm, one can expect and increase in ulnar axial load of
approximately 40%.
ANSWER

50. The greatest increase in compartment pressure during reamed intramedullary nailing of a tibial shaft
fracture occurs with the use of
1. an awl for entry portal creation
2. a tourniquet
3. a large intramedullary nail
4. a static locking construct
5. continuous traction

Intramedullary nailing is the treatment of choice for most tibial shaft fractures. A nail is effective at
controlling bending and lateral displacement. Because the majority of the tibial medullary canal is relatively
straight, a nail restores axis alignment for fractures in the diaphysis. There is immediate loss of medullary
arterial flow with a some bone necrosis around the nail. To compensate, the periosteal blood supply assumes a
larger role in perfusing the cortex. The medullary arterial system usually regenerates within a few weeks.
There is significantly less cortical necrosis with a loosely fitting intramedullary nail than with a snug nail after
reaming. Nailing provides excellent stability for most tibial shaft fractures. Closed nailing after reaming
offers fracture control for weight bearing with a low risk of infection or other treatment complications.
Reamed intramedullary nails offer better stability and strength, and thus earlier unsupported weight
bearing. For uncontaminated and uninfected delayed unions and nonunions, reamed nailing usually provides
an excellent and reliable functional treatment, if the fracture is appropriately located and the nail can be
inserted with a closed technique or with minimal soft tissue stripping. Nonreamed intramedullary nails can
be used for closed tibial fractures and have been advocated for use in closed fractures with significant soft
tissue injury or compartment syndrome. However, compared with larger-diameter nails designed for insertion
with reaming, their smaller-diameter shafts, proportionately larger locking screw holes, and smaller locking
screw diameters typically result in reduced ultimate strength and shorter fatigue life.
Fractures that need immediate surgery for soft tissue considerations may be treated immediately with
intramedullary nails. It is important to be aware that restoration of length decreases muscular compartment
volume, which may precipitate a compartment syndrome, especially if fascial compartments remain intact.
Tibial shaft fractures have a known incidence of compartment syndrome. The injury itself is usually the
main cause. There is an increase in compartment pressure when the canal is reamed or when an unreamed
nail is inserted. These quickly return to baseline levels. An awl for entry, tourniquet, large nail or static
locked nail apparently do not cause as much increase in compartment pressure as traction. Traction decreases
the compartment volume which can lead to increased pressure. A thigh post has also been demonstrated to
elevate compartment pressures.
ANSWER

5.

continuous traction

References:
Browner Jupiter: Skeletal Trauma, 1998, pp 2226-38.
Nassif, Effect of Acute Reamed vs Unreamed Intramedullary Nailing on Compartment pressure
when Treating Closed Tibial Shaft Fractures: A Random Prospective Study; JOT 14, 2000: 5548.

51. Which of the following are known predictive factors for curve progression in idiopathic
scoliosis?
1.
2.
3.
4.
5.

Peak height velocity, female gender, open triradiate cartilage, premenarchal status
Peak height velocity, male gender, open triradiate cartilage, preadrenarchal status
Peak height velocity, female gender, closed triradiate cartilage, premenarchal status
Peak height velocity, female gender, open triradiate cartilage, postmenarchal status
Family history, female gender, open triradiate cartilage, premenarchal status

Neither paper really addresses the question, but we know that being female is predictive. Earlier
age of diagnosis (open growth plates) means that there is more time for the disease to progress,
as it slows down significantly after the growth plates fuse. Premenarchal status follows the
above two. And peak height velocity is the period where the greatest progression is expected.
Menarche occurs approximately 7 months after PHV. Growth is complete approximately 2 years
after menarche.
Little DG, Song KM, Katz D, Herring JA: Relationship of peak height velocity to other maturity
indicators in idiopathic scoliosis in girls. JBJS 82(A): 685-693
Lowe TG, Edgar M, Margulies JY, et al.: Etiology of idiopathic scoliosis: Current trends in
research. JBJS 82(A): 1157-1168
ANSWER

52. What muscle is at greatest risk for denervation when the posterior capsule is released during
a rotator cuff repair?
12345-

Deltoid.
Subscapularis.
Biceps.
Supraspinatus.
Infraspinatus.

Warner JP, Krushell RJ, Masquelet A, Gerber C: Anatomy and relationships of the
suprascapular nerve: Anatomical constraints to mobilization of the supraspinatus and
infraspinatus muscles in the management of massive rotator-cuff tears. J Bone Joint Surg Am
1992;74:36-45.
Explanation:
The above reference is an anatomic study of 31 shoulders that examined the anatomy of
the neurovascular supply of the supraspinatus and infraspinatus muscles and to assess the risk for
injury of the neurovascular pedicle when the rotator cuff is mobilized by a standard
anterosuperior approach. The authors felt that some of the failures in restoration of normal
power of external rotation and abduction after massive cuff repair could be attributed to injury to
the suprascapular neurovascular pedicle (which supplies the supraspinatus and infraspinatus
muscles) during mobilization of the cuff rather than a failure of the tendon repair itself.
In their study, the authors found that in mobilizing a torn retracted cuff (i.e., the
supraspinatus and infraspinatus muscles), only one additional centimeter of lateral mobilization
was possible. This was limited by the neurovascular structures in the supraspinatus and
infraspinatus fossae. The suprascapular nerve runs an oblique course across the supraspinatus
fossa and is fixed to the floor and underneath the transverse scapular ligament. In their study, the
average distance from the posterior rim of the glenoid to the motor branches of the infraspinatus
muscle was 2 centimeters. Thus, in mobilizing the infraspinatus muscle and posterior capsule,
the surgeon only has 2 centimeters before hitting the nerves to the infraspinatus muscle. Thus
the preferred response is 5. The average distance from the origin of the long tendon of the
biceps to the motor branches of the supraspinatus was 3 centimeters (#4 is wrong), and the motor
branches to the supraspinatus muscle are fewer, usually smaller, and significantly shorter than
those to the infraspinatus muscle.
ANSWER

53. Within the functional spinal unit, the nucleus pulposus functions:
1. to resist compressive loads
2. to resist tension loads
3. to resist shear loads
4. independent of the annulus fibrosis
5. independent of the facet joints

Motion segments in the spine are comprised of two vertebral bodies and an intervertebral disk.
The four regions of the disk include: the cartilaginous end plate, the outer annulus fibrosus, the
inner annulus fibrosus, and the nucleus pulposus. The dense collagenous cross-ply,
circumferential lamellae of the outer annulus fibrosus resist large tensile stresses, minimize disc
bulging, and reduce the strains that are developed during axial compressive and torsional
loading, sagittal and transverse bending, or axial torsional loading of the spine. This includes
shear loads on the spine.
The nucleus pulposus provides a hydrostatic barrier that limits the deformation of the disc. The
high degree of hydration (70 to 80%) allows the nucleus pulposus to resist compressive loads
while being contained by the outer annulus.
Disc mechanical function depends not only on the annulus fibrosus and nucleus pulposus, but
also on the cartilage end plate. Motion segment studies suggest that the cartilage end plate
undergoes significant compressive deformation. Although morphologically distinct from the
annulus and nucleus, the cartilaginous end plate functions in conjunction with the two structures
to maintain uniform stress distrubution across the boundary between the vertebral body and the
disk.
ANSWER

54. A 52 year-old man sustains a displaced bimalleolar (Weber B) ankle fracture in a fall. History
reveals that he has had type I diabetes mellitus for the past 30 years. Examination reveals a
closed injury with mild swelling, palpable pulses, and decreased sensation to the midcalf level.
Management should consist of
1. closed treatment with a short leg cast
2. percutaneous pin-in-plaster technique
3. external fixation spanning the ankle
4. amputation
5. open reduction and internal fixation of the medial and lateral malleoli
The treatment of ankle fractures in diabetics can be fraught with complications including
delayed wound healing, surgical infection, and Charcot arthropathy. For this reason, and the
somewhat sparse literature comparing the clinical outcomes of non-operative versus operative
intervention in displaced ankle fractures, there has been some debate as to the best way to handle
these fractures in diabetics.
In the article Complications of Ankle Fractures in Diabetic Patients Orthopedic Clinics
of North America. 32 (1) Jan. 2001, the authors admit the lack of consensus in treating these
fractures, but recommend the following:
A. Treatment of Nondisplaced Ankle fractures in diabeticsthe nondisplaced fracture in
the diabetic typically heals uneventfully with cast immoblization followed by bracing.
The authors suggest a protocol of cast immobilization for a minimum of 8 weeks nonweight bearing. After this, the patients follow a protocol of brace immobilization in
DM short leg walker or molded, lined, and bivalved AFO for another 8-16 weeks with
progressive weight bearing. They futher recommend an increase of 25lb at 2 to 4
week intervals until 75% of the patients weight is achieved.
B. Treatment of Displaced Ankle Fractures in diabeticsSchon et al. (Clin.Orthop
349:116-131, 1998) described a series of 13 pre-Charcot displaced ankle fractures.
Four ankles where treated non-operatively and nine underwent ORIF. After a
minimum of 3 months in cast or brace, the non-operative treatment resulted in
significant varus deformity, non union, or both in all cases, requiring arthodesis or
ORIF. Conversely, those undergoing ORIF were managed with NWB and
immobilization for 8-12 weeks. These ankles showed only 1 in 7 progressive
displacement. While this series is small, the authors similarly recommend operative
intervention of displaced ankle fractures in diabetics. They contend that despite the
potential for possible delayed wound healing, infection and loss of fixation secondary
to osteopenia, articular alignment is more important to help minimize residual
deformity and the possibility of developing Charcot arthropathy. Surgical fixation
involves ORIF using 1/3 tubular plate lateral and 3.5/4.0/4.5 screw medially.
Additional stabilization consists of syndesmotic screws. After surgery, NWB and cast
immoblization are recommended for 8 weeks, with cast changes every 2 to 4 weeks.
A removable DM CAM walker or molded plastizote lined and bivalved AFO is used
with weight bearing increased 25 lb.at 2 to 4 week intervals for another 8 to 12 weeks
until 75% of the patients weight is reached. Then full weight bearing allowed.
ANSWER

55. Treatment of complex regional pain syndrome of the knee includes


1) Knee manipulation
2) Passive range of motion exercises
3) Beta-blocking agents
4) Alpha-blocking agents
5) Parasympathetic blockade

The treatment of reflex sympathetic dystrophy can include sympathetic blockade (not
parasympathetic), physical therapy (active and active-assisted ROM exercises) and
pharmacologic agents. Drugs that have shown some effectiveness in treating sympathetically
maintained pain include alpha-blockers, antidepressants, and anticonvulsants. Narcotics and
benzodiazepines are not recommended, as they can lead to dependence, depression, and
increased pain.
ANSWER

56: Based on the radiographic findings shown in Figures 12a and 12b, the patient is most likely
to have defects in what other organ systems?
1.) Visual
2.) Endocrine
3.) Renal
4.) Hematologic
5.) Digestive

Klippel-Feil syndrome is a congenital fusion of the cervical vertebrae that may involve two
segments, a congenital block vertebra, or the entire cervical spine. Congenital cervical fusion is a
result of failure of normal segmentation of the cervical somites during the third to eighth week of
life. The skeletal system may not be the only system affected during this time, and
cardiorespiratory, genitourinary, and auditory systems frequently are involved. In most patients
the exact cause is not known. One proposed cause is a primary vascular disruption during
embryonic development that results in fusion of the cervical vertebrae and other associated
anomalies. Beals and Rolfe suggested that a global insult with variable effects on different
tissues or multiple separate insults could explain the fusions of the cervical vertebrae and other
associated anomalies. Gunderson et al. showed that in a few patients this is an inherited
condition. His study suggests that fusion of C2-C3 may be an autosomal dominant inheritance,
but the inheritance pattern of other cervical vertebral fusion patterns could not be determined.
Maternal alcoholism also has been suggested as a causative factor. Tredwell et al. found a 50%
incidence of cervical vertebral fusions in roentgenograms of patients with fetal alcohol
syndrome. About one third of patients with Klippel-Feil syndrome have urogenital anomalies.
Because the cervical vertebrae and genitourinary tract differentiate at the same time and location
in the embryo, fetal maldevelopment between the fourth and eighth weeks of development may
produce both genitourinary anomalies and Klippel-Feil syndrome. These renal anomalies usually
are asymptomatic, and children with Klippel-Feil syndrome should be evaluated with an
ultrasound or intravenous pyelogram because the renal problems can be life threatening. The
most common renal anomaly is unilateral absence of the kidney. Other anomalies include
malrotation of kidneys, ectopic kidney, horseshoe kidney, and hydronephrosis from ureteral
pelvic obstruction.
Canale: Campbell's Operative Orthopaedics, 10th ed., Copyright 2003 Mosby, Inc. 1737-1738.

ANSWER

57. A multiply injured patient sustains a closed midshaft femoral fracture in a motor vehicle
accident. An external fixator is applied for stabilization the day of injury because of
hemodynamic instability. Four days after injury, the patient is stable and the pin sites are clean.
Treatment should now consist of
1 continued external fixation
2 removal of the external fixator and plate fixation
3 removal of external fixator and intramedullary nailing
4 removal of external fixator and traction
5 removal of external fixator and curettage of the pin tracts, followed by intramedullary
nailing 2 weeks later

The correct answer is 3. As with any femur fracture IM nailing is the definitive management of
this fracture. In a multiply injured patient, quick methods of temporary fixation may be
necessary due to the patients overall health status. In an Article from JBJS 2000, from the
Shock Trauma Hospital, a retrospective review involving patients who were initially stabilized
and then were converted to definitive fixation at a later time was performed to evaluate their
algorithm for treatment. The study incorporated 49 fractures which represented 4% of femur
fractures over an eight year period. All fractures were initially stabilized within 24 hours and
then subsequently taken back for definitive repair from 1 to 49 days later. 45 of 49 had a single
stage procedure for ex fix removal and IM nailing. Four of the remaining fractures, required
debriedment 8 to 15 days prior to IM fixation. All of these four fractures had gross purulent
drainage from the pin sites. Infection rates in the 45 single stage procedures were 1.7%. In
summary, the authors recommend single stage external fixator removal and IM nailing as long as
there is no pin tract infection noted on clinical exam. Choice 5 would be correct if the stem
spoke of gross purulent drainage.
ANSWER

58. Wrestler with swelling & redness over the anterior knee after abrasion 2 weeks ago. No
knee effusion and active ROM 0-100 degrees. Radiographs show some soft tissue
swelling anterior to the patella. Management should consist of:
1.
2.
3.
4.
5.

NSAIDS
physical therapy
MRI
corticosteroid injection
aspiration

A university of Iowa study showed prepatellar bursitis is easily diagnosed. There is usually
history of trauma but the exact incident is often unknown. Swelling is superficial to the
patella and the posterior knee can be normal. SEPTIC CASES ARE OFTEN
ASSYMPTOMATIC! However, there may be local evidence of infection. In the Iowa study
of 136 wrestlers, 13 developed initial prepatellar bursitis. 5 had no recurrences but the other
8 had 20 recurrences. Half of 6 cases in 4 wrestlers with septic prepatellar bursitis had no
clinical signs of infection at presentation. Therefore it is necessary to aspirate and get a gram
stain of the fluid. All but one infection had penicillin resistant staph aureus. Treatment
consists of rest, immobilization, padding, aspiration, compressive dressings, steroid injection,
and surgery (Antibiotics for infected cases of course). Steroid injection appeared ineffective.
This injury is more frequent in lighter weight classes and during the off season.
Aspirate them all, start antibiotics if clinically infected or positive cultures/stains.
ANSWER

59. A 28-year-old man underwent surgical fixation for an intra-articular distal humeral
fracture 8 weeks ago now reports progressively restricted elbow motion. Radiographs at
the time of union are shown in Figures 13a and 13b. Management should now consist of
12345-

oral indomethacin.
irradiation with a single dose of 700 cGy.
physical therapy with dynamic splinting.
physical therapy and delayed ectopic bone excision at 12 months.
immediate elbow release and ectopic bone excision.

The radiographs demonstrate an AP and lateral of the elbow s/p ORIF distal humerus fracture
with significant heterotopic bone over the anterior aspect of the distal humerus. Regan and
Reilly have described three factors that contribute to posttraumatic elbow stiffness:
1. high degree of articular congruity and conformity of the elbow
2. predisposition of periarticular ossification of the brachialis
3. difficulty in achieving rigid internal fixation requiring lengthy post-op mobilization
The options for posttraumatic elbow arthritis are splinting, arthroplasty, interposition
arthroplasty, distraction arthroplasty elbow arthrodesis, and simple elbow contracture release.
Indomethacin and radiation therapy, although helpful in preventing heterotopic bone formation
would not be useful in this patient who already has significant bone formed with restricted
motion.
The recommendations on the timing of excision of the ectopic bone have changed within the last
ten years. Originally surgeons advocated delaying contracture release one to 2 years to allow
bone maturation. Surgeons advocated waiting until bone scans and serum alkaline phosphatase
were normal before heterotopic bone was removed. In 1999 Viola et al. published a prospective
study of early simple elbow contracture release followed by a short course or NSAIDs and
physical therapy with good results. Early elbow release prevented prolonged soft tissue
contracture with the potential for articular cartilage degeneration and is now the standard of care.
ANSWER

5.

References:
Viola RW, Hanel DP: Early simple release of prostraumatic elbow contracture associated with
heterotopic ossification. J Hand Surg AM 1999;24:370-380
Koval KJ (ed): OKU &. Rosemont, IL, AAOS, 2002, pp 407-416, pp 307-316.

60. What is the most significant mechanism by which a cell controls its phenotype?
12345-

Protein degradation
MRNA degradation
Translation efficiency
Transcription
Posttranscriptional mRNA processing

There are 6 potential control points for gene regulation:


1. initiation of transcription
2. posttranscriptional processing of the mRNA (i.e. splicing)
3. mRNA degradation
4. efficiency of translation
5. posttranslation processing
6. protein degradation.
The most significant among these control mechanisms is initiation of transcription.
This is
the most sensible of the control points because the cell does not waste energy
synthesizing a useless RNA. Also, multiple genes may be required simultaneously,
and a common control mechanism allows for coordinate expression at the
appropriate time and place. Although initiation of transcription is the major control
point for the expression of many genes, this form of control does not exclude others
from providing a cell with the ability to fine-tune the expression of a particular gene.

Although examples of regulation at each of these steps in gene expression have been found in,
control of transcription initiation- the first step- is the most important mechanism for determining
whether or not most genes are expressed and how much of the encoded mRNAs, and
consequently proteins, are produced.
Buckwalter JA, et al: Orthopaedic Basic Science ed 2. Rosemont IL AAOS, 2000, pp 19-76
ANSWER

61- A patient with CMC joint arthritis of the thumb undergoes trapezium excision and
interposition arthroplasty. One year after treatment, radiographs reveal that there has
been 25% subsidence of the thumb metacarpal compared with preoperative height. This
degree of subsidence will have what effect on surgical outcome?
12345-

Will not affect functional outcome


Will result in diminished thumb motion
Will result in diminished pinch strength
Will result in diminished grip strength
Will result in moderate activity-related pain

Answer: 1- will not affect functional outcome


Excisional arthroplasty has been used effectively in relieving pain associated with basal
joint arthritis and preserving thumb motion. One complication of the procedure has been loss of
thumb strength and stability. In an effort to improve thumb function and prevent proximal
migration, operative methods have been developed to reconstruct the ligaments b/w the base of
the thumb MC and the index MC (the palmar oblique ligament). Tendon interposition was
developed for that purpose using the FCR ligament.
The surgery involves a posterior approach between the APL and EPB with incision of the
capsule and excision of the trapezium. 12 cm of the FCR is then obtained and pulled through
drill holes in the base of the thumb MC and tied onto itself while the thumb MC is maintained in
the abducted fist position with a K-wire. The remainder of the FCR is then balled up and placed
in the now empty trapezium fossa as a spacer. The patient is then immobilized in a thumb spica
for 4 weeks.
Two studies (listed below) looked at the results of this procedure. The findings were that
the main trapezial space ratio decreased by about 33%. When analysis was performed as to the
effect this had on grip strength, pinch strength, pain and thumb motion, there was found to be no
correlation between this loss of height and those four factors.
The conclusion is that tendon interposition is helpful in restoring thumb motion, pain
relief and strength. However, the secondary goal of reconstructing the palmar oblique ligament
is not achieved by this procedure although this has no affect on clinical outcome.
ANSWER

1- Yang SS, Weiland AJ: 1st MC subsidence during pinch after ligament reconstruction and
tendon interposition basal joint arthroplasty of the thumb. J Hand Surg Am 1998; 23:879883.
2- Tomiano MM, Pelligrini VD, Burton RI: Arthroplasty of the basal joint of the thumb:
Long term follow-up after ligament reconstruction and tendon interposition arthroplasty.
J Hand Surg Am 1996; 21:202-9.

62. Early failure of a unicompartmental knee arthroplasty that is the result of polyethylene wear
is primarily caused by:
1. malalignment
2. instability
3. metal backing of the tibial component
4. gamma irradiation sterilization and shelf storage in air
5. obesity
McGovern et al reported that the mechanical toughness of polyethylene sterilized by gamma
irradiation in air decreases after a long shelf life. 75 unicondylar knee replacements were
followed clinically and radiographically. 30 knees were revised at a mean of 18 months.
Components retrieved at time of revision were examined for presence of oxidation. The rate of
polyethylene wear increased as the shelf life increased. An inverse correlation was noted between
the shelf life of the polyethylene and the time of revision. 7 components fractures and 10 had
fractured and fragmented. The study shows the early and severe wear of tibial polyethylene
gamma-sterilized components stored for < 4.4 years.

References:
McGovern TF et al: Rapid polyethylene failure of unicondylar tibial components
sterilized with gamma irradiation in air and implanted after a long shelf life. JBJS Am
2002; 84:901-06.
ANSWER: 4

63.

Figures 14a and 14b show the MRI scans of a 38-year-old man who has an enlarging,
painless mass in the left distal thigh. A biopsy specimen is shown in Figure 14c. What is
the most likely diagnosis.
1.
2.
3.
4.
5.

Lipoma
Desmoid tumor
Fibrosarcoma
Angiosarcoma
Liposarcoma

Most liposarcomas appear well defined on MRIs, mostly with lobulated margins. Welldifferentiated liposarcomas are mainly composed of fat with septations or nodules and are
hyperintense on T2-weighted images. After the administration of contrast material, welldifferentiated liposarcomas may enhance minimally or not at all.
Looking at the T1 and T2 images we see a lobulated mass, that is heterogeneous,
juxtacortical, and very large. It is mostly dark on T1 (which helps rule out lipoma). It is
hyperintense (enhances) on T2 weighted images due to its high cellularity (which we can see on
the histologic section). You can also see some adipose-like cells on the histologic section (the
larger vacuolated cells). Due to the MRI and histologic picture liposarcoma is the one best
answer to this question. Furthermore, although no real history is given, the fact that the mass is
painless is also a key since most liposarcomas are painless.
FYI: Liposarcoma is second in frequency only to malignant fibrous histiocytoma (MFH)
among the soft-tissue sarcomas. It occurs almost exclusively in adults and is found most often in
the thigh or retroperitoneum. It rarely arises from a lipoma and does not occur in the
subcutaneous tissues. Liposarcomas are divided into histologic subtypes with different
microscopic appearances and slightly different behavior characteristics. Surgical resection with
a wide surgical margin is the treatment of choice. When amputation is required to obtain an
adequate surgical margin, local irradiation can be used as an adjuvant and a limb-sparing
operation can be done. Currently, no evidence exists that adjuvant chemotherapy is indicated for
patients with liposarcoma, although numerous studies are being done to investigate its use. As is
the case for all sarcomas, the lung is the most common site of metastasis; however, liposarcoma
has an unusual propensity to metastasize to the retroperitoneum, mediastinum, and bone.
ANSWER

: 5

Grainger & Allison's Diagnostic Radiology: A Textbook of Medical Imaging, 4th Ed


Springfield, D.:
(289). P 50-7.

Liposarcoma.

Clin-Orthop Related Research.

1993 Apr.

64. Which of the following findings is one of the diagnostic criteria for diffuse idiopathic skeletal
hyperostosis?
1. Flowing osfficiation along the anterolateral aspect of at least four contiguous vertebrae
2. Disk space collapse in the involved vertebral segments
3. Marginal syndesmophytes over four contiguous vertebrae
4. Sacroiliac erosion or sclerosis
5. Facet joint ankylosis

Diffuse idiopathic skeletal hyperostosis (DISH) is a common disorder characterized by back pain
and spinal stiffness. In older individuals, the prevalence has been shown to be as high as 1525%.The usual presentation is a middle-aged or older patient with chronic mild pain in middle to
lower back, spinal stiffness, and typical radiographic findings in the thoracic spine. Tendinitis
may be present and dysphagia is an occasional complication. The diagnostic criteria for DISH
are:
1. Flowing ossification along the anterolateral aspect of at least four contiguous
vertebrae.
2. Preservation of disk height in the involved vertebral segment; the relative absence of
significant degenerative changes, such as marginal sclerosis in vertebral bodies or
vacuum phenomenon
3. Absence of facet-joint ankylosis; absence of sacroiliac erosion, sclerosis, or intraarticular osseous fusion.
The first criteria is the essence of DISH while the other two criteria serve to eliminate the
differential diagnosis of spondylosis of degenerative spinal disorders and ankylosis spondylitis
(AS) as alternative diagnoses.
ANSWER: 1
Belanger TA, Rowe DE: Diffuse idiopathic skeletal hyperostosis: Musculoskeletal
manifestations. J Am Acad Orthop Surg 2001; 9: 258-67.

65. An 18month old child has bilateral corner fractures of the distal femoral metphyses of
unknown origin. Following a skeletal survey, the first step in management should consist of
12345-

ANSWER

Notification of child protective services


bilateral long leg casts and discharge
bilateral percutaneous pinning, long leg casts, and sicharge
hospital admission and Bryants traction
hosptial admission and modified Bryants traction

notifiactioan of protective services

Loder RT, et.al. Fracture patterns in battered children. JOT 1991; 5; 428-433.

66. What is the most common location for longitudinal peroneus brevis tendon tears?
12345-

ANSWER

musculotendinous junction
proximal to the fibular groove
at the fibular groove
At tand below the peroneal tubercle of the calcaneus
At the point of insertion on the fifth metatarsal

at the fibular groove

\Sanmarco GJ, et.al. Chronic peroneus brevis tendon lesions. Foot Ankle 1989; 9; 163-70.

67. Figures 15a through 15c show the plain radiograph, CT scan, and biopsy specimen of a 23
yo man with acute onset hip pain. History reveals that he underwent a trochanteric advancement
at age 6 yr. What is the most likely diagnosis?
1.
2.
3.
4.
5.

osteomyelitis
lymphoma
eosinophylic granuloma
tuberculosis
Ewings sarcoma

The xray shows a geographic lytic lesion in the proximal femur. The CT shows serpiginous
tracts and irregular areas of bone destruction and reactive formation along with a walled off area.
The path slide shows mixed cells with a predominance of polymorphonuclear cells. The xray is
nonspecific. The CT is consistent with chronic osteomyelitis- the walled off area is a
sequestrum. The path supports the CT, as does the remote history of surgery in the region.
ANSWER

The reference given is NEJM 1997;336:999-1007.

68. A 54 year old man sustains a patellar fracture in a fall. Examination reveals that the patient
can perform a straight leg r3aise on the injured side. Xrays reveal a transverse fracture with 2
mm of displacement of the articular surface and no comminution. Management should consist of
1)
2)
3)
4)
5)

Immobilization in a long leg cast and no weight bearing


Immobilization with knee straight and full weight bearing
No immobilization and full weight bearing
Tension band wire construct
Lag screw fixation

Ref Rockwood and Green


Patient with a closed patella fracture but with no extensor lag. No extensor lag, no need for
surgery. Answer 3 is not desirable the fracture will not heal optimally with no immobilization.
So the answer is 1 or 2. The best answer is 2. With the knee immobilized straight, weight
bearing is fine. Although technically, answer 1 would suffice, the guy would still heal. But
answer 2 is better because ( in my opinion) because the it specifically says immobilization in
extension.
ANSWER

69. In which of the following types of studies can an incidence rate be determined?
1- Cross-sectional
2- Prospective cohort
3- Case control
4- Randomized trial
5- Retrospective review

Incidence: The number of new cases of the disease or condition that arise during a specific time
interval.
Incidence rate: The number of new cases divided by the population at risk but without the
disease. It gives a far better estimation of the significance of exposure
Cross-sectional study: Evaluates a group at one point in time. Often described as a snap-shot
of a population or sample. Causal links between diagnosis and variables are found, but no
conclusions are drawn
Prospective Cohort Study: A group of disease free subjects is followed over time to identify
onset of disease or injury. This type of study identifies incidence. Cohort studies are useful for
establishing relative risk.
Case Control: Subjects in the study group are chosen based on the presence of an injury or the
presence of disease. The group is then analyzed to identify associated, clinically relevant
information. Case-control studies by definition begin with a population and its disease and
cannot be evaluated prospectively to determine exposure.
Randomized trial: An experiment arranged to produce a chance distribution of subjects into
different treatment or control arms. Randomization is done to cancel out the influence of factors
that are not under study. With successful randomization, the chance of one individual being
placed in a given study arm is independent of the placement of others, and the only differences
between the groups are those intended by the experimenter (e.g., different drugs taken).
Retrospective review: This type of review is conducted after treatment is rendered and assesses
the services provided on a case-by-case or aggregate basis.
ANSWER

2:

Buckwalter JA, Einhorn TA, Simon SR (eds): Orthopaedic Basic Science: Biology and
Biomechanics of the Musculoskeletal System, ed 2. Rosemont, IL, American Academy of
Orthopaedic Surgeons, 2000, pp 2-17.
Janssen HF: Experimental design and data evaluation in orthopaedic research. J Orthop Res
1986;4:504-509.

70. Which of the following factors is considered predictive of poor wound healing potential
following lower extremity amputation?
12345-

ANSWER

serum albumin level of 4.0 g/dL


absolute lymphocyte count of 1,750/mm3
TcpO2 of 15mmHg
Ankle-brachial index of 0.7
Toe pressures of 40mm Hg

Buckwalter J, et.al. Orthopaedic Basic Science; 2000, p2-17.

71. A patient has numerous light brown skin macules, with freckling in the axillae and groin.
Tibial radiographs are shown in Fig 16a and b. He has a defect in the gene coding for
1. Merlin
2. Osteoclast activating factor
3. Prostaglandin E2
4. Neurofibrillin
5. Dystrophin

------------------------------------------------------------------Lovell & Winters Peds Ortho. Neurofibromatosis


Clinical findings (1) Caf au lait spots Tan, macular, and melanotic in origin and is
located in and around the basal layer of the epidermis. Lesions may vary in shape, size, number,
and location. They are frequently found in areas not exposed to the sun. The presence of caf au
lait spots may UNRELATED to neurofibromatosis. (2) Nodules are found only in postpubescnet
children. Dermal neurofibromas . . . these soft tumors may grow under, be flush with, or raised
above the level of the skin. (3) Nevus. Hyperpigmentation in up to 6% . . . dark-brown
pigmented areas of skin. (4) Plexiform neurofibroma. Bag of worms feeling, ropey .. tender. . .
lesions of skin, (5) Verrucous hyperplasia, (6) Axillary and Inguinal freckling. Other findings:
(1) Kyphoscoliosis, (2) scoliosis, (3) Paraplegia, (4) Spondylolisthesis, (5)Disorders of bone
growth, (6) CONGENITAL BOWING and pseudoarthrosis of the tibia. . . The single bone most
commonly affected by neurofibromatosis is the tibia. Bowing of the tibia is most
characteristically anterolateral and usually evident within the first 2 years of life.
Koval, OKU 7, pp 209-218
JAAOS 1999; 7: 217-230 Neurofibromatosis in childrem: Role of orthopaedist.
Diagnosis when 2 of 7 criteria met. (1) More than 6 caf-au-lait spots at least 15 mm in
adults or 5 mm in children, (2) two or more neurofibromas of any type or one plexiform
neurofibroma, (3) freckling in axillae or inguinal regions (Crowe sign), (4) Optic glioma, (5)
Two or more Lisch nodules (iris hamartomas), (6) A distinctive bone lesion such as sphenoid
dysplasia, thinning of cortex of a long bone, with or without pseudoarthrosis, (7)1st degree
relative with NF-1 by above criteria.
ANSWER

:4

72. What is the most common site of fracture because of osteolysis following cementless total
hip arthroplasty with an extensively porous-coated femoral component?
1- Central acetabulum
2- Ischial ramus
3- Greater trochanter
4- Between the lesser trochanter and the stem tip
5- Femoral stem tip

One listed reference states that five percent of femur fractures involve just the greater trochanter
and these can usually treated nonoperatively. It doesnt specifically state that greater trochanter
fractures are most common. However, given that proximal osteolysis and proximal stress
shielding are more common with an extensively porous-coated femoral component and that
distal fixation by ingrowth is generally stable, greater trochanter fractures is the best answer.
ANSWER

Pritchett IW: Fracture of the greater trochanter after hip replacement. Clin Orthop 2001 ;
390:221-226.
Heekin RD, Engh CA, Herzwurm PI: Fractures through cystic lesions of the greater trochanter: A
cause of late pain after cementless total hip arthroplasty. J Arthroplasty 1996; 11:757- 760.
Pellici PM, Tria AI, Garvin KL (eds): Orthopaedic Knowledge Update: Hip and Knee
Reconstruction 2. Rosemont, lL, American Academy of Orthopaedic Surgeons, 2000, pp 175180.

73.

What is the most common infection occurring in a toddlers and preschoolers hand?
12345-

paronychia
felon (pulp infection)
thenar space abscess
herpetic whitlow
septic flexor tenosynovitis

The first sentence in the OKU 7 Pediatric hand section on infections states, The most common
infection in the toddler and preschoolers hand is herpetic whitlow, a viral infection caused by
herpes simplex virus. It is characterized by painful vesicles on the distal phalanx of a single
digit without systemic involvement. It is self-limited and surgical treatment is not indicated.
Children are also susceptible to bite wounds, which should be treated the same as for adults with
debridement and antibiotics.
Obviously abscesses such as a thenar space abscess and septic flexor tenosysnotivitis would be
less common than either a paronychia or felon, so they can be ruled out. The incidence of felon
and paronychia is fairly similar.
ANSWER:

herpetic whitlow

Koval KJ(ed): Orthopaedic Knowledge Update 7. 2002, pp. 329-337.

74. A 35 year old man who sustained a comminuted radial head fracture in a fall undergoes
surgery the night of the injury. Postoperative radiographs are shown in Figures 17a and 17b. The
anticipated success of this treatment would most likely be compromised if the
1 patient is employed as a computer operator
2 patient receives no prophylaxis for heterotopic ossification
3 patient has tenderness at the distal radioulnar joint
4 annular ligament is disrupted
5 elbow is splinted for 4 days before beginning range of motion exercises
Comminuted Radial Head Fractures
More commonly associated with higher energy trauma, comminuted fracture patterns are
rarely amenable to stable internal fixation. Soft tissue swelling and ecchymosis suggest the
likelihood of capsular, brachialis, or ligamentous injury, including a dislocation that has
undergone spontaneous reduction. Capitellar fractures, Monteggia fracture-dislocations, and
wrist and forearm injuries may also accompany the comminuted radial head fracture. In view of
the possibility of significant soft tissue trauma, the timing of surgery in this setting may be of
more significance. Although this is uncommon, some patients have had satisfactory outcomes
after early postinjury mobilization and delayed (beyond 6 weeks) radial head excision.
Although radial head resection represents the most commonly performed procedure
for comminuted fractures, it has been observed to be associated with instability and late
posttraumatic arthritis when performed in the setting of medial collateral ligament or
interosseous membrane disruption. Excision alone is contraindicated when either of these two
soft tissue injuries is diagnosed. Either replacement or reconstruction is indicated in these cases.
Treatment Recommendation
In the setting of a low-energy injury, an elderly patient, or a lack of physical findings
suggestive of associated soft tissue disruption, resection of the radial head alone is a
straightforward approach that in most instances is not associated with long-term sequelae. If,
however, medial collateral ligament instability or disruption of the DRUJ is observed (the
Essex-Lopresti lesion), treatment should consist of either internal fixation of the radial
head or silicone replacement. After radial head reconstruction or replacement, the elbow
should be tested for valgus stability, and manual testing of the stability of the DRUJ should be
carried out. Should valgus instability be observed, exploration and repair of the medial collateral
ligament complex are recommended. Suture anchors placed in the medial column of the distal
humerus can be used to reattach the avulsed medial collateral ligament. Similarly, if distal ulnar
instability is present, it is best to pin the ulna to the radius with the forearm in supination. Elbow
flexion and extension can then be started, with the K-wire left in place for 4 to 6 weeks.
ANSWER

Skeletal Trauma 116-1134


OKU 2 trauma 39-51

75. A 92-year-old man who lives alone and is a community ambulator sustains a displaced
femoral neck fracture. Following a detailed informed consent discussion that
recommends surgery, he refuses treatment. What is the next most appropriate step?
11- Go ahead with surgery after obtaining administrative consent.
12- Obtain a second orthopaedic opinion.
13- Obtain consent from his next of kin.
14- Assess the patients global mental capacity, including orientation to person, time
and place.
15- Evaluate the patients specific decisional capacity by gauging his understanding of
his condition, treatment alternatives, and consequences.

Patients who have decisional capacity may refuse any treatment, even life-preserving treament.
Physicians are obliged by law and by ethics to respect the treatment refusals of competent
patients even if the physician disagrees with those choices or does not share the patients values.
Even when the consequences will be dire, a patients choices must be respected..
Society, however recognizes that from time to time people who are in the grip of fear,
depression, psychosis, or who are overwhelmed by difficult circumstances may express
preferences that do not reflect the values, commitments, or goals they usually endorse. Society
also recognizes that patients may sometimes refuse urgent or important medical treatment
because their decisional capacity is seriously impaired. Society has, therefore, specifically
entrusted physicians with the power to assess the decisional capacity of a patient and, in certain
circumstances, to take measures to override patient refusal
The decision to respect of override a patients refusal depends on whether the refusal
reflects and autonomous choice. It involves: 1) the ability to adopt values; principals and goals;
2) the ability to understand the situation; 3) the ability to make choices that reflect ones
commitments; and 4) the ability to act on them..The physician must assess the patients ability
to comprehend the facts, to invoke reasons that express values, to logically tie values to facts, to
reach a conclusion, and to adhere to that conclusionA patient who refuses to provide reasons
for refusing treatment that would save life or limb, should be told that, if he or she does not
explain the reasons, the refusal would be taken as a demonstration of incapacity, and others with
make the decision on the patients behalf. (Capozzi and Rhodes, JBJS 84A, 4: 691-3, 2002).
ANSWER

76. A 37 y/o woman w/ thoracic back pain has hemoptysis and dyspnea for the past month. A thoracic spine
radiograph shows a compression fracture of T9 and a mass in the right lung. What is the best course of
action?
1.
2.
3.
4.
5.

Metastatic work up
Physical therapy
Transpedicular biopsy of T9
Thoracotomy with partial resection of the mass
T9 corpectomy with anterior column reconstruction

This question is fairly straight forward. A middle aged patient with a history of a lung mass, hemoptysis
and dyspnea has an acute thoracic compression fracture. Without saying that she is a smoker, they are
inferring that this patient likely has lung cancer that is likely metastatic. Metastasis is the most commonly
encountered tumor in the spine. It would be unusual for a young patient to have an osteoporotic compression
fracture.
The question is asking the treatment for this problem. Physical therapy (2) would obviously be
inappropriate given the history and radiographic finding. The treatment of the metastatic lesion does not
prolong the pts survival. The goals of treatment are to control pain, restore neural functioning, correct
instability and to prevent pathologic fractures. The patient has no neurologic changes or instability that would
need an aggressive corpectomy (5). While a biopsy (3) may be needed in some cases, we have a pretty good
idea of the source of the lesion. If the lesion was solitary, advanced imaging followed by a biopsy would
likely be the choice. Resection of the primary mass (4) would not be the answer since metastatic disease is
already likely present. The appropriate answer should be to suspect that this is in fact a metastatic lesion on
work it up (1). The work up would include a bone scan, CT of the chest and abdomen. An MRI of the spine
lesion may be necessary to be sure that the cord has minimal compression.
ANSWER

1.

Metastatic work up

References:
Fardon OKU: Spine 2, 2002, pp411- 430.

77. What is the most frequently encountered complication following juvenile hallux valgus
correction?
1.
2.
3.
4.
5.

Recurrence of the deformity


Hallux varus
Transfer metatarsalgia
Nonunion of the first metatarsal osteotomy
Osteonecrosis fo the first metatarsal head

Recurrence is the most common complication. Juvenile hallus valgus is thought to occur more
frequently and have a higher chance of recurrence in children with pes planus, a long first
metatarsal, and metatarsus adductus (although Coughlin showed this not to be the case in his
study). Shoewear is considered less of a cause. Recurrence is seen in 10% of cases.
Coughlin MJ: Juvenile hallux valgus: etiology and treatment. Foot and Ankle Int 1995;16: 682697
ANSWER

78. The primary mechanism by which nonsteroidal anti-inflammatory drugs exert their
effectiveness is inhibition of
12345-

lipoxygenase.
leukotrienes.
phospholipase.
cyclooxygenase.
thromboxane A2.

References:
Buckwalter JA, Einhorn TA, Simon SR (eds): Orthopaedic Basic Science: Biology and
Biomechanics of the Musculoskeletal System, ed. 2. Rosemont, IL, American Academy of
Orthopaedic Surgeons, 2000, pp. 217-237.
Explanation:
Recall the cyclooxygenase and lipooxygenase pathways from page 229 of Orthopaedic
Basic Science. Steroids inhibit phospholipase. NSAIDs inhibit cyclooxygenase. Thus the
preferred response is 4.
ANSWER

79. A 22- year old man who sustained a gunshot wound to the hip 5 months ago is unable to
ambulate without crutches and reports a recent increase in hip pain with activity.
Examination reveals no open wound, and the neurologic and vascular status of the
extremity is intact. Radiographs are shown in Figures 18a and 18b. Management should
consist of:
1. iliac crest bone grafting
2. hemiarthroplasty
3. revision of cannulated screw fixation
4. valgus intertrochanteric osteotomy
5. quadratus femoris muscle pedicle bone graft

The treatment of symptomatic nonunion of intracapsular fractures is surgery. However, there


are several options and a decision regarding the type of surgical procedure must be tailored to the
individual patient.
Replacement of cancellous screws with a larger screw may be considered in the absence of
malalignment and shortening. It is essential that the sphericity of the femoral head and the
articular cartilage space have been maintained. This technique involves removal of the existing
implant and refixation with a larger diameter screw such as the dynamic hip screw. This should
be augmented with a supplementary cancellous screw placed superiorly to prevent rotation.
Fixation combined with proximal femoral osteotomy is indicated in the treatment of nonunion
with shortening or varus angulation of the proximal femur. Typically an angulation type
osteotomy has been reported to be done at the intertrochanteric level and stabilized using a 120
degree angled blade plate. The rationale of this procedure is to convert shearing forces at the
fracture site into compressive forces.
Autogenous grafting of the nonunion can be done. This can be as a pedicle type graft or as a
free vascularized or nonvascularized graft of the fibula or the iliac crest. A pedicle graft may be
done using the iliopsoas or the quadratus femoris muscle. The muscles are transposed to the
posterior aspect of the femoral neck and are secured using a screw placed into the
intertrochanteric region. This may be supplemented by cancellous autograft from the posterior
iliac crest. This technique has the advantage of revascularization and may be used in patients in
whom nonunion coesixts with avascular necrosis. It is also useful in the treatment of the
neglected fracture.
Although hip arthroplasty remains a suitable option for the treatment of symptomatic nonunion,
it has been shown to have higher risk of failure than primary hip arthroplasty for osteoarthritis.
Therefore, it is advisable to do procedures to preserve the femoral head in younger patients such
as this.
An algorithm is included which depends upon presence of avascular necrosis? Preservation of
anatomy? And age.
ANSWER

80. A child who has a malunion of a supracondylar humeral fracture now has a typical cubitus
varus (gunstock) deformity. What is the most common indication for early surgical correction of
this deformity?
1.
2.
3.
4.
5.

Poor cosmesis
Tardy ulnar nerve palsy
Posterior shoulder instability
Rotatory subluxation of the radiocapitellar joint
Anterior subluxation of the medial triceps

Poor cosmesis is the most common indication for early surgical correction of cubitus
varus deformity after a supracondylar fracture in children (Gurkan et al., Journal of Ped.
Orthopaedics, 2002; 22:198-202. This is probably the case because the normal carrying angle
increases from childhood to adulthood and an increase in valgus is not as cosmetically noticeable
as a complete reversal to a varus position.
Cubitus varus deformity is the most common deformity arising after a supracondylar
fracture. Several causes have been suggested and include:
1. Medial displacement and rotation of the distal fragment is most often blamed. Smith
showed in his experimental studies that varus tilting of the distal fragment was the
most important cause of change in carrying angle.
2. LaBelle et al. found varus tilting of the distal fragment to be the cause of deformity in
all their patients with cubitus varus deformity.
3. Kasser noted that osteonecrosis and delayed growth of the trochlea, with relative
overgrowth of the normal lateral side of the distal humeral epiphysis is also a rare
cause of progressive cubitus varus deformity.
Rotational malalignment may occur, but is not a significant cause of deformity. Malrotation of
the distal humerus is compensated for, to a large degree, by the shoulder joint and so any
rotational component of the cubitus varus is of little consequence.
In treating cubitus varus deformity surgically, there are several options. These include:
1. Medial opening wedge osteotomy with bone graft
2. oblique osteotomy with derotation
3. lateral closing wedge osteotomy
The vast majority of the time, a lateral closing wedge osteotomy is the easiest, safest, and most
stable method to correct the deformity. This technique is described by Voss et al: J. Pediatr
Orthop 14:474, 1994).
ANSWER

Campbells Operative Orthopaedics. pp. 1446-1451

81. Nonsurgical treatment of cubital tunnel syndrome includes use of a night splint that should
be designed to maintain the elbow at
1) 45 degrees and forearm in pronation
2) 45 degrees and forearm in supination
3) 45 degrees and the forearm in neutral
4) 90 degrees and the forearm in neutral
5) 90 degrees and the forearm in pronation

This position creates the least pressure on the ulnar nerve.


ANSWER

82: Figures 19a and 19b show the radiographs of a 14-year-old boy who fractured his talus and
ankle 12 weeks ago. What conclusion can be drawn based on the radiographic findings?
1.) Osteomyelitis is present
2.) Complex regional pain syndrome is present
3.) Malunion is present
4.) Union is absent
5.) Osteonecrosis is absent

Osteonecrosis of the ankle and foot typically occurs in the talus as a consequence of talar neck
fractures with vascular compromise of the bone at the level of the sinus tarsi. Osteonecrosis of
the tarsal navicular bone can occur in children (Kohler disease). A form of osteonecrosis of the
tarsal navicular bone has also been described in adults (Mueller-Weiss syndrome).
Osteonecrosis of the ankle and foot region is also frequently seen in the second metatarsal
head (Freiberg disease), with sclerosis and flattening of the metatarsal head seen at conventional
radiography, and in the first metatarsal sesamoid bone. MR imaging is valuable in assessing the
presence, size, and fragment viability of posttraumatic osteonecrosis. Areas of inhomogeneous
signal intensity surrounded by a hypointense band, sometimes with a second band of high signal
intensity on T2WI (double line sign), are characteristic findings in osteonecrosis before
subchondral fracture and collapse occur. These findings can also be seen in posttraumatic
osteonecrosis of the talus.
Osteonecrosis involving the head of the second metatarsal, tarsal navicular, or sesamoid bone
exhibits MR imaging features somewhat different from those described in osteonecrosis of the
talus. Low signal intensity without an obvious demarcating interface is seen on both T1WI and
T2WI.

ANSWER

83 What is the best predictor for a low back injury in a collegiate athlete?
1 Type of sport
2 Gender
3 History of back injury
4 Level of athletic experience
5 Satisfaction with coaching staff

The correct answer is 3. According to the article, History of Low back injury is a risk factor for
recurrent injuries in varsity athletes, by Greene et. Al., history of back injury is a risk factor for
recurrent injury. This study prospectively looked at 679 college varsity athletes were surveyed to
determine a history of low back injury. The athletes were then followed and 6.8% suffered a
subsequent back injury in the following year. There were no differences between female and
male athletes with regard to percentages, nor were there differences between contact and non
contact sport participants. Satisfaction with the coaching staff was also reviewed and proved to
be a non factor with regard to predictors of Low back injury. The study does not specifically
address the level of athletic experience; however, the relative risk of injury in the players with
previous injury was > 6. The authors hypothesize that this predisposition is due to the athletes
desire to return to full competition before full rehabilitation can be achieved.
ANSWER

84. Which of the following substances has been implicated in the pathogenesis of
osteoarthritis?
1. Interleukin 1
2. Tissue inhibitor of matrix metalloproteinase
3. Cox-1
4. BMP-2
5. Antigenic Specific T cells

Interleukin 1 is a mediator of cartilage degradation particularly in inflammatory events. This


material may originate in monocytes of inflamed joints. It is also synthesized by
chondrocytes as a paracrine activity. IL-1 enhances enzyme synthesis and activation of a
number of enzymes systems in cartilage including latent colllagenase, stromelysisn, and
gelatinase,and TPA. Plasminogen is presumed to be synthesized by the chondrocytes or
passes across the synovial membrane to enter the matrix. The stimulation of collagenase and
stromelysin production by IL-1 is thought to be important in elevating catabolic activity in
cartilage.
ANSWER

85. Figures 20a and 20b show the radiographs of a 14 year-old boy that had Legg-Calve Perthes
disease when he was 8 years old. He now reports hip pain, popping, and locking. Treatment
should consist of
1- incisional biopsy.
2- hip fusion.
3- removal of the loose body.
4- total joint arthroplasty.
5- core decompression.

Twenty to forty years after the onset of symptoms from LCPD the majorities (70-90%) of
patients are active and pain free. Most maintain a good range of motion despite the fact that few
have normal radiographs. Increasing pain, decreased ROM and loss of function are observed in
only those patients with flattened irregular femoral heads at the time of primary healing and in
those patients premature physeal closure.
Although the incidence of hip pain in adolescent children whom had LCPD as a child is small,
the vascular insult to the femoral head predisposes the adolescent to certain pathology.
Osteochondritis dissecans may present during adolescence with or without loose body fragments.
The incidence has been reported around 2-4%. Usually the femoral head appears to heal and the
child has a symptom free period before presenting later with intermittent pain and stiffness.
Other causes of late hip pain after LCPD are impingement of the flattened femoral head on the
superior acetabular rim and acetabular labrum tear.
The AP and frog lateral radiographs of the hip clearly demonstrate a large loose body fragment.
The remainder of the femoral head looks intact without flattening. Due to the patients young
age, simple removal of the loose body would be a good option. Wood et al reported on 17 cases
of osteochondritis dissecans of the femoral capital epiphysis of which six were treated with
surgical excision of a loose fragment. All returned to fairly normal living and gradual filling of
the surgical defect with bone. They reported 85% resolution of catching and 50% reduction in
pain after removal of the loose body.
ANSWER

Reference:
Grossbard GD: Hip pain in adolescence after Perthes disease. JBJS Br. 1981;63;572-574
Wood JB, Klassen RA, Peterson HA: Osteochondritis dissecans of the femoral head in children
and adolescents: A report of 17 cases. JPO 1995; 15:313-316

86. An otherwise healthy 64 year old man undergoes total shoulder arthroplasty for OA. Because
of a long standing internal rotation contracture, a subscapularis lengthening procedure is
performed to rebalance the joint. What important concept needs to be kept in mind regarding
postoperative physical therapy?
12345-

Intiation of PT must be delayed by several weeks


Elevation should be performed in a sitting position
External rotation isometrics should be avoided
External rotation stretching should be limited
Internal rotation should be limited to eccentric techniques.

In contrast to contracture in primary glenohumeral osteoarthritis, patients with capsulorrhaphy


arthropathy usually have a thick subscapularis tendon and anterior capsule from the previous
instability procedure. According to the following articles, although subscapularis lengthening
significantly increases shoulder external rotation, it often results in internal rotation weakness. It
is difficult to compare the preoperative with the postoperative internal rotation strength because
the true strength of the subscapularis cannot be determined when there is an internal rotation
contracture. Nonetheless, in these articles the authors believe that improved external rotation is
critical for shoulder elevation and function and is more important than subscapularis strength.
Thus, subscapularis lengthening was performed to improve external rotation and aftert a
subscapularis lengthening procedure it is important not to stress the lengthened unit hence the
limiting of external rotation stretching,
Although the question does not indicate that the patient had an anterior stabilization procedure
that resulted in glenohumeral osteoarthritis, both references address this issue. Both references
indicate that osteoarthritis with an internal rotation contracture is a severe complication of
anterior stabilization procedures like the Putti-Platt, Magnuson-Stack, and Bristow. For patients
undergoing total shoulder arthroplasty with a severe internal rotation contracture, a subscapularis
lengthening is done to restore the normal biomechanics of the shoulder and functional outcome.
In one of the referenced studies, patients with less than 20 degrees of external rotation
preoperatively were treated with a lengthening procedure. The lengthening procedure increased
external rotation to a mean of 46 degrees compared to 27 degrees without it. Post-operative
rehab should not include much external rotation stretching as this could lead to rupture of the
repair.
ANSWER

Bigliani LU, et al: Glenohumeral arthroplasty for arthritis after instability surgery. J Shoulder
Elbow Surg 1995; 4 87-94
Green A, et al: Shoulder arthroplasty for advanced glenohumeral arthritis after anterior instability
repair. J Shoulder Elbow Surg 2001; 10: 539-545

87. A 27-yo patient sustains the closed femoral fracture shown in figure 21a and 21b. This
fracture pattern is most likely the result of which of the following forces?
12345-

Pure Torsion
Pure Bending
Pure Compression
Four-Point Bending
Torsion plus bending

Page 376 of the orthopaedic basic science book has a very nice figure demonstrating each
AO fracture class and the force required for development of that fracture pattern.
Type A: Simple
1- Spiral: Torsion
2- Oblique: Uneven Bending
3- Transverse: Pure Bending
Type B: Butterfly
1- Butterfly by torsion: Torsion+ bending
2- Butterfly by bending, one: Bending + compression, low speed
3- Butterfly by bending, several: Bending + Compression, high speed
Type C: Comminuted
1- Comminuted by torsion: High speed torsion
2- Segmental: 4 point bending
3- Crush: Crush

ANSWER

Reference:
Orthopaedic Basic Science, 2nd edition page 376.

88. Highly cross-linked UHMW polyethylene has what effect on tensile and fatigue strength
when compared with UHMW polyethylene?
1. increased tensile and fatigue strength
2. increased tensile strength and decreased fatigue strength
3. decreased tensile and fatigue strength
4. decreased tensile strength and no change in fatigue strength
5. no change in tensile or fatigue strength

Crosslinking of UHMWPE has gained popularity recently in total joint replacements as a method
to improve wear resistance. Crosslinking has been proven to drastically improve the wear
resistance of the polymer. Recent reports suggest however that crosslinking may have the
disadvantage of decreased fatigue strength. Fatigue crack propagation resistance is a concern
with tibial components, where large cyclic stresses may lead to subsurface cracks and ultimately
to delamination wear.
Baker et al evaluated the effect of crosslinking on strength and fatigue crack propagation
resistance. Scanning EM was employed to compare crosslinked with non-crossliked
polyethylene. Crosslinking of polyethylene was found to result in decreased tensile and fatigue
strength. Baker et al concluded that the increased wear resistance provided by crosslinking may
not be beneficial for fatigue propagation resistance.
References:
Koval KJ (ed): OKU 7. AAOS, 2002, 39-45.
Baker DA et al: Study of fatigue resistance of chemical and radiation crosslinked medical
guide UHMW polyethylene. J Biomed Mater Res 1999; 46:573-81.
ANSWER: 3

89.

A newborn is undergoing serial manipulation and casting for congenital idiopathic


clubfoot deformity. Which of the clubfoot deformities should be corrected last?
1.
2.
3.
4.
5.

Forefoot pronation
Forefoot adduction
Medial talonavicular subluxation
Hindfoot varus
Ankle equinus

In his classic article, Ponseti describes congenital club foot (talipes equinovarus) as
having four components: equines, varus, adductus, and cavus. The forefoot is in adduction and
supination. The hindfoot is in equinus and varus. The goal of treatment is to reduce or eliminate
these four deformities so that the patient has a functional, pain-free, plantigrade foot, with good
mobility and without calluses, and does not need to wear modified shoes. In the newborn or
young infant treatment should begin with stretching, manipulation, and serial casting. The main
components of this type of treatment are as follows:
1. The cavus deformity of the hind part of the foot is corrected by supination of the fore
part of the foot with respect to the hind part.
2. The varus deformity of the hind part of the foot is corrected with displacement or
lateral rotation of the navicular, together with the cuboid and the anterior aspect of the
calcaneus, without pronation of the foot.
3. Maintenance of the correction of the varus deformity of the hind part of the foot
entails marked external rotation of the foot distal to the talus. This is accomplished
by a toe-to-groin plaster cast with the knee flexed 90 degrees and the foot in
maximum external rotation.
4. The equinus is corrected last, by dorsiflexion of the foot with the heel in valgus
angulation.
ANSWER

Ponseti, I.V.: Treatment of congenital club foot. J Bone Joint Surg Am 1992;74:448-454.

90. The pelvic radiograph shown in Figure 22 reveals which of the following findings?
1. Spur sign
2. Femoral head subluxation
3. Femoral head fracture
4. Anterior wall acetabular fracture
5. Posterior wall acetabular fracture

The radiograph shown in Figure 22 is an obturator oblique view of the right hemipelvis (Judet
view) showing a posterior wall acetabular fracture. Judet views are helpful for visualization of
acetabular fractures. The iliac oblique allows for assessment of the posterior column and anterior
wall (IPCAW) while the obturator oblique allows for assessment of the anterior column and
posterior wall (OPWAC).
ANSWER

Letrounel E, Judet W, Elson RA: Fractures of the acetabulum, ed 2. Berlin, Germany, SpringerVerlag, 1993.
Koval KJ (ed): Orthopaedic Knowledge Update 7. Rosemont, IL, American Academy of
Orthopaedic Surgeons, 202, pp 407-16, pp 395-405.

91. The bony process shown in Figure 23 may be associated with compression of which of the
following nerves?
12345-

ANSWER

ulnar
radial
musculocutaneous
posterior interosseous
median

median

Ivins GK, Suprocondylar process syndrome; A case report. J Hand Surg Am 1996; 21; 279281.

92. Figure 24 shows the radiogrpah of a 47-year old pateint with typeI diabetes mellitus who has
insidious onset ankle pain. What is the most likely diagnosis?
12345-

ANSWER

syndesmotic disrution
osteosarcoma
maisonneuve fracture
neuropathic fibular fracture
osteomyelitis of the fibula

Brodsky JW, The diabetic foot, in Coughlin & Mann Surgery of the foot and ankle, 1999; p.
940-945.

93. Figures 25a through 25e show the plain radiograph, MRI, bone scan, and biopsy speciment
of a 14 yo girl who has increasingly worse left hip pain. What is the most likely diagnosis?
1.
2.
3.
4.
5.

Nonossifying fibroma
desmoplastic fibroma
osteofibrous dysplasia
fibrous dysplasia
fibrosarcoma

From Millers:
Fibrous dysplasia is a benign, developmental abnormality of bone. It can be monostotic or
polyostotic. There may be associated areas of yellow or brown pigmentation of the skin.
McCune-Albright syndrome is polyostotic involvement, with skin changes, and endocrine
abnormalities (precocious puberty). The most common site is the proximal femur, where it can
lead to a shepherds crook deformity. The typical age range is 10-25 yr. Xray appearance
varies from highly lytic to ground glass appearance; There is usually a rim of sclerotic bone
circumscribing. Histology shows proliferative fibroblasts with a dense, collagenous ECM;
along with irregular areas of trabecular bone and osteoid giving it an alphabet soup or
Chinese letters appearance. Treatment is symptomatic.
ANSWER

94. The strongest bone pin interface will be achieved with external fixation pins that are
coated with
1)
2)
3)
4)
5)

Calcium Sulfate
Titanium
Hydroxyapatitie
Polyglucoloic acid
Silver

Ref Sanden, Olerud, Petrin-Mallmin, Larsson. Hydroxyappetite improves fixation of pedicle


screws. JBJS B 2002;94:387-391
ANSWER

Prospective study done in Sweden where they randomly used stainless steel, partial HA coated,
and full HA coated pedicle screws in lumbar fusion for 1-2 level disease. Then at an average of
12 months later, they went back in and took these screws out (indication not mentioned in
article). When they took them out , they measured the torque required to remove them. Lo and
behold, the fully HA coated screws required much more extraction torque (600 Ncm, for you
weenies who want to know). Titanium screws are tough get out sometime, more so than
Stainless Steel, but there is no reference to titanium. As for these other things, there is no
reference for them.

95 A 27-year-old chef sustains a traumatic injury to the index finger in an electric mixing
machine. Examination reveals an isolated 2 cm x 2 cm loss of palmar skin over the proximal
phalanx with exposure of the flexor tendons. Coverage of this defect is best accomplished with
1- An axial flag flap from the long finger
2- Split-thickness skin grafting
3- Full-thickness skin grafting
4- A Moberg advancement flap
5- A thenar flap

The axial flag is the correct choice because the skin can be moved from the dorsum of the long
finger to the palmar aspect of the index finger to cover the flexor tendons. The skin can be
moved to the palmar surface of the same digit or to the neighboring digit. The flagstaff
contains the pedicle consisting of dorsal vein, dorsal branch of digital artery, and dorsal branch
of digital nerve. See figure below. Split thickness skin graft cannot be done over exposed
tendons. Full thickness skin graft would work, but does not provide sensation. The Moberg
advancement is most indicated for amputations distal to thumb IP joint, in which a V-Y flap
cannot be used. It leads to excessive stiffness in the fingers. A V-Y flap is used locally to cover
finger tip skin loss from amputations. The thenar flap can be used for coverage of finger tip
injuries the index, long, ring, (and in some cases the little finger. In this case, since it is not a
fingertip injury, but a more proximal injury, the thenar flap is unnecessary.

ANSWER

Iselin F: The flag flap. Plast Reconstr Surg 1973;52:374-377. Manske PR (ed): Hand Surgery
Update. Englewood, CO, American Society for Surgery of the Hand, 1994, pp 27-34.

96. The equinovarus foot position in pateints with Duchenne muscular dystrophy is caused by
12345-

ANSWER

Achilles tendon shortening and persistant function of the long toe flexors
contracture of the gastrocnemius-soleus complex and spasticity of the long toe
flexors
contracture of the gastrocnemius-soleus complex and spasticity of the anterior
tibialis
contracture of the gastrocnemius-soleus complex and persistant function of the
posterior tibialis
spasticity of the gastrocnemius soleus complex and the posterior tibialis muscles

Scher DM, Surgical Prevention of foot deformity in pateints with Duchenne muscular dystrophy.
JPO 2002; 22; 384-391.

97. A 17-yo soccer player reports lateral and posterior pain after jamming her knee in a game.
Examination reveals rage of motion of 0 to 100 degrees and a 1+ knee effusion. The Lachmans
test is positive. External rotation of the tibia is increased with the knee in 30 deg of flexion but
not at 90 deg of flexion. There is increased laxity to varus stress with the knee at full extension.
What is the most likely diagnosis?
1. Isolated tear of the ACL
2. Isolated tear of the PCL
3. Tears of the ACL and PCL
4. Tears of the ACL and posterolateral complex
5. Tears of the PCL and posterolateral complex
The characteristic history of an anterior cruciate ligament (ACL) injury involves a rapid
deceleration or rotational injury to the knee. An audible"pop" is noted in 30% to 50% of patients. Knee
ligament injuries occur as a result of direct contact (30%) and noncontact (70%). Direct contact injuries
typically involve at least one cruciate and one collateral ligament. Immediate on-field examination
demonstrates a positive Lachman test and a hemarthrosis occurs within hours of the injury. The pivot shift
test is difficult to assess in the awake patient once joint swelling (effusion) and muscle spasm have
developed. Joint line tenderness may suggest meniscal or capsular injury but is not specific in the first 10
to 14 days after the
injury. Collateral ligament injury is detected by varus or valgus laxity with the knee in 25 of flexion.
Significant collateral ligament laxity in full extension is indicative of a more severe injury involving
multiple ligaments and potentially both cruciates.
Several recent basic science studies have provided new insight into the importance of restoring
posterolateral complex structures after injury. The posterolateral structures, which include the popliteus,
popliteofibular ligament, biceps tendon, iliotibial tract, and lateral collateral ligament, play an important
role in knee stability. The popliteus has been shown to be an important secondary stabilizer against
posterior tibial translation, especially when the PCL is absent. Other studies also have demonstrated the
synergistic effect of the PCL and posterolateral corner structures. One biomechanical study demonstrated
increased posterior tibial translation, external rotation, and PCL graft stresses associated with deficiency
of posterolateral structures.
The mechanism of injury for posterolateral complex injuries typically involves hyperextension
with a varus moment and a twisting force. A posterolaterallydirected blow to the medial tibia with the
knee in extension (resulting in hyperextension and external rotation) is the most common mechanism of
injury. Combined injuries (usually
PCL, but also ACL) are common. Isolated posterolateral complex injuries may result in posterolateral
pain, peroneal nerve paresthesias (up to 30%), and instability with knee extension. Several physical
examination tests, including the posterolateral drawer test, external rotation recurvatum test, reverse pivot
shift test, and the external
rotation test, depend on comparison with the contralateral (normal) knee. The most reliable test is for
asymmetric tibial external rotation (Fig. 4). A difference of more than 10 compared with the contralateral
limb is evidence of a pathologic condition. The test is performed at 30 and 90 of flexion. With an
isolated posterolateral injury, the difference is noted only at 30. With a combined PCL and posterolateral
injury, the difference is noted at 30 and 90 of knee flexion.

ANSWER

98. A 46-year-old man undergoes a bone grafting procedure from the anterior iliac crest. Two
weeks after surgery, he reports decreased sensation about the anterior thigh. The most likely
diagnosis is a neurapraxia of what nerve?
1- Ilioinguinal
2- Femoral
3- Cluneal
4- Saphenous
5- Lateral femoral cutaneous

The lateral femoral cutaneous nerve provides sensory innervation to the anterolateral thigh.
Anterior cutaneous of the femoral nerve provides sensory innervation to the anteromedial thigh.
Cluneal nerves are at risk with posterior iliac crest bone graft. Saphenous nerve provides sensory
innervation to the medial lower leg. The femoral nerve should be safe if the incision is lateral to
the ASIS. However, the lateral femoral cutaneous nerve is variable in its course. It typically
passes under the inguinal ligament, but it may cross over the iliac crest. Murata et al. estimated
that ~10 % of lateral femoral cutaneous nerves may be at risk.
ANSWER

Ahlmann E, Patzakis M, Roidis N, Shepherd L, Holtom P: Comparison of anterior and posterior


iliac crest bone grafts in terms of harvest-site morbidity and functional outcomes. J Bone Joint
Surg Am 2002;84:716-720.
Murata Y, Takahashi K, Yamagata M, Shimada Y, Moriya H: The anatomy of the lateral femoral
cutaneous nerve, with special reference to the harvesting of iliac bone graft. J Bone Joint Surg
Am 2000;82:764-767.

99.

What outcomes instrument is the most specific for disorders involving the lumbar spine?
12345-

SF-36
Oswestry disability index
Minnesota multiphasic personality index
Visual analog scale
Patient self-assessment of satisfaction

Expect more questions like this on future exams as evidence based medicine becomes more
prominent. The SF-36 is a measure of general health, and is not spine specific. The MMPI,
although often utilized in spine studies to evaluate a patients mental state, is not spine specific
but rather psychologically specific. The visual analog pain scale is not spine specific. The
patient self assessment is notoriously unreliable. The Oswestry disability index may be the
most useful in specialty care settingsin which the disability level is likely to remain relatively
high throughout a trial (i.e. chronic severe low back pain). It uses a long questionnaire to
specifically assess low back pain and disability. It is a validated, but time consuming index.

ANSWER

Oswestry disability index

Deyo RA, et.al.; Outcome measures for low back pain research: A proposal for standardized
use. Spine 1998; 23: 2003-2013.
Fanuele JC, et.al.; The impact of spinal problems on the health status of patients: Have we
underestimated the effect? Spine 2000; 25: 1509-1514.
Fardon DF, et.al.; OKU: Spine 2. 2002, pp.139-144.

100. Figures 26a through 26e show the plain radiograph, MRI scans, bone scan, and biopsy
specimen of a 62 year old man who has progressive left shouldr pain. Immunohistochemistry
staining on the tissue is positive for leukoccyte common antigen and negative for CD 99.
Staging studies show no disease elsewhere. What is the best local treatment.
12345-

ANSWER

forequarter amputation
extraarticular wide excision
intraarticular wide excision
intralesional curretage
radiation therapy

radiation

Durr HR; Malignant lymphoma of the bone. Arch orthop trauam surg. 2002; 122; 10-16.

101. Which of the following conditions is characterized by congenital vertebral abnormalities,


anal and renal anomalies, tracheaoesophageal fistula, and partial or complete radial dysplasia?
16- VATER syndrome.
17- Goldenhars syndrome.
18- Polands syndrome.
19- Klippel-Feil syndrome.
20- Sprengels deformity

VATER association (Vertebral defects, imporforate anus, tracheoesophageal fistula, and radial
and renal dysplasia) is the term used to describe the nonrandom association of multiple
malformations of the vertebrae, lower GI tract, trachea and esophagus, renal tract, lungs, heart
and radius. Approximately 60% of patients with vertebral anomalies (all sites) have associated
abnormalities. The site does not correlate with occurence of the associated abnormality.
Goldenhars syndrome (vertebral malformations with malformations of the ear, lip, and palate),
Sprengels malformation (vertebral malformations with malformations of the scapula and
clavicle), and Polands syndrome (vertebral malformations with malformation of the pectoral
muscles and the hand) have no clear embryologic explanation and no clear pattern of inheritance.
Klippel-Feil syndrome includes congenital cervical vertebral anomalies and is associated with
poor cervical mobility and potential instability. It is classically associated with low posterior hair
line, short neck, and restricted motion.
ANSWER

102. In patients w/ osteoporosis, the incidence of fracture is directly related to


1.
2.
3.
4.
5.

age at the time of peak bone mass


age at the onset of menopause
serum calcium levels
the degree of bone mass loss
osteoclast and osteoblast activity

Osteoporosis is a systemic, age-related metabolic disorder affecting the axial and appendicular skeleton.
It results for decreased bone mass and an increased incidence of fractures. The bone mass that remains has a
normal calcium content and normal bone matrix. Primary osteoporosis is an age related loss of bone mass.
Secondary osteoporosis is loss of bone as a result of an associated endocrinopathy or disease state. Primary
osteoporosis is subclassified into type I post menopausal and II senile. Type I is related to estrogen
deficiency and affects primarily trabecular bone mass and is osteoclast mediated (vertebral and distal radius
fractures more common). Type II is osteoblast mediated, is caused by the affects of aging, calcium deficiency,
and increased parathyroid hormone and primarily affects cortical bone. Fractures of the hip, pelvis, and prox
humerus are more common.
Bone mass peaks in the mid thirties. Bone loss begins at a rate of 0.3% per year in men and 0.5%/ year
in women. With menopause, bone loss accelerates to 2-3%/year (for 6-10 years before returning to
premenopausal levels). Estrogen deficiency is directly related to postmenopausal osteoporosis.
Risk factors for osteoporosis include inadequate calcium and Vit D intake, inadequate WB exercise,
smoking, excessive ETOH use, Caucasian race, chronic steroid use, and early menopause. Secondary causes
include hyperparathyroidism, Cushing syndrome, hyperthyroidism, Vit D related disorders.
The incidence of fracture is related directly to degree of bone mass loss. Bone mineral density can be
measured precisely utilizing the dual energy x-ray absorptiometry (DXA). World Health Society has
classified bone density based on comparison w/ the peak bone mass of a young same gender normal
individual. Within one standard deviation is normal, 1.0-2.5 osteopenic, and greater than 2.5 has osteoporosis.
ANSWER

4.

the degree of bone mass loss

References:
Fardon OKU Spine 2 2002 pp123-33.
Bernstein: Metabolic bone disorders of the spine, Rothman: The spine 1992, pp1381-1403.

103. The force required to initiate sliding of the lag screw in cephalomedullary nails used for
stabilization of proximal femoral fractures is inversely related to the
1.
2.
3.
4.
5.

length of the nail


length of the lag screw medial to the nail
length of the lag screw lateral to the nail
length over which contact occurs between the lag screw and the device
diameter of the screw

Not the most useful tidbit of orthopaedic knowledge, but I would chance it to guess that the
writer of the question is also the author of the paper in this reference.
A cephalomedullary nail requires more force than a DHS type device because there is less
contact of the screw with the stem compared to a barrel in a plate. This means that there is more
force creating a bending moment at the screw nail interface in a cephalomedullary nail than at
the screw plate interface in a DHS, and there is a greater chance of cutout in the former. With a
long barrel, the screw will slide under axial loads at the hip joint whereas in the
cephalomedullary nail, more bending will occur at the nail screw interface instead of sliding.
The more rigid the device (because of less sliding), the more chance of cutout.
ANSWER

Loch DA, Kyle RF, Bechtold JE, Kane M, Anderson K, Sherman RE: Forces required to initiate
sliding in second generation intramedullary nails. JBJS 80(A): 1626-1631

104. In addition to phantom limb pain and residual limb discomfort, which of the following
areas is likely to be painful after a lower extremity amputation for trauma?
12345-

Ipsilateral hip.
Contralateral knee.
Lower back.
Upper extremity.
Neck.

References:
Smith DG, Ehde DM, Legro MW, Reiber GE, del Aguila M, Boone DA: Phantom limb,
residual limb, and back pain after lower extremity amputations. Clin Orthop 1999;361:29-38.
Koval KG (ed): Orthopaedic Knowledge Update 7. Rosemont, IL. American Academy
of Orthopaedic Surgeons, 2002, pp. 127-137.
Explanation:
In the above reference, Smith and his colleagues studied the sensations and pain reported
in 92 individuals with unilateral lower extremity amputations. Pain after limb amputation is a
common sequelae that often becomes chronic, limiting quality of life and functional capacity.
Phantom limb pain has been defined as painful sensations perceived in the missing body part,
whereas residual limb pain is pain perceived as emanating from the residual portion of the limb
(the stump).
Using a visual analog scale, participants reported the frequency, intensity, and
bothersomeness (what the?) of phantom limb, residual limb, and back pain and nonpainful
limb sensations. A survey of medication use for each category of sensations also was included.
Stastical analyses revealed that nonpainful phantom limb sensations were common and more
frequent than phantom limb pain. Residual limb pain and back pain were also common after
amputation. Back pain surprisingly was rated as more bothersome than phantom limb pain
or residual limb pain. Back pain was significantly more common in persons with above knee
amputations. Thus the preferred response is 3.
ANSWER

105. A 26-year-old woman has chronic toe pain after hitting a bedpost 3 months ago. A
radiograph is shown in Figure 27. Her injury represents an avulsion of the:
1. lumbrical insertion
2. extensor digitorum brevis
3. extensor digitorum longus
4. extensor hallucis longus
5. flexor digitorum longus

This radiograph shows an avulsion of the extensor digitorum longus . A common synovial
sheath surrounds the four tendons of the extensor digitorum longus as they diverge on the
dorusm of the foot and pass to their distal attachments. Each tendon forms a membranous
extensor expansion (dorsal aponeurosis) over the dorsum of the proximal phalanx of the toe,
which divides into two lateral slips and one central slip. The central slip inserts into the base of
the middle phalanx and the lateral slips converge to insert into the base of the distal phalanx.
The extensor hallucis longus has its distal attachment on the dorsal aspect of the base of the
distal phalanx of the great toe. The flexor digitorum longus inserts at the bases of the distal
phalanges of the lateral four toes on the plantar aspect. The lumbricals have their proximal
attachment in the tendons of flexor digitorum longus and their distal attachment at the medial
aspect of the expansion over the lateral four digits. The lumbricals serve to flex the proximal
phalanges and extend the middle and distal phalanges of the lateral four toes. The extensor
digitorum brevis extends digits 2 to 4 at the metatarsophalangeal joints.
ANSWER

106. When compared to a 95 degree screw-plate construct, an intramedullary nail used to treat a
reverse obliquity-type intertrochanteric femoral fracture most often results in
1. a shorter time to fracture union
2. a lower reoperation rate at 1 year
3. a higher perioperative mortality rate
4. a higher perioperative transfusion requirement
5. improved function at one year

The reverse obliquity fracture represents an uncommon, albeit significant proportion of


hip fractures. In their paper, Reverse Obliquity Fractures of the Intertrochanteric Region of the
Femur, JBJS, 83-A, 5, May 2001, Haidukewych et al. describe reverse obliquity fractures in 2%
of all hip fractures in their 2472 patients examined, and 5% of all intertrochanteric fractures. The
typical intertroch fracture involves a fracture line running obliquely from the greater trochanter
proximally to the lesser tranchanter distally. In the reverse obliquity fracture, the fracture line
runs distal-lateral and to proximal-medial.
In their series, the authors found 55 reverse obliquity fractures in 1035 intertroch
fractures. These fractures were treated in a variety ways. Treatment methods included sliding
hip screws, blade-plates, dynamic condylar screws, cepahlomedullary nails and intramedullary
hip screws. Their results showed that 68% of those patients treated with internal fixation healed
without an additional operation and 32% of the 47 hips treated with internal fixation failed to
heal or had failure of fixation at follow-up of 15 months.
Their findings showed that fixed angle devices out-performed the sliding hip screws. In
those patients treated with sliding hip screws, 9 of 16 failed. In those treated with blade plates, 2
of 15 failed. In those treated with dynamic condylar screws, 3 of ten failed. In those treated with
cephalomedullary nails 1 of 3 failed. And, finally, in those treated with intramedullary hip
screws, 0 of 3 failed. The authors contend that the sliding hip screws fail because of the
biomechanics of their design in which the direction of the compression must be perpendicular to
the fracture line (as in intertroch fractures). However, in the reverser obliquity fractures, the
design is prone to sliding of the proximal fragment and medialization of the distal fragment.
In another paper by Sadowski et al. JBJS, 84:372-381 (2002), the authors examine the
difference between 95 degree screw-plate constructs and IM nails in reverse obliquity or
transverse intertrochanteric fractures. The fractures treated in their series were classified as
AO/OTA 31-A3. Nineteen patients were treated with DCS constructs and twenty with proximal
femoral nails. Their results showed that patients treated with IM nails had shorter operative time,
fewer blood transfusions, and shorter hospital stays. Implant failure and/or nonunion was noted
in 7 of 19 hips treated with DCS, while only 1 in 20 treated with IM nail did not heal at a
minimum of one year follow-up.
ANSWER

107. What is the most common congenital hand anomaly?


1) Symbrachydactyly
2) Camptodactyly
3) Syndactyly
4) Polydactyly
5) Constriction ring syndrome

According to Miller, syndactyly is the most common congenital hand anamoly. Syndactyly
presenting alone in an individual is autosomal dominant with reduced penetrance and variable
expression yielding a positive family history in 10-40% of cases. Ray involvement is: Thumbindex 5%, Index-middle 15%, Middle-ring 50%, ring-pinky 30%.
ANSWER

108: A 36 year-old man sustained a traumatic brachial plexus injury to his dominant right upper
extremity 3 years ago. Examination reveals no active deltoid or biceps function, and he is unable
to bring his hand to his face. He has 1/5 elbow flexion strength, 4/5 pectoralis major strength,
and 4/5 triceps strength. His trapezius and rhomboids are intact. Distally he has good hand
function. What surgical procedures will best restore his ability to bring his hand to his face?
1.) Glenohumeral arthrodesis and pectoralis major transfer
2.) Glenohumeral arthrodesis and triceps transfer
3.) Glenohumeral arthrodesis and Steindler flexorplasty (proximal flexor pronator
transfer)
4.) Scapulothoracic arthrodesis and pectoralis major transfer
5.) Scapulothoracic arthrodesis and Steindler flexorplasty (proximal flexor pronator
transfer)

Previously published studies on transfers to restore elbow flexion have rarely compared the
outcomes among different techniques. We found no comparison between the Steindler procedure
and pectoralis major tendon transfer. This study compared functional outcomes after these two
types of elbow flexorplasty performed in conjunction with shoulder arthrodesis for patients with
brachial plexus injuries. The same examiner assessed 14 patients who underwent shoulder fusion
and elbow flexorplasty. Five had a pectoralis major tendon transfer, and nine underwent a
modified Steindler procedure. All patients had brachial plexus lesions that were irreparable or
had not responded to neurolysis, repair, or grafting. Average follow-up time was 60 months
(range, 12 months-10 years). All patients completed a questionnaire (pain, function) and
underwent an objective assessment of range of motion, isometric strength, isotonic power, grip,
and pinch strength. The BTE Work Simulator (Baltimore Therapeutic Equipment, Hanover, MD)
was used for muscle strength testing. Improvement was noted in daily activities including
dressing and bilateral activities and was reflected in a significant improvement over preoperative
function. No differences (at p = .05 level of significance) were found in objective tests between
the pectoralis major transfer and the Steindler groups. In the patient requiring stabilization of the
shoulder and flexorplasty, the pectoralis major tendon transfer was at least equivalent to the
modified Steindler flexorplasty in terms of range of motion, strength, and subjective measures.
ANSWER

Beaton DE, Dumont A, Mackay MB, Richards RR: Steindler and pectoralis major flexorplasty:
A comparative analysis. J Hand Surg Am 1995;20:747-756.

109. An otherwise healthy 75-year-old woman who lives alone sustains the fracture shown in
Figures 28a and 28b. Management should consist of
1 Sling immobilization until comfortable, followed by functional rehabilitation
2 Open reduction and internal fixation using a blade plate
3 Hemiarthroplasty
4 Intramedullary nailing
5 Total shoulder arthroplasty

The correct answer is 3. In Ianottis article, Displaced Three- and Four Part Humeral
Fractures: Evaluation and Management, he outlines an algorithm for diagnosis and
treatment. In Figure 28b it can clearly be seen that this is a For Part Humeral Fracture with
fracture involving the Greater Tuberosity, Lesser Tuberosity, Humeral Head, and
Metaphyseal components. Furthermore there is notable displacement of both lesser and
greater tuberosities. Of note on the plain films is the patients obvious osteopenia. The
algorithm is as follows Three or for part fractures in patients who are not medically stable
for surgery get non operative treatment. Of those who are medically stable, those with either
Valgus impacted four part fractures or a young patient with good bone stock, an ORIF should
be attempted. Finally, those who are older with poor bone stock, a hemiarthroplasty is
recommended. The role of TSA and IM nailing is not recommended for fixation of these
fracture types. Therefore in our elderly patient with poor bone stock and a four-part
Humerus fracture, hemiarthoplasty is the preferred treatment.
ANSWER 3

110. A 34 year old man struck by a car sustains the closed injury shown in figure 29
(proximal tib/fib fx). The patient is awake and alert, has no dorsiflexion of the foot or toes,
and has decreased sensation on the dorsum of the right foot. Leg compartments are soft.
Compartment pressure measurements are 28,32,18, and 20 mmHg in the anterior, lateral,
superficial posterior, and deep posterior compartments. The blood pressure is 120/70mm Hg.
The ABI is .95. After appropriate splinting, the next step should consist of:
1. Fasciotomies of the anterior and lateral compartments without peroneal nerve
exploration.
2. Fasciotomies of all 4 compartments w/out peroneal nerve exploration.
3. Peroneal nerve exploration.
4. Lower extremity angiography
5. Continued observation in anticipation of stabilization procedures.

observation in anticipation of stabilization procedures. In a British study of 116 patients with


tibial diaphysis fractures, 3 had acute compartment syndrome. They used 24 hour continuous
monitoring and found that if a differential pressure (DBP-Compartment Pressure) was less than
30 they missed no cases. However, if an absolute pressure of 30mmHg was used, there would
have been 50 fasciotomies (mostly unnecessary). OKU 7 mentions that absolute pressures of 30
to 45 used to be used.
ANSWER

111. 30-year old woman has isolated knee pain and swelling exacerbated by activities. A
radiograph and MRI scans are shown in Figures 30a through 30c. A biopsy specimen is
shown in Figure 30d. What is the most likely diagnosis?
12345-

Synovial sarcoma
Synovial chondromatosis
Rheumatoid arthritis
Hemophilia
Pigmented villonodular synovitis

Pigmented villonodular synovitis (PVNS) is a rare reactive, inflammatory disorder that is of


unknown etiology. It is considered a reactive disorder rather than a true neoplasm but is locally
aggressive and has a high rate of reoccurrence. It is characterized by fibrohistiocytic
proliferation of the synovial membrane with villous and nodular protrusions. PVNS may be
either localized, diffuse or both, and may be intraarticular or extraarticular. PVNS is relatively
rare and accounts for less than 5% of all primary soft tissue tumors. The knee is most commonly
affected followed by the hip and shoulder. It occurs usually during the 3rd and 4th decades. An illdefined, periarticular soft tissue mass may be seen on plain films as is seen in figure 30a. MRI
may demonstrate a large effusion, low signal intensity on both T1- and T2-weighted images
(because of hemosiderin deposition), hyperplastic synovium and occasional bony erosions.
Histologically PVNS is characterized by the presence of hemosiderin-laden multinucleated giant
cells. The hemosiderin deposits on the biopsy specimen confirm this diagnosis.
Synovial chondromatosis is a metaplastic condition involving articular or tendon sheaths.
Cartilage nodules are formed in the synovial membranes and firm nodules may be found on
physical exam. The peak incidence is during the 5th decade and 70% of cases involve the knee.
Clinical presentation consists of pain, swelling and limitation of motion. Radiographs usually
demonstrate stippled or ring like nodules due to mineralization of the cartilaginous matrix.
Microscopically, hyaline or myxoid cartilage is found.
Synovial sarcoma is a highly malignant tumor that occurs within close proximity to joints but is
usually extraarticular. Mineralization is present on 25% of plain radiographs. Histologically
both epithelial cells in glands and spindle cells are present.
Rhematoid arthritis is a common inflammatory arthritis related to a cell mediated immune
response. It usually polyarticular and presents with an insidious onset of morning stiffness.
Radiographically periarticular erosions are common. The clinical presentation, imaging and
histology presented are not consistent with RA.
Both hemophilia A and B are X linked recessive bleeding disorders. Hemophilia A is due to
factor VIII deficiency while hemophilia B is due to factor IX deficiency. Both disorders are
characterized by repeated hemarthrosis after minor trauma leading to synovitis, cartilage
destruction and joint deformity. Because they are X linked, they almost always found in males.
ANSWER

Flandry F, Hughston JC: Pigmented villonodular synovitis. JBJS 1987;69:942-949


Dorman HE, Czerniak B: Bone tumors. St. Louis , MO, Mosby, 1998, pp 1041-1086

112. A 28 yo man sustained the injury shown in figure 31 is hemodynamically unstable. In


addition to fluid resuscitation, the next most appropriate step in management should include
12345-

angiography and embolization


an emergent ex lap
external pelvic fixation
ORIF
Closed reduction and perc pinning

External immbolization is indicated only if the pelvis ring is skeletally unstable. In the ED it
may consist of a sheet tied around the iliac crest for open book injuries or an ex-fix or pelvis
clamp with traction for more unstable injuries. The goal is to provide as much skeletal
stabilization as early as possible. It decreases blood loss by providing a tamponade, opposing
bleeding fractures edges, and limiting motion of the soft tissues so as not to break up the intial
pelvic clot. External skeletal stabilization is part of resuscitation. Angiography with selective
embolization is indicated for patients with stable or unstable pelvis fractures with external
fixation in whom other sources of bleeding have been ruled out.
The figure shows an open book pelvis where the pelvic volume is increased. Initial management
for a patient with this type of injury is to reduce the pelvic volume with external pelvic
stabilization. Initial external treatment, which may consist of sandbags and straps, beanbags, or
military antishock trousers (MAST), may be applied in the field. Although MAST stabilizes the
pelvis, they have been associated with decreased ventilatory ability, significant delay in transport
time, and compartment syndromes of the lower extremities. Deflation of MAST must be gradual
to avoid a sudden increase in the intravascular space, which can cause shock or cardiac arrest.
External stabilization in the emergency department may consist of a sheet tied around the iliac
crest for open book injuries or an external fixator or pelvic clamp with traction for more unstable
injuries. The goal is to provide as much skeletal stabilization as early as possible. External
stabilization decreases blood loss by providing a tamponade, opposing bleeding fracture edges,
and limiting motion of the soft tissues so as not to break up the initial pelvic clot. External
stabilization should be considered part of the resuscitation effort, and external fixation has been
demonstrated to decrease injury severity score-dependent mortality and transfusion requirements.
Candidates for angiography with selective embolization include patients with stable pelvic
fractures or unstable pelvic fractures that have been stabilized with external fixation and in
whom other sources of bleeding (for example, in the chest, abdomen, and retroperitoneum) have
been ruled out.
Patients with a stable pelvic injury may also have arterial bleeding. Up to 18% of patients with
stable pelvic injuries needing more than six units of blood in the first 24 hours required
angiographic embolization.
ANSWER

Cook RE, et al: The role of angiography in the management of hemorrhage from major fractures
of the pelvis. JBJS Br 2002; 84; 178-182
OKU 7. Rosemont IL, AAOS , 2002 pp 395-405

113.
A 68-yo man who has severe knee pain is no longer able to carry out routine
activities despite the use of NSAIDs and intra-articular cortisone injections. Radiographs are
shown in figure 32a through 32c. Treatment should now consist of
12345-

total knee arthroplasty with lateral soft tissue release


rotating hinge knee arthroplasty
simultaneous distal femoral osteotomy and total knee arthroplasty
combined distal femoral and proximal tibial osteotomy
distal femoral osteotomy

The success of total knee arthroplasty is dependent in part on the proper restoration of the
mechanical axis as well as soft tissue balancing. Most arthritic knees have some degree of
associated deformity and secondary imbalance of the soft tissue envelope but these tend to be
periarticular. As a result, they can be addressed easily with TKA as well as soft tissue release on
the concavity of the deformity.
In cases of severe extra-articular deformity, complex imbalance will exist of the collateral
ligaments and these are poorly addressed with resection of the intra-articular bone or soft tissue
releases. Such deformities are usually the result of fracture malunion, periarticular osteotomies
or metabolic diseases, such as Rickets and Paget disease.
In cases where more than 10 degree of deformity exists in the coronal plane and 20
degree in the sagittal plane, a simultaneous or staged distal femoral osteotomy and TKA will
limit the amount of imbalance and the need for use of constrained implants.
ANSWER

Explanation of Wrong Answer: 1- Does not address the extra-articular deformity present in this
patient. 2- Constrained implants limit function and should be avoided if another option is
available. 4,5: do not address the OA already present in this patient and are not likely to relieve
his pain significantly
Lonner JH et al: Simultaneous Femoral Osteotomy and TKA for treatment of OA associated with
severe Extra-articular deformity. JBJS 82A 342-348

114. Figure 33 shows the radiograph of a 48-year-old man who sustained a fracture-dislocation
of his dominant arm and a significant head injury in a fall from a roof. Eight days after injury he
is medically cleared for surgery. Treatment should consist of:
1. rotator cuff repair
2. open reduction
3. shoulder arthrodesis
4. total shoulder arthroplasty
5. humeral head arthroplasty
Goldman et al reviewed 26 hemiarthroplasties performed for acute 3 & 4 part proximal humerus
fractures. 73% had slight or no pain. ROM averaged 107 degrees of forward flexion and 31
degrees of external rotation. Strength and stability were rarely problematic. Common functional
limitations included lifting, carrying objects, and using the hand at or above shoulder level. This
study demonstrates that hemiarthroplasty for acute proximal humerus fractures offers predictably
painfree shoulders, but ROM and functional recovery are less predictable.
Norris et al treated 23 patients with failed treatment of 3 and 4 part proximal humerus fractures
with shoulder arthroplasty. 10 patients were initially treated closed and 13 were initially treated
open. 17 patients were treated with TSA and the remaining 6 were treated with hemiarthroplasty.
Reduction in pain was noted in 95% of patients. Forward elevation increased from 68 to 92
degrees, while external rotation increased from 6 to 27 degrees. 53% were able to perform
activities at or above shoulder level. Late arthroplasty is difficult with results inferior to acute
arthroplasty. However, late arthroplasty remains a satisfactory option if primary treatment
(closed, ORIF) fails.
The figure in this question demonstrates a complex 4 part fracture dislocation of the proximal
humerus. This type of fracture pattern is associated with a high incidence of osteonecrosis,
particulary in this case in which surgery is delayed for 8 days. Therefore, prosthetic replacement
is likely to provide the best outcome. Since the patient does not appear to have significant
degenerative change in his glenoid, hemiarthroplasty without glenoid resurfacing is indicated.

References:
Goldman RT et al: Functional outcome after humeral head replacement for acute 3- and
4-part proximal humerus fractures. JSES 1995; 4:81-86.
Norris TR et al: Late prosthetic shoulder arthroplasty for displaced proximal humerus
fractures. JSES 1995; 4:271-80.
ANSWER: 5

115.

A 10-year-old boy sustains a 100% displaced closed transverse midshaft femoral fracture.
Neurovascular examination is intact. Initial radiographs show a 5-cm overriding of the
fragments with no comminution. Management should consist of
1.
2.
3.
4.
5.

Closed reduction with immediate hip spica casting.


Skin traction, followed by hip spica casting in 3 weeks.
A rigid reamed intramedullary antegrade nail, entering at the piriformis fossa.
A rigid unreamed retrograde nail.
Two flexible intramedullary nails.

First off, lets discuss the obviously wrong answers. Answers 2, 3, and 4 are no-brainers.
Skin tra `ction for 3 weeks is unacceptable because we know that skin traction cannot be
maintained for 3 weeks w/o skin complications. Rigid IM nailing, either antegrade or retrograde
is not an option. With the use of a piriformis starting point, risk of AVN to the femoral head is a
concern in a skeletally immature patient. A retrograde nail would damage the distal femoral
physis in a skeletally immature patient.
That leaves 1 and 5 as very viable alternatives. Heinrich et. al. argued that flexible nails
offer results just as good as spica casting but with less disruption to family life and a shorter
hospitalization. They point out that psychoses and both emotional and gastrointestinal
disturbances result from spica casting. They also contend that surgical intervention facilitates
nursing care and the medical management of patients with multi-system inury and multiple
fractures.
They performed a prospective analysis of 78 femur fractures in 77 children. All were
treated with flexible intramedullary nails. All children achieved symmetric hip and knee motion.
Eighty percent of the children had a normal varus/valgus alignment. 92% had normal AP
alignment. In their study also, they compared outcomes in children 6-9 versus those >/= 10
years of age. They found that flexible IM nails offer excellent fixation for all kids, but AP and
rotational malalignment was less freqent in the patients >/= to 10 years of age. Their conclusions
were that flexible IM nailing provided comparable results to those achieved with traction/casting
techniques in children aged 6-9 years old and better results than traction/casting techniques in
children >/= 10 years of age (for the aformentioned reasons).
ANSWER

Heinrich, S.D., Drvaric, D.M., Darr, K., MacEwen, G.D. The Operative Stabilization of
Pediactric Diaphyseal Femur Fractures and Flexible Intramedullary Nails: A Prospective
Analysis. J Ped Ortho. 1994;14:501-507.

116. A cognitively intact 83-year-old woman who lives with her daughter sustains a proximal
humeral fracture. Which of the following observations is most likely to lead to suspicion of elder
abuse?
1. Contusion on the patients forehead
2. Disparity in histories from the patient and her daughter
3. History of a femoral neck fracture
4. Spiral fracture pattern
5. Ecchymosis extending down arm to the elbow.
Approximately 1-2% of elderly persons living in their own homes are abused. Abusers are
predominantly adult children, spouses, and other relatives, and are often financially dependent on
the abused person. Some of the presentations that suggest abuse or neglect of an elderly patient
include:
1. Delays between injury or illness and the seeking of medical attention
2. Disparity in histories and the patient and suspected abuser or a history that is given
solely by the caregiver (different mechanism or chronology of injury)
3. Implausible or vague explanations (fracture not explained by mechanism)
4. Frequent visits to the emergency room despite plan for medical care
5. Presentation of a functionally impaired patient without his/her caregiver
5. Laboratory findings that are inconsistent with history given

ANSWER

:2

Chen AL, Koval KJ: Elder abuse: The role of the orthopaedic surgeon in diagnosis and
management. J Am Acad Orthop Surg 2002; 10: 25-31.

117. What is the predominant cell type found in fracture callus just prior to calcification of the
chondroid callus?
12345-

ANSWER

chondroclasts
osteoblasts
osteocytes
hypertrophic chondrocytes
proliferative chondrocytes

BUckwalter J; Orthopaedic basic Science, 2000, p371-399.

118. A 17 year old boy sustains a wound to his proximal forearm from a handgun.
Radiographs reveal a radiopaque projectile in the soft tissues of the midforearm but no fractures.
Examination reveals that he has no wrist or finger extension and he reports minimla pain with
passive finger motion. Initial management should consist of
12345-

ANSWER

surgical debridement of the wound and removal of the bullet


surgical debridement of the wound and explooratin of the radial nerve in the
proximal forearm
local wound care and exploration of the radial nerve in2-3 weeks if ther are no
signs of infection.
local wound care and immediate electromyography
local wound care and delayed electromyography

Katzman Bm, et.al. Peripheral nerve injuries secondary to missiles. Hand Clinics 1999; 15;
233-244.

119. Figure 34 shows the radiograph of a 77 yo woman who underwent THA 10 yr ago. What is
the predominant cause of the proximal femoral bone loss?
1.
2.
3.
4.
5.

Stress shielding
polyethylene debris-induced osteolysis
senile osteoporosis
modulus of elasticity of the femoral stem
diffuse osteopenia

ANSWER:

The radiograph shows a long full-coat stem well fixed distally with proximal bone loss due to
stress shielding.

120. The retrocalcaneal bursa is located between the


1)
2)
3)
4)
5)

ANSWER

skin and Achilles tendon


skin and superior calcaneal tuberosity
posterior calcaneal tuberosity and Achilles tendon
posterior and superior calcaneal tuberosities
Achilles tendon and superior calcaneal tuberosity

It just is. Anatomy. Learn it. Love it.

121. A 52-year-old woman has had a painful planovalgus foot deformity for the past several
years. Immobilization, shoe modification, and an orthotic have failed to provide relief.
Examination reveals that she is unable to perform a single stance heel rise. Hindfoot
valgus and forefoot supination are not manually correctable. Treatment should consist of
1- Achilles tendon lengthening and triple arthrodesis.
2- Posterior tibial tendon debridement and flexor digitorum longus tendon transfer
3- Posterior tibial tendon reconstruction and medial displacement calcaneal osteotomy
4- Posterior tibial tendon reconstruction and lateral column lengthening
5- Subtalar arthrodesis.
The classification system originally developed by Johnson and Strom in 1989 is considered
useful in the management of tibialis posterior tendon insufficiency.
Stage I
o Swelling, pain, inflammation, and often effusion within the tibialis posterior
tendon sheath.
o Irritability is noted with passive eversion of the foot along the course of the
tibialis posterior tendon.
o Mild weakness to manual testing may be present; however, no deformity of the
foot is demonstrated when compared with the opposite foot.
o The patient is able to invert the foot actively on a double-leg toe raise test and is
able to perform a single-leg toe raise as described in the next section.
Stage II
o Loss of function of the tibialis posterior tendon and inability to perform a singleleg toe raise
o Attempted compensation by use of the tibialis anterior muscle and tendon unit as
an accessory inverter of the hindfoot.
o Hindfoot remains flexible.
o With the hindfoot in neutral the forefoot can be brought into neutral.
o Generally, mild lateral or sinus tarsi impingement pain is present.
Stage III
o Loss of function of the tibialis posterior tendon occurs
o Fixed hindfoot deformity with valgus abduction occurs, and degenerative changes
may be apparent on roentgenograms.
o Significant lateral sinus tarsi pain is present.
Stage IV disease was described by Myerson et al.
o Valgus positioning of the ankle joint in addition to stage III findings.
The treatment regimens are as described in the question. First attempt immobilization and / or
shoe modification. If these are unsuccessful, and the patient still has a flexible, correctible
deformity then a calcaneal sliding osteotomy can be performed. If the patient has a RIGID
deformity, then a triple arthrodesis needs to be done. The Achilles lengthening needs to be done
because as the foot goes into valgus, the relative length of the Achilles tendon becomes shorter
and contracted.

ANSWER

Mann RA: Flatfoot in adults, in Coughlin MJ, Mann RA (eds): Surgery ofthe Foot and Ankle, ed
7. St Louis, MO, Mosby, 1999, pp 733-767.
Fortin PT. Walling AK: Triple arthrodesis. Clin Orthop 1999;365:91-99.

122. Figures 35a and 35b show the radiographs of a 70-yearold patient who has sever left
shoulder pain. What is the most likely diagnosis?
12345-

ANSWER

chondrosarcoma
tumoral calcinosis
pigmented villonodular synovitis
synovial chondromatosis
myositis ossificans

Milgram JW; Synovial osteochondromatosis, A histopathological study of 30 cases. JBJS 1977;


59; 792-801.

123. An 11-month old infant has arthrogryposis multiplex congenital. Examination reveals
flexion, abduction, and external rotation hip contractures. Radiographs reveal that both hips are
dislocated. Management of the dislocations should consist of
1. A pavlik hardness
2. Adduction traction
3. Closed reduction under anesthesia, adductor longus tenotomy, and a spica cast
4. Open reduction
5. Femoral head resection

---------------------------------------------------------------------Yau P et al. Twenty-Year Follow-up of Hip Problems in Arthrogryposis Multiplex Congenita


Journal of Pediatric Orthopaedics. 2002; 22:359363
Arthrogryposis is a spectrum of disease characterized by congenital contracture of joints
in multiple areas. The most common form is amyoplasia, in which there is symmetric
involvement of the joints in both the upper limb and the lower limb. The reported incidence of
hip problems in arthrogryposis congenita multiplex ranges from 65% to 80% (1, 4,7). These
include contracture, subluxation, and dislocation. However, there are only a few papers in the
literature reporting the long-term results of the treatment of hip problems in arthrogryposis. In
this paper, we report the result of hip problems in patients with 0arthrogryposis after a mean
follow-up of 20 years.
Staheli L et al. Management of Hip Dislocations in children with Arthrogryposis. Journ Ped
Ortho. 1987; 7:681- 685.
131 patients with arthrogryposis. 18 hip dislocations. ROM of patients treated by open
reduction following the medial approach was greater than in those treated by anterolateral
incision and greater than in bilateral cases treated by closed reduction.
ANSWER

124. A 22-year-old man sustains a proximal tibial fracture at the junction of the metaphysis and
the diaphysis. Which of the following fracture site deformities are most commonly associated
with intramedullary nailing using a starting point well centered on the AP view?
12345-

Posterior angulation and varus


Posterior angulation and valgus
Posterior angulation and internal rotation
Anterior angulation and valgus
Anterior angulation and varus

Valgus, apex anterior angulation, and residual displacement at the fracture site are common after
nailing of proximal third tibia fractures. Surgical errors of a medialized nail entry point and a
posteriorly and laterally directed nail insertion angle contributed to malalignment. Freedman and
Johnson saw malalignment (greater than 5 degrees) in 58% of proximal third fractures, 7% of
middle third fractures, and 8% of distal third fractures. Proximal third tibial fractures may require
a neutral or slightly lateral entrance angle to ensure a more anatomic reduction and
centromedullary nail orientation to offset the tendency for valgus angulation.
ANSWER

Freedman EL, Johnson EE: Radiographic analysis of tibial fracture malalignment following
intramedullary nailing. Clin Orthop 1995;315:25-33.
Lang OJ, Cohen BE, Bosse MJ, Kellam JF: Proximal third tibial shaft fractures: Should they be
nailed? Clin Orthop 1995;315:64-74.
Koval KJ (ed): Orthopaedic Knowledge Update 7. Rosemont, IL, American Academy of
Orthopaedic Surgeons, 2002, pp 479-488.

125.

A 35-year-old man sustains a dislocation of his dominant shoulder in a fall. The shoulder
is reduced and placed in a sling, but he returns 6 hours later with the shoulder dislocated
again, despite use of the sling. A CT scan is shown in Figure 36. Management should
now consist of
12345-

replacement of the sling with a modified spica cast.


open reduction and internal fixation.
percutaneous pin fixation.
arthroscopic labral repair.
Arthroscopy and removal of the loose fragment.

Despite appropriate treatment with closed reduction and a sling, this gentleman remains grossly
unstable. The most common cause of this is a glenoid rim fracture, although a substantial labral
tear could possibly do this as well. The CT scan demonstrates a glenoid rim fracture. Such
instability will require a surgical approach. Removal of the fragment will only make the
shoulder more unstable. Percutaneous pinning is not a good idea around the glenoid rim and its
various neurovascual structures in the triangular and quadrangular spaces. Thus the realistic
answers are arthroscopic labral repair and ORIF. Since there is a fracture of bone demonstrated
on the CT scan, the correct answer is ORIF.
Several investigators have been working on the indications for arthroscopy. In 2004, small labral
lesions can be repaired both by open procedures and arthroscopically. However, the literature
shows in high demand athletes (specifically NFL football players), that open repair has better
long term results. Open and arthroscopic procedures tend to function equally in lower demand
individuals. Glenoid rim lesions which require repair should be done through open incisions. If
the fragment is small enough and does not lead to instability, it can be excised arthroscopically.
Cofield fixed 10 glenoid rim fractures through an open posterior approach with 90% good
results.
ANSWER

open reduction and internal fixation.

Bigliani LU, et.al.: Glenoid rim lesions associated with recurrant anterior dislocation of the
shoulder. Am J Sports Med 1998; 26: 41-45.
Kavanagh BF, et.al.; Open reduction and internal fixation of displaced intra-articular fractures of
the glenoid fossa.. JBJS 1993; 75; 479-484.

126. What is the major advantage of allowing early active motion of a repaired zone II flexor
tendon injury?
1 increased tendon excursion
2 greater repair strength
3 less postoperative pain
4 better patient compliance
5 faster tendon healing

The development of adhesions between tendons or between tendons and the tendon
sheath is a serious problem in tendon healing. Experimentally, it has been shown that early
mobilization enhanced tendon healing and significantly reduced tendon adhesions. Clinically, it
has been observed that early protective mobilization is more likely than tendon immobilization to
maximize tendon gliding (excursion) and healing. DIP joint motion produces 1 mm of
differential excursion for each 10 degrees of joint flexion. However, passive flexion of the DIP
joint has been noted experimentally to result in buckling of the tendon distal to the A3 pulley. In
the cited reference by Horii, it was found the utilization of a synergistic wrist splint, provided
greater tendon excursion with passive finger extension and flexion synergistic with dynamic
wrist motion. Basically active motion makes the tendon glide while passive motion may make it
bunch up in the tendon sheath.
Early controlled mobilization of the fingers after direct repair of flexor profundus tendons
in zone II has been said to improve the final result and is thought to decrease the formation of
peritendinous adhesions by mobilizing the repaired tendons in relation to their surrounding
tissues. Experimental data indicate that a correct tendon suture will withstand the tensile force of
active flexion in closing a fist during the weakest phase of healing. Therefore, the introduction
of an active component of finger flexion seems suitable if passive flexion programs prove to be
insufficient.
ANSWER

:1

JHS 1991 16 669 Hagberg tendon excusion and dehiscence


JHS 1992 17 559 Horii comparitive flexor tendon excurions

127. Item deleted.

128. A 22 y/o laborer sustains the injury shown in Figures 37a and 37b. Figures 37c and 37d show the
postoperative radiographs obtained after application of a short leg cast. Management should now consist of
1.
2.
3.
4.
5.

revision of the fibular fixation


repair of the deltoid ligament
arthroscopy and retrieval of the osteochondral fragment
ORIF of the medial malleolus fracture
reduction and fixation of the syndesmosis

The original radiographs demonstrate a distal fibular fracture (Weber B) with a widened mortise. The
fact that the mortise is widened indicates that there is a disruption of the deltoid ligament. Ankle
biomechanics require ORIF of the fibula as was completed. Post operative radiographs demonstrate
appropriate alignment and fixation of the lateral malleolus with persistent widening of the medial mortise.
The reason for this is due to disruption of the tibia/ fibula syndesmosis. While not as common in Weber B
fractures, appropriate reduction and fixation of the fibula should have stabilized and closed down the mortise
which is obviously not the case. Without a concentric ankle mortise, the ankle joint will be unstable and
progress to arthritis.
The treatment for this condition is reduction and fixation of the syndesmosis (5). Fixation is usually
completed with a screw from the fibula to the tibia distally. (Which needs to be removed later or else it will
break.) Often there may be scar tissue or a loose body in the medial joint which may needed to be removed.
However, merely removing a piece blocking reduction will not change the fact that the syndesmosis has been
disrupted. Thus arthroscopy and retrieval of the osteochondral fragment (3) is not adequate. The fibula has
been fixed anatomically and does not need to be revised (1) nor is the medial malleolus even fractured (4).
While repairing the deltoid ligament may help his medial stability this is rarely necessary and does not change
the fact that the syndesmosis is disrupted. Even with an intact deltoid ligament, the tibia can still displace
medially.
ANSWER

5.

reduction and fixation of the syndesmosis

References:
Browner Jupiter Skeletal Trauma 1998 pp2327-2404.

129. A 12 yo boy has pain in the medial arch of his left foot with weight bearing activities.
Nonsurgical management has failed to provide relief. Radiographs show and Ogden type II
accessory navicular (accessory ossicle joined to the prominence of the navicular by a
synchondrosis). Treatment should consist of
1.
2.
3.
4.
5.

ANSWER

arthrodesis of the ossicle to the navicular


excision of the ossicle and the navicular prominence
talonavicular arthrodesis, with elevation of the medial arch
calcaneal neck lengthening by opening wedge osteotomy and bone graft
subtalar joint arthrodesis

: 2

Accessory navicular is the most common accessory bone in the foot, 4-14% of the population.
Type I: pea-sized sesamoid in the center of the most distal portion of the tibialis posterior
tendon
Type II: most frequently symptomatic type; ossicle joined to the tuberosity of the
navicular by a synchondrosis or syndesmosis
Type III: large, horn shaped navicular, probably from fusion of a type II
Treatment with shoe stretching or casting for 4-6 weeks. Surgical excision is recommended for
symptoms refractory to immobilization. Simple excision has >90% good to excellent results.
Lovell and Winters Pediatric Orthopaedics

130. A patient underwent shoulder arthroplasty for cuff tear arthropathy 8 months ago. Prior to
surgery, he was able to elevate his arm 90. Postoperatively he is unable to elevate his arm
and he has significant cosmetic deformity. A postoperative radiograph is shown in Figure
38. What is the most likely explanation for his inability to elevate the arm?
12345-

The coracoacromial arch is disrupted.


The humeral component is too large.
The supraspinatus muscle was not repaired.
The coracohumeral ligament was divided.
The glenoid was resurfaced.

References:
Lee TQ, Black AD, Tibone JE, McMahon PJ: Release of the coracoacromial ligament
can lead to glenohumeral laxity: A biomechanical study. J Shoulder Elbow Surg 2001;10:68-72.
Bigliani LU, Levine WN: Subacromial impingement syndrome. J Bone Joint Surg Am
1997;79:1854-1868.
ANSWER

Explanation:
The coracoacromial arch is superficial to the rotator cuff and consists of the acromion, the coracoacromial
(CA) ligament, and the coracoid itself. The CA ligament extends from the outer edge of the coracoid and widens to
insert onto the anteromedial aspect and the undersurface of the acromion. The coracohumeral (CH) ligament forms
the superficial roof of the rotator interval (with the interval being between the supraspinatus and subscapularis
tendons).
Compression of the rotator cuff and subacromial bursa between the CA arch and the humerus is a well
established clinical finding resulting in shoulder pain that is worse with overhead activity. Upon failure of
conservative management, surgical treatment for this impingement syndrome has often included the release of the
CA ligament as it has been thought to have no functional importance.
In the reference listed above, Lee and his colleagues found that the CA ligament has a role in the static
stabilization of the glenohumeral joint at lower levels of abduction. In addition, it has been known that the CA
ligament limits superior humeral head translation.
Based on the radiographs shown in Figure 38, it is readily apparent that the humeral component has
migrated superiorly. This migration is a result of disruption of the coracoacromial arch and CA ligament. Thus the
preferred response is 1.

131. A 64-year-old man who underwent revision total knee arthroplasty 6 months ago has
leg pain after walking. AP and lateral radiographs are shown in Figures 39a and 39b.
The cause of pain is most likely related to:
1. prior patellectomy
2. tibial stem position
3. limb malalignment
4. the posterior stabilized implant
5. the notched femoral component

Radiographs show a diaphyseal engaging tibial stem. Barrack, et al studied pain at the end of
the stem after revision total knee arthroplasty. They found that localized pain at the end of the
stem was present on the femoral side in 11% of their patients and in 14% of patients studied with
press fit tibial stems. Patients with press fit stems who had pain at the end of the stem were
significantly more likely to be dissatisfied with the results of the surgery and the degree of pain
relief. There was a tendency for the percent canal fill on the tibial side to be higher in patients
who experienced pain at the end of the stem versus those without pain at the end of the stem
(71% versus 64%). There was no significant difference in stem diameter between patients with
and without pain at the end of the stem. Patients without pain, in fact, averaged a millimeter
larger stem diameter. As with thigh pain after cementless hip replacement, the etiololgy of pain
at the end of the stem in revision total knee arthroplasty remains uncertain. The location of the
pain in the diaphysis suggests there may be stress concentration or stress transfer to the tip of the
stem. This conclusion is supported by the findings of bone scans of the patients in whom this
was performed. The trend toward more shin pain with higher degrees of tibial canal fill suggests
the surgical technique and patient anatomy may also play a role.
ANSWER

132. Persistent chronic draining osteomyelitis of the tibia is most likely to develop into what type
of malignancy?
1.
2.
3.
4.
5.

multiple myeloma
osteosarcoma
chondrosarcoma
melanoma
squamous cell sarcoma

Osteomyelitis is considered chronic if it persists for greater than 6 weeks. In those


patients with chronic osteomyelitis and a concomitant draining sinus, there is a 0.5% chance that
the draining sinus will develop into a squamous cell cancer (Ortho. Clinics of N. America. 29 (1),
Jan 1998. This transformation occurs secondary to metaplasia in the epithelial lining of the sinus
tract.
In their paper, Malignant Lesions Arising in Chronic Osteomyelitis, Clinical Ortho.
and Related Research, 362, 181-189, McGrory et al. present their findings in 53 patients with
malignant transformation of chronic osteomyelitis. In their series, the etiology of the
osteomyelitis was post-traumatic in 32 patients and hematogenous or spontaneous in 21 patients.
The tibia was the most common site of the osteomyelitis, followed by the femur and the foot.
The mean age of the patients was 59, there were 44 men and nine women. The draining
osteomyelitis was present for an average of 34 years prior to malignant degeneration and foul
smelling discharge and pain were the most common presenting symptoms of malignancy. A
fungating mass was present in 18 patients, an ulcerated skin lesion in six, and no lesion in 29
patients.
The etiology of the malignant transformation is unclear; according to the authors,
however, they contend that it is likely a continuum, in which cellular proliferation begins in
reaction to the chronic drainage. This continues, evolving from simple hyperplasia to
pseudoepitheliomatous hyperplasia, carcinoma in situ, and finally invasive carcinoma.
In their series, 39 of 49 patients with SCC underwent amputation, with five patients
developing local recurrence and four developing metastases. Seven patients underwent local
excision and one patient, with a femoral lesion, underwent radiation therapy. Amputation is
generally considered the most reliable means of treatment.
In a more recent study, Saglik and his collegues demonstrated the correlation between
chronic osteomyelitis and squamous cell carcinoma. They collected 13 cases of squamous cell
carcinoma arising from chronic osteomyelitis between February 1991 to March 1999. The mean
age of those treated was 56.5. Twelve patients underwent amputation of the involved limb and
one underwent a limb salvage procedure. Average follow up was 56 months. Among those 13
patients treated, two died of the disease. Historically, the treatment of choice for these tumors
arising from chronic osteomyelitis has been amputation.(Int Orthop-01 Jan-2001; 25(6); 389-91)
ANSWER

133. A 45-year-old man recently underwent a transfemoral amputation as a result of a traumatic


event. The patient desires to remain very active and can walk at a very fast pace. What type of
prosthetic knee joint is most appropriate?
1) Hydraulic
2) Pneumatic
3) Constant friction
4) Polycentric
5) Manual locking

A hydraulic (fluid control) knee is best used in active patients who prefer greater utility and
variability at the expense of more weight. Manual locking has limited indications and is used in
weak, unstable patients, those patients just learning to use prostheses, and blind amputees.
Constant friction is most commonly used in children. Its major disadvantage is that it allows
only single speed walking, and relies on alignment for stance stability so isnt recommended for
older, weaker patients.
ANSWER

134: Figure 40 shows the radiograph of a patient who has arm pain. What is the best initial
course of action?
1.) MRI
2.) Skeletal Survey
3.) Open biopsy
4.) Immobilization
5.) Methylprednisolone injection

A unicameral bone cyst is probably not a true neoplasm. Unicameral bone cysts occur usually in
the first two decades of life, and males predominate two to one. Most cysts are in the proximal
humerus or proximal femur, but any bone in the extremities may be involved. The
roentgenogram usually is characteristic, and biopsy for diagnosis ordinarily is not necessary. The
cyst usually is asymptomatic unless a fracture or impending fracture is present. About 50% of
patients have a complete pathological fracture, and another 25% have infraction of a thinned
cortex. Treatment is indicated when repeated fracture is likely. Although obliteration of a
unicameral cyst after healing of a fracture does occur, it is not the rule. Those cysts that abut a
physis have in the past been termed active and have a greater incidence of persistence after
treatment than those separated from the physis by normal bone, called inactive cysts. With
advancing age the cysts tend to move away from the physis, and many reports document
decreased frequency of recurrence after age 10 years. Growth disturbance may occur, regardless
of whether the cyst is treated. The traditional method of treating unicameral cysts has been
curettage with or without bone grafting. Recurrence of a degree requiring reoperation after
curettage occurs in about 25% of patients.
Aspiration of the cyst followed by instillation of methylprednisolone has been found to produce
satisfactory results in about 80% of patients, although in a study by Goel et al. only one of three
unicameral cysts of the calcaneus responded to this treatment. The technique requires the
insertion of two needles with stylets into the cyst under roentgenographic control. The stylets are
removed and the cyst fluid is aspirated. If fluid is not found, open biopsy is advisable to rule out
the presence of another lesion. If fluid is aspirated, one needle is removed, and 40 to 200 mg of
methylprednisolone is instilled. Some authors advise a cystogram using water-soluble contrast
material to ensure that the injected drug reaches the entire cyst cavity. If necessary, injections are
repeated at 2-month intervals. Approximately 75% of patients require more than one injection.
Canale: Campbell's Operative Orthopaedics, 10th ed., Copyright 2003 Mosby, Inc. 796797.

ANSWER

135. Which of the following cell types is the key initiator of the events associated with
periprosthetic cells?
1 Osteoblast
2 Osteocyte
3 Osteoclast
4 Neutrophil
5 Macrophage

The correct answer is 5. Periprosthetic cells are activated by a systemic inflammatory response
mediated by a number of hematologically significant messengers. Of the choices listed above,
macrophages are the only component which is increased during an inflammatory response that is
not only an effective producer of a cascade of chemical messengers which initiate the activities
of periprosthetic cells, but also are available to carry out many of the above listed tasks. .
Osteoblasts and Osteoclasts are involved in breakdown and production of Osteocytes, while
neutrophils are generalized inflammatory cells which in turn recruit macrophages to the site of
irritation/ inflammation.
ANSWER

136. Figs 41A and 41B show the radiograph of a 28 year old man who sustained a twisting injury
to his lower extremity while playing soccer. Closed reduction under anesthesia is unsuccessful.
What structure is most likely blocking reduction?
1.
2.
3.
4.
5.

Os trigonum
Anterior Tibial Tendon
Posterior Tibial Tendon
Extensor Digitorum Longus
Extensor Digitorum Brevis

Answer: subtalar dislocation is defined as the simultaneous dislocation of the subtalar and
talonavicular joints without associated dislocation of the calcaneocuboid or tibiotalar joints, and
without talar neck fracture. Medial dislocations are 4 times as common. Prompt reduction is
essential to minimize skin necrosis and circulatory compromise. Closed reductions cannot be
achieved in approximately 5 to 10% of medial dislocations and 15-20% of lateral dislocations.
Blocks to closed reduction include extensor digitorum brevis (MEDIAL DISLOCATIONS) and
Posterior tibial tendon (LATERAL DISLOCATIONS). If unstable reduction occurs, the presence
of a large intraarticular fracture must be ruled. Reduction and fixation of the of the fragment will
often stabilize the joint. CT scan is recommended after reduction. Excision or fixation of
fragments is recommended.
ANSWER

137. A 19-year old college football player reports cramping pain in his left calf and a
tingling sensation in his toes following vigorous exercise. Examination is
unremarkable other than slightly diminished pulses with the knee in hyperextension and
the foot in dorsiflexion. Pre- and post exercise compartment pressures show a
reduction in the pressures following exercise. What is the most likely diagnosis?
12345-

Neurilemoma of the popliteal nerve


Superficial peroneal nerve entrapment
Popliteal artery aneurysm
Popliteal artery entrapment syndrome
Exercise-induced transient compartment syndromes

Popliteal artery entrapment syndrome (PAES) is a relatively rare disorder caused by entrapment
of the popliteal artery leading to claudication of the lower extremity. It can mimic exercise
induced compartment syndrome and has four variants depending on the site and cause of artery
entrapment. It typically occurs in young athletic individuals and patients report cramping pain in
the calf with activity. Tingling may be present. The popliteal artery is usually entrapped by the
medial head of the gastrocnemius muscle so pulses may be diminished or absent with
gastrocnemius tightening (knee hyperextension and foot dorsiflexion). A popliteal aneurysm
does not cause exercise induced ischemia and a palpable mass can often be felt.
The superficial peroneal nerve is a branch of the common peroneal nerve and courses through
the anterolateral compartment to innervate the peroneus brevis and peroneus longus. The nerve
can become entrapped as it exits the deep fascia and becomes superficial. Multiple ankle sprains
are a risk factor. Patients usually complain of pain, paresthesia and numbness over the lateral
border of the distal portion of the calf and the dorsum of the foot and ankle.
Neurilemoma is a benign tumor of nerve sheath origin (schwann cell). It is usually solitary and
present on the surface of a peripheral nerve. Patients present with a painful nodule associated
with a positive Tinels sign in the distribution of nerve.
Exercise induced compartment syndrome is caused by elevated compartment pressure that
results in ischemia. Patients complain of pain with an insidious onset during exercise which
usually resolves with rest. Radiating pain, numbness and tingling may occur along the course of
affected nerves. Compartment pressures can be monitored to help in the diagnosis but the
guidelines are not agreed on. Generally compartment pressures that are less than 15 mm Hg preexercise, greater than 30 mmHg one min post-exercise and return to less than 15 mm Hg 5
minutes later are considered positive for exercise induced compartment syndrome. Arterial flow
remains normal.
ANSWER

Clanton TO: Athletic injuries to the soft tissues of the foot and ankle, in Coughlin MJ, Mann RA:
Surgery of the Foot and Ankle, Ed 7 St. Louis, MO 1999.
Radonic et al. Popliteal Artery Entrapment Syndrome: Texas Heart Institute Journal;27(1)313,2000.

138. Examination of an 11 yo boy reveals a gait the includes bilateral trendelenburg lurch and a
right short leg pattern. He has accentuated lumbar lordosis and a protuberant abdomen. He has
no pain. Radiographs are shown in 42a and 42b, Treatment should consist of
12345-

open reduction both hips


open reduction of the left hip and arthrodesis of the right hip
bilateral greater troch advancement
bilateral THA
epiphyseodesis at an appropriate time to achieve limb length equality at completion of
growth

The diagnosis is untreated DDH with bilateral hip dislocations. At this stage in the patients
disease and given the above choices the only option is 5. I cannot find a specific reason for the
answer other than to achieve limb length equality.
After a certain age, even the most skillful reduction of a dislocated hip is unlikely to produce a
mobile, pain-free hip over the long term. Conversely, many high dislocations may remain mobile
and pain free for decades, despite an inefficient gait. Bilateral pain-free dislocations discovered
after age 6 or 7 years probably should not be reduced. This rules out answer 1. Unilateral
dislocations usually create additional difficulties with limb-length discrepancy and spinal
malalignment. Unilateral dislocation therefore should be strongly considered for reduction when
discovered at any age up to adolescence. After skeletal maturity, arthroplastic reconstruction is
usually the primary treatment of choice for the symptomatic dislocation. Thus answer 4 is not
correct because the patient is not skeletally mature.
The question states that the child has a limb-length discrepancy. Since he has bilateral
dislocations but no pain he could tolerate his dysfunction until he reaches skeletal maturity.
Therefore, correction of the limb-length deformity is the higher priority so that he does not
develop a spinal deformity. Therefore answer 5 is correct.
ANSWER

Lovell and Winters Pediatric Orthopaedics ed 5. Philadelphia PA, Lippincott Williams and
Wilkins, 2000 905-956
OKU 7. Rosemont IL, AAOS , 2002 pp 387-394

139. Which of the following tests is considered most accurate (sensitive and specific) in
detecting spinal infections
12345-

Indium 111-labeled WBC scan


Technetium TC 99m Scan
Combined technetium Tc 99m and Gallium 67 Citrate Scan
CT
Plain Radiograph

Imaging is an invaluable tool in assessing patients with suspected infections.


Radiographs are usually the first screening tool used and may demonstrate progressive osteolysis
and end plate destruction. These findings are usually best seen on the AP view. As the disease
progresses, the disc space will narrow and eventually collapse. These changes, however, may
not be seen for up to several weeks after the process has begun.
CT scan is useful in delineating the extent of bony destruction and soft tissue extension
and is helpful in preoperative planning. It may not be very usefull, especially early in the
process for detecting these changes.
Technetium Tc 99m scan has about a 90% sensitivity but is nonspecific, especially in
adults with degenerative joint disease. This study is dependent on local blood flow and may
result in false-negative (affects sensitivity) results in areas of relative ischemia.
When combined with gallium 67 citrate scans, Technetium Tc 99 scans become both
highly sensitive and specific. The marker in the gallium scan is an analog of ferritin and is
secreted by leukocytes at the sites of infection. Gallium scan also normalize during the recovery
phase and may be useful in evaluating treatment response. It is important to recognize however,
that gallium scans are not useful in leukopenic patients and may not detect low-virulence
organisms.
Indium 111-labeled WBC scan have poor sensitivity in vertebral osteomyelitis (17%) and
are not recommended
ANSWER

Reference:
Tay BK et al: Spinal Infections. JAAOS 2002; 10: 188-197

140. A 25-year-old man has had medial knee pain with sports for the past 4 weeks. He has no
swelling, locking, or giving way. Examination reveals no effusion, 5/5 quadriceps strength,
medial joint line tenderness, AROM from 0 to 140 degrees, and negative Apley and
McMurray tests. Radiographs are normal. A sagittal proton density-weighted image is
shown in Figure 43. What is the next most appropriate step in management?
1. arthroscopy with meniscus repair
2. arthroscopy with partial meniscectomy
3. arthroscopy with open meniscus cyst exploration
4. aspiration of meniscus cyst
5. observation

Shepard et al retrospectively identified 76 anterior horn meniscal tears on MRI. 31 patients


underwent arthroscopy, of which 74% had intact anterior horns and (false-positive MRI) and
58% had intact anterior and posterior horns.
Shankman et al arthroscopically evaluated 22 patients who had speckled increased signal
intensity at the anterior horn of the lateral meniscus on MRI. Arthroscopy revealed that the
anterior horn of the lateral meniscus was normal in all 22 patients.
These studies the high false-postive rate associated with the finding of an isolated anterior horn
tear on MRI. As always, correlation with the patients presenting symptoms and exam is required
prior to considering any intervention.
The MRI in this question reveals an anterior horn meniscal tear on MRI. Given the patients lack
of mechanical symptoms and normal examination, a period of observation is indicated.
References:
1. Shepard MF et al: The clinical significance of anterior horn meniscal tears diagnosed on
MR scans. Am J Sports Med 2002; 30:189-92.
2. Shankman S et al: Anterior horn of the lateral meniscus: another potential pitfall in MR
imaging of the knee. Radiology 1997; 204:181-84.
ANSWER: 5

141. Figrues 44a and 44b show the radiograph and bone scan of a 70 year old man who has right
sided pelvic pain. What is the most likely diagnosis?
12345-

ANSWER

hemangioma
senile osteoporosis
multiple myeloma
metastatic prostate cancer
pagets disease

Kaplan FS; Pagets disease of bone: pathophyisiology diagnosis and management. JAAOS 1995;
3; 336-244.

142. Figure 45 shows the radiograph of a patient with rheumatoid arthritis who sustains a
fracture as a result of a fall. Which of the following factors is the most likely cause of the
fracture?
1. Anterior femoral notching
2. Stress riser from total hip arthroplasty
3. Osteoporosis
4. Limb malalignment
5. Osteolysis
This is a confusing question. The figure shows an AP radiograph of a patient with bilateral total
knee replacements and a right total hip. The patient appears to be extremely osteopenic, but hard
to say for sure based on the great quality of the radiographs we are usually given. No clear
fracture can be seen other than possible pubic rami fractures. The reference given is a study on
The consequences of anterior femoral notching in total knee arthroplasty. The prevalence of
ipsilateral supracondylar femoral fracture in patients with total knee replacement has been cited
to be 0.3 to 4.2%. The most commonly suggested predisposing factors are osteopenia, revision
arthroplasty, rheumatoid arthritis, use of steroids, neurologic disorder, malalignment of the
components, and notching of the anterior femoral cortex. The prevalence of notching of the
anterior femoral cortex has been between 3.5 to 26.9% in various series. From the paper by Lesh
et al., of the periprosthetic femoral fractures reported in the literature, 50 (30.5%) of 164 of them
were associated with intraoperative notching. However, the majority of these patients had other
risk factors. A 3 mm deep notch leads to a 29.2% reduction in torsional strength. Looking at
periprosthetic femur fractures after TKA, however, numerous authors have shown that there are
more knees without notching than there are knees with notching. Authors have postulated that
this reduction in strength is only in the immediate postoperative period and osseous remodeling
does occur, leading to strengthening of the distal femur.

ANSWER

Lesh ML, Schneider DJ, Deol G, Davis B, Jacobs CR, Pellegrini VD JR: The consequences of
anterior femoral notching in total knee arthroplasty: A biomechanical study. J Bone Joint Surg
Am 2000; 82: 1096-1101.

143. A 24 year old hockey player has persistent deep hypothenar palm pain after falling with his
arm extended behind him. Plain radiographs and special radiographic views fail to show any
abnormalities. What study will provide the most cost effective analysis?
12345-

ANSWER

bone scan
ultrasound
CT
electrodiagnostic studies of the median and ulnar nerves
MRI of the wrist

Ploivy KD, et.al., Fractures of the hook of the hamate: A failure of clinical diagnosis. J Hand
Surg 1985; 10:101-104.

144. A 3 year old girl has a left thoracolumbar scoliosis measuring 40 degrees that has
progressed 12 degreees in the past year. Neurologic examination is intact. Radiographs show a
hemivertebra at T12., with a contralateral bar from T11 to L1. Based on these findings, what is
the best course of aciton?
12345-

ANSWER

brace treatement to prevent development of compensatory curves


in situ anterior and posterior fusion
anterior fusion]
posterioir fusion with a growth rods technique
excision of the congenital bar

McMaster MJ, Congenital scoliosis caused by unilateral failure of vertebral segmentation with
contralateral hemivertebra. SPINE 1998; 23; 998-1005.

145. Figures 46a & b show the MRI of a 29 yo man who has shoulder pain and weakness.
Physical therapy has failed to provide relief. History reveals that he injured the shoulder 3 yr ago
when he dove for a volleyball. Examination reveals full shoulder motion, external rotation
weakness, and a postitive impingement sign. In addition to decompression of the cyst, what is
the most appropriate surgical procedure to address this problem?
1.
2.
3.
4.
5.

distal clavicle excision


acromioplasty
Bankart repair
rotator cuff repair
superior labral repair

12345-

he does not have AC symptoms


the MRI slices shown do not show a subacromial spur or underlying tendinopathy
he does not have instability
he does not have a rotator cuff tear in the MRI cuts
he does have a cyst (causing impingement) with an associated abnormal signal in the
superior labrum, both suggestive of a tear. Cyst excision without labral repair would be
pointless

ANSWER

146. What structure exits from the lesser sciatic foramen?


1)
2)
3)
4)
5)

Piriformis
Inferior gluteal nerve
Posterior cutaneous nerve of the thigh
Nerve to the obturator internus
Obturator internus

The greater sciatic foramen is bounded, in front and above, by the posterior border of the hip
bone; behind, by the sacrotuberous ligament; and below, by the sacrospinous ligament. It is
partially filled up, in the recent state, by the Piriformis which leaves the pelvis through it. Above
this muscle, the superior gluteal vessels and nerve emerge from the pelvis; and below it, the
inferior gluteal vessels and nerve, the internal pudendal vessels and nerve, the sciatic and
posterior femoral cutaneous nerves, and the nerves to the Obturator internus and Quadratus
femoris make their exit from the pelvis. The lesser sciatic foramen is bounded, in front, by the
tuberosity of the ischium; above, by the spine of the ischium and sacrospinous ligament; behind,
by the sacrotuberous ligament. It transmits the tendon of the Obturator internus, its nerve, and the
internal pudendal vessels and nerve.
ANSWER

147 Which of the following fracture positions is most likely to result in a poor outcome
following a tibial plateau fracture?
1- 3 mm of condylar widening
2- 3 mm of articular step-off
3- 5 mm of articular gap
4- 5 of varus malalignment
5- 5 of valgus malalignment

You either knew it or you didnt.


Straight from the abstract of the second article:
Clinical function was found to deteriorate rapidly with increasing values of residual
medial tilt of the tibial plateau, whereas lateral tilt of the plateau was well tolerated up to
5 degrees. Articular step-off up to 3 mm and condylar widening up to 5 mm had no
adverse effects.
Radiologic criteria for surgical treatment of tibial plateau fractures (referenced from the second
article)

Fracture of the lateral tibial condyle


o Lateral tilting of the plateau > 5 degrees
o Step-off > 3 mm
o Condylar widening of > 5mm
Fracture of the medial condyle
o All fractures except nondisplaced fissures
Laterally tilted bicondylar fracture
o When medial condyle is undisplaced, see criteria for lateral tibial condyle
o When medial condyle is displaced, operation is always recommended
Medial tilted bicondylar fracture
o All
Axial bicondylar fracture
o All except nondisplaced fissures
So to sum it up. Varus = bad
ANSWER

Lansinger 0, Bergman B, Komer L, Andersson GB: Tibial condylar fractures: A twenty-year


follow-up. J Bone Joint Surg Am 1986;68: 13-19.
Honkonen SE: Indications for surgical treatment of tibial condyle
fractures. Clin Orthop 1994;302: 199-205.

148. Examination of a 6 year old boy with a painless waddling gait reveals equal limb lengths.
Positive mild bilateral Trendelenburg signs, and hip abduction limited to 45 degrees. A
radiograph is shown in Figure 47, and a skeletal survey shows no other abnormalities.
Management should consist of
12345-

ANSWER

observation with repeat radiographs in 6 months


physical therapy for abductor strengthening
distal transfer of both greater trochanters
percutaneous pinning of both proximal femoral physes
bilateral hip valgus osteotomies

Carroll k, et.al, Coxa vara: surgical outcomes of valgus osteotomies. JPO 1997; 17; 220-224.

149.Figure 48 shows the radiograph of a 10 yo girl who has an asymptomatic humeral mass.
What is the best course of action?
1. Open biopsy
2. Needle (core) biopsy
3. Repeat radiograph in 3 months
4. MRI
5. CT of the chest
---------------------------------------------------------The referenced texts were missing/not in our library. Ortho Clin North Am 1996; 27: 583-589 & ICL 1987; 36: 87114.
Spoke with the Man himself and Getty wasnt certain about the diagnosis and thought this might have been
a two picture question with the orthogonal view missing. THUS, his best guess is that this is a juxtacortical
enchondroma. Below are two sources and an excerpt from an article. (this patient however is female and only 10,
and the mass is asymptomatic which goes against this diagnosis in any case)
* Periosteal Enchondroma (also called juxtacortical enchondroma), a cartilaginous tumour which originates
below the periosteal connective tissue and produces a characteristic radiological appearance with local cortical
thickening and a saucer-like depression at the apex representing the radiolucent chondroma which occasionally
contains foci of calcification. Clinical presentation is with a slow growing painful lump over a tubular bone which
may continue to grow beyond skeletal maturity.
* Atypical enchondroma (juxtacortical chondroma): an aysmptomatic - well circumscribed calcified lesion
which has not changed in size is most consistent w/ an enchondroma; more peripherally located lesions are also
more likely to represent an enchondroma;
Periosteal chondroma (also named juxtacortical chondroma, eccentric chondroma) is a relativity
uncommon, slowly growing, benign cartilaginous tumor that represents less than 0.65% of bone tumors. It grows
eccentrically, affecting the cortical bone, beneath or within the periosteum. It is widely accepted that Lichtenstein
and Hall first described this lesion as a distinct entity, although other cases, named subperiosteal epiphysial
chondroma, eccentric chondroma, and periosteal chondroma, were previously reported. To include those tumors that
arise in zones lacking periosteum, such as the femoral neck, Jaffe proposed the term juxtacortical chondroma.
Periosteal chondroma is more common in male than female individuals and more common during the second and
third decades of life; it affects the metaphysis or the region around the diaphysis of tubular bones, especially the
proximal humerus, femur, and phalanges, with pain and swelling.
The radiologic appearance of periosteal chondroma is usually typical. The tumor is a well-defined small
lesion, between 2 and 4 cm in diameter, although up to 35% of tumors may be poorly circumscribed. It develops in a
saucerlike eroded area of the cortex, with a band of reactive sclerosis and a buttress of periosteal new bone. The
lesion is demarcated from the medullary cavity by the intervening cortex of the bone. The cartilaginous matrix may
show foci of calcification as dots or ringlike formations. Other peripheral cartilaginous entities included in the
differential diagnosis are enchondroma protuberans and soft tissue chondroma. In rare cases, an enchondroma
causes an eccentric expansion with bulging of the cortex and produces the picture of enchondroma protuberans.
However, this entity lacks nuclear atypia and shows isolated islands of cartilage surrounded by plates of lamellar
bone, a histologic pattern indicative of benign enchondromatous tissue. Soft tissue chondroma may show cytologic
atypia and rarely invasion of the bone. Consequently, differentiation between this lesion and chondroma might be
very difficult. However, the discrimination has no relevant clinical significance. Other lesions that may involve the
periosteum are osteochondroma, aneurysmal bone cyst, subperiosteal hemangioma, subperiosteal ganglion, and
pigmented villonodular synovitis. The first may have a radiologic appearance similar to that of periosteal
chondroma. However, in a typical osteochondroma, the cortex of the bone fuses with the cortex of the lesion, and
there is communication between the medullary cavity of the host bone and the tumor. The other lesions mentioned
have distinct microscopic features sufficient to allow their specific recognition.
The treatment of choice of periosteal chondroma is marginal excision. Recurrence is uncommon and
usually due to incomplete resection. All authors agree that periosteal chondroma must be correctly diagnosed to
avoid overtreatment prompted by its worrisome histologic characteristics. We recommend excellent communication
among pathologists, radiologists, and surgeons because isolated histologic findings are not sufficient for an accurate
diagnosis.

ANSWER

150. Phantom limb pain has been reported in what percentage of adult lower extremity
amputees?
1- 0% to 10%
2- 20% to 30%
3- 40% to 50%
4- 60% to 70%
5- 80% to 90%

ANSWER

60% to 70% of adult lower extremity amputees experience phantom limb pain. Residual limb
pain and back pain are also common after amputation.
Smith DG, Ehde DM, Legro MW, Reiber GE, del Aguila M, Boone DA: Phantom limb, residual
limb, and back pain after lower extremity amputations. Clin Orthop 1999;361 :29-38.
Koval KJ (ed): Orthopaedic Knowledge Update 7. Rosemont, IL, American Academy of
Orthopaedic Surgeons, 2002, pp 127-137.

151.

Figure 49 shows the AP radiograph of a 50-year-old patient with rheumatoid arthritis


who has hip pain. History reveals that the patient underwent total hip arthroplasty 6 years
ago but has had poor function for the past year. Laboratory studies for infection are
negative. Reconstruction of the acetabulum is best managed with
12345-

a bipolar head with morcelized bone graft.


an antiprotrusio cage with morcelized bone graft.
a cementless acetabular cup with structural bone graft.
a cemented acetabular cup with structural bone graft.
a cemented acetabular cup with the defect filled with cement.

Radiographs show cavitary lysis (Type II) around the acetabular cup (rather than segmental bone
loss Type I), which will lead to a protrusio deformity once the cysts are cleaned out during
surgical revision. The treatment of such acetabular deformity is mildly controversial.
The bipolar head can be eliminated as an option it is only used in revision situations for gross
instability. Cemented acetabular components have always shown increased rates of loosening, so
choices 4 and 5 can be eliminated. Although a cementless cup is commonly used in the
acetabulum, the host bone must be live and incoporate into it. Choice 3 pairs a cementless cup
with structural bone graft which would be dead bone. The cup would never integrate and would
loosen rapidly. The best choic is an antiprotrusio cage with morcelized bone graft.
Haddad, et.al., operated on 48 revision hips with cavitary bone loss and a protrusio deformity.
46 of 48 healed appropriately after filling the cavity with morcelized bone (some will
occasionally use cement), placing the cage into the acetabulum, using screws to fix the cage into
the host bone, and cementing a polyethylene cup into the cage.
ANSWER

Antiprotrusio cage with morcelized bone graft.

Haddad FS, et.al.; Acetabular reconstruction with morcellized allograft and ring support: A
medium term review. J Arthroplasty 1999; 14: 788-795.
Markovich GD: Acetabular reconstruction in revision total hip arthroplasty: A review of options.
Am J Orthop 1998; 27: 662-670.

152. A 26 year old patient sustains the closed isolated injury shown in Figure 50.
management should consist of
12345-

ANSWER

closed reduction and casting


open reduction internal fixation using Rush rods
open reduction internal fixation using rush rods and bone grafting
open reduction internal fixation using dynamic compression plates
open reduction internal fixation using dynamic compression plates and bone
grafting

Wright RR, et.al., The necessity of acute bone grafting in diaphyseal forearm fractures: A
retrospective review. JOT 1997; 11; 288-294.
OKU 7 p 3-7-316.

153. A 19-year-old boy has had right lower leg discomfort for the past 4 months and a recent
fever. Laboratory studies reveal a WBC count of 10,000/mm3 and an erythrocyte
sedimentation rate of 31 mm/hr. A radiograph and MRI scan are shown in figures 51a and
51b. A biopsy specimen is shown in Figure 51c. What laboratory finding is most likely
associated with this disease entity?

21- Acid fast bacilli


22- Gram positive cocci
23- Translocation (9;22)
24- Translocation (11;22)
25- Mutation of the Rb gene

The scenario that is being described, and the answers given, give two possible general diagnoses:
infect vs tumor. The images provided rule out infection, as neither AFB or GPC are in the path.
Thus, you must know what the path looks like for the remaining three choices. A translocation at
9;22 is found in approximately 25% of myxoid chondrosarcomas. The Rb gene is a tumor
suppressor gene, mutation of which has been found in 50-60% of Osteosarcomas. A
translocation at 11;22 is found in 85% of Ewings sarcomas.
The histology of chondrosarcoma: mesenchymal cells producing cartilaginous matrix,
often with little cellular atypia; osteosarcoma are distinguished by the production of osteoid
bony islands hugged by a rim of malignant osteoblasts; and Ewings sarcoma, a primative,
small, round cells with scant cytoplasm.
So, Ewings Sarcoma is the answer of choice. It is the 2nd (1st=osteosarcoma) most
common malignancy of childhood (80% occur before age 20, peak preavalence 10-20y.) It
comprises 10% of primary bone tumors, males at greater risk than females, Lower extremities
and pelvic girdle are most common locations, but can be anywhere. Presents with pain, swelling,
elevated ESR, fever, anemia, leukocytosis. Xray appearing is mottled with periosteal reaction,
often see onion-skinnig, frequently associated with large soft tissue mass.
ANSWER

154. Figure 52 shows the anteroposterior axis of the distal femur (vertical line) and the epicondylar axis
(horizontal line). How is the epicondylar axis oriented in relationship to the posterior condylar line (line
between the most posterior aspect of the posterior femoral condyles)?
1.
2.
3.
4.
5.

Parallel
Approximately 3 degrees internally rotated
Approximately 3 degrees externally rotated
Approximately 6 degrees internally rotated
Approximately 6 degrees externally rotated

The epicondyles of the distal femur are important anatomic landmarks for proper rotational alignment of
the femoral component in TKA. Biomechanical analysis reveals that the transepicondylar axis is essentially
parallel to the primary center of rotation of the knee. The attachment of the collaterals makes them the logical
reference for rotation of the femoral component. The posterior condylar angle is defined by the angle between
the surgical epicondylar axis and the tangent to the posterior condyles. In the anatomic study referenced, the
posterior condylar angle averaged 3.11 degrees of external rotation. When Dr. Goldberg completes his total
knees, he asks us to draw these lines in the femur. We then are asked to place the provisional anterior cutting
guide in 3 degrees of external rotation in reference to the epicondylar axis. This reproduces the posterior
condylar angle and also assist w/ patellar tracking.
ANSWER

3.

Approximately 3 degrees externally rotated

References:
Griffin: Anatomy of the epicondyles of the distal femur: MRI analysis of normal knees. J Arthroplasty 2000;
15: 354-9.

155. A 35 yo woman sustained a lateral subtalar dislocation of the left foot in a motor vehicle
accident. Initial attempts at closed reduction under sedation are unsuccessful. The inability to
reduce the joint is most likely the result of
1.
2.
3.
4.
5.

the lack of adequate anesthesia


interposition of the extensor digitorum brevis
entrapment of the posterior tibial tendon
impingement of the talar head on the calcaneus
fracture of the sustentaculum tali

20% of lateral subtalar dislocations cannot be reduced closed. Most commonly, the tibialis
posterior tendon is interposed around the talar neck. The FDL tendon can also prevent reduction.
Hence, a medial incision is used to free the TP tendon and reduce the dislocation.
Medial subtalar dislocations are irreducible 10% of the time because of the extensor brevis,
peroneals, talar head button-holing through the retinaculum or interposed neurovascular bundle.
Skeletal Trauma
ANSWER

156. A 12-month-old boy has refused to move his right lower extremity for the past 24 hours;
however, his parents deny any history of trauma. He has a rectal temperature of 102.2F
(39C). Examination reveals marked guarding during attempted passive motion of the
right hip; however, motion of the lumbar spine and right knee is tolerated. Laboratory
studies show a peripheral WBC count of 15,500/mm3 (normal 3,500 to 10,500 mm3) and an
erythrocyte sedimentation rate of 60 mm/hr (normal up to 20 mm/hr). A pelvic radiograph
is normal. Management should now consist of
12345-

radiographs of the spine.


radiographs of both lower extremities.
blood cultures.
needle aspiration of the right hip.
pelvic MRI.

Sucato DJ, Schwend RM, Gillespie R: Septic arthritis of the hip in children. J Am Acad
Orthop Surg 1997;5:249-260.
Kocher MS, Zurakowski D, Kasser JR: Differentiating
between septic arthritis and transient synovitis of the hip in children: An evidence-based clinical
prediction algorigthm. J Bone Joint Surg Am 1999;81:1662-1670.
A child who has an acutely irritable hip can pose a diagnostic challenge. In the second
reference above, Kocher and his colleagues attempted to determine the diagnostic value of
presenting variables for differentiating between septic arthritis and transient synovitis and to
develop a prediction algorithm. Here are their 4 variables: fever (38.5 orally during the week
prior to admission), non-weight bearing, ESR > 40, WBC > 12.

In our vignette, the boy may not yet be of age to walk and thus non-weight bearing is
assumed given his irritability. Based on the above algorithm, there is a 99.8% predicted
probability of septic arthritis. The first step is to aspirate the hip. In the first reference above,
Sucato and his colleagues recommend operative intervention in an expeditious manner in all
cases of a septic hip regardless of patient age or causative organism. If purulent fluid is obtained
by aspiration, they generally perform an arthrotomy immediately without waiting for laboratory
fluid analysis.
ANSWER
4

157. What structure is the primary static restraint to inferior glenohumeral translation with
the arm in 0 degrees abduction?
1. anterior band of the inferior glenohumeral ligament
2. posterior band of the inferior glenohumeral ligament
3. middle glenohumeral ligament
4. superior glenohumeral ligament
5. superior labral complex

The shoulder is stabilized by both static and dynamic restraints. Static restraints include the
articular anatomy, the labrum, negative intra-articular pressure, and the glenohumeral ligaments.
Dynamic restraints include joint compression, barrier effect, and steering effect of the rotator
cuff muscles. The glenohumeral ligaments are discrete capsular thickenings that limit excessive
rotation and translation of the humeral head.
The superior glenohumeral ligament works with the coracohumeral ligament to prevent inferior
instability in the adducted arm. The inferior glenohumeral ligament is composed of an anterior
and posterior band with an interposed axillary pouch. This complex is a major anterior stabilizer
of the glenohumeral joint, especially with the arm abducted and externally rotated. The middle
glenohumeral ligament, which runs obliquely over the subscapularis, functions principally to
prevent anterior instability when the shoulder is externally rotated and abducted 45 degrees. The
labrum is a fibrocartilaginous thickening surrounding the glenoid that deepens the glenoid cavity
preventing abnormal motion. It also serves to anchor the inferior glenohumeral ligament
complex.
ANSWER

158. A 23-year-old woman who sustained a femoral shaft fracture in a motor vehicle accident is
treated the night of injury with reamed, statically locked intramedullary nailing. An
intraoperative radiograph is shown in figure 53. Management should now consist of
1.
2.
3.
4.
5.

touchdown weight bearing for 6 weeks


weight bearing as tolerated
open reduction and stabilization of the femoral neck fracture
conversion to a cephalomedullary (reconstruction) nail with piriformis entry
conversion to a cephalomedullary nail with trochanteric entry

In their paper Treatment of Complex Fractures: Complicated Femoral Shaft Fractures,


Ortho. Clinics of N. America. 33:(1). Jan 2002, Russell et al. review the current treatments of
femoral shaft fractures with ipsilateral femoral neck fractures. In this paper, they quote the
incidence of femoral neck fracture in addition to femoral shaft fracture at 2%-6%. Frequently, as
may have been the case in this question, the femoral neck fracture is not initially diagnosedup
to 33% of the time. For this reason, they suggest that comminuted, midshaft femoral fracture
secondary to axial loading should raise the suspicion for possible femoral neck fracture.
They also note that several investigators found femoral neck fractures occurring after the
antegrade insertion of a nail for a femoral neck fracture. The authors recommend, therefore,
intraoperative radiography of the femoral neck in 15 degress of internal rotation after insertion of
the nail.
As for treatment options, there are several options. If the femoral neck fracture is visible
initially, it can be stabilized first, followed by fixation of the femoral shaft with retrograde
intramedullary nailing or dynamic compression plating (as described by Swiontkwski et al.,
JBJS, 66:260-268, 1984, 129-136). Alternatively, antegrade nailing of the femoral shaft fracture
can be followed by screw fixation of the femoral neck fracture as described by Wu and Shih,
Acta Orthop Scand 62:346-351, 1991. Finally, a cephalomedullary nail can be used as well. As
with the antegrade nail, the disadvantages of potential compromise of the femoral head blood
supply also apply.
Whatever, method is employed, the bottom line is that the femoral neck fracture must be
fixed. In a young, healthy woman such as in this question, Swontkowski (JBJS,76-A, (1), Jan
1994) argues that multiple pins or screws is the best method of choice. Patients older than 75
should have prosthetic replacement of some type. In patients with low functional demands,
chronic illness, or poor bone density, bipolar replacement or THA should be considered. Patients
of any age who have chronic illness or reduce life expectancy should be managed with a unipolar
replacement. Whatever the case may be, the evidence is clear that in a medically stable patient
(and especially the elderly), the chance of post-operative morbidity is substantial reduced is the
operation is carried out within the first 24 hours of the injury.
ANSWER

159. Figures 54a and 54b show the radiograph and MRI scan of an 18-year-old woman who has
knee pain that awakens her from sleep. A chest CT scan is negative for metastatic disease. A
bone scan is shown in Figure 54c, and a biopsy specimen is shown in Figure 54d. What is the
Musculoskeletal Tumor Society stage of her disease?
1) Ia
2) Ib
3) IIa
4) IIb
5) III

The lesion shown is high-grade intramedullary osteosarcoma, or classic osteosarcoma. It most


commonly occurs about the knee in children and yound adults. Other common sites include the
proximal humerus, proximal femur, and pelvis. Patients present primarily with pain. More than
90% of intramedullary osteosarcomas are high grade. Because this lesion penetrates the matrix,
it is considered extracompartmental. High grade, extracompartmental lesions are classified as
IIb.
ANSWER

160: Most medical errors result primarily from:


1.) negligence
2.) malice
3.) system failure
4.) inexperience
5.) physician fatigue

Introduction.--Error can be defined as either an unintentional act of omission or commission or


an act that has an unintended outcome. Most studies of errors in medicine have focused on
iatrogenic injuries, and reports of treatment-related iatrogenic injury are disturbingly high. Yet,
most errors do not result in injury. Other sources of medical error include missed diagnoses
resulting in death and operational errors, such as delayed treatment or the failure to get follow-up
laboratory studies. It has been estimated that hospitals have a 1% error rate, which would not be
tolerated in industry.
Reasons for the High Error Rate in Medicine.--There may be a lack of awareness of the
problem's severity because errors are not widely reported and they usually do not result in harm.
However, most importantly, the culture of medical practice strongly emphasizes infallibility,
which discourages admission of mistakes and shared learning from mistakes. The malpractice
threat further discourages disclosure or investigation of medical errors.
Theories of Error.--Errors can be classified by the level of cognitive activity involved. Slips are
skill-based errors that occur when there is a failure in automatic cognitive activity, caused by a
break in routine when attention is diverted. Mistakes are rule-based errors that occur when the
wrong stored rule is applied or knowledge-based errors that occur when a lack of knowledge or
misinterpretation of the problem results in faulty problem-solving in a novel situation. Mistakes
may be caused by biased memory, premature decision-making, confirmation bias,
overconfidence, and stress. Human factor research has focused on interactions between humans
and equipment and emphasizes the multiple and additive sources of errors, some of which may
be latent. With human factor theory, error may be thought of as inevitable, with preventive
efforts focusing on simplification, constraints, standardization, and reversibility.
Prevention of Medical Errors.--Data on errors should be routinely collected and investigated,
with an understanding that error is inevitable but manageable. Mechanisms to reduce the
likelihood of error include designing checklists and protocols that reduce the reliance on
memory, improving information access, structuring critical tasks to minimize error (e.g.,
computerizing medication orders with programs to prevent lethal overdose or prescriptions that
cause known allergic reactions in a patient), standardizing processes, and maintaining ongoing
training in safe practice.

ANSWER

161. On the axial T1-weighted MRI scan shown in Figure 55, taken through the L4-L5 disk
space, the structure at the tip of the arrow represents
1 the dorsal root ganglion
2 the vertebral artery
3 a facet joint cyst
4 a far lateral disk herniation
5 a central disk herniation

straight forward anatomy question. Note that the signal is identical to that of spinal canal and
is noted to be symmetric (it is on both sides) without signals consistent with fluid or
inflammation.
ANSWER

162. A 1-year old boy has a 2 cm limb-length discrepancy and asymmetrical limb circumference.
Management should consist of:
1.
2.
3.
4.
5.

Abdominal Ultrasound and serum AFP


skeletal survey
skeletal survey and a PA radiograph to determine bone age
MRI of lower extremities
Collagen proppeptide scan

Answer: 1: Abdominal ultrasound and serum AFP. I think this response refers to the mother
during the next pregnancy. There are asymmetric deformities and elevated AFP in amniotic
band syndrome (ABS). The answers were not in the references.
ANSWER

163. For complete amputations through the midforearm, replantation usually is not
recommended if the warm ischemia time is more than how many hours?
12345-

4
6
8
12
24

The duration of ischemia that tissue will tolerate is dependent on the amount of muscle present.
Muscle has a much higher metabolic requirement and therefore is more sensitive to ischemia.
Keeping the amputated part hypothermic, lowers the metabolic rate and lengthens the window
for replantation. Bone, tendon and skin can tolerate approximately 8-12 hours of warm ischemia
and as long as 24 hours of cold ischemia. Muscle on the other hand, necroses after 6 hours of
warm ischemia or 12 hours of cold ischemia. In general, replantation is contraindicated if the
warm ischemia time is greater than 6 hours and 8-10 hours if the amputated part is kept
hypothermic. If the amputated part is replanted after the acceptable time period it will be
susceptible to reperfusion injury.
There are two accepted methods of preserving an amputated part. One way is to place the
amputated part in a saline filled plastic bag that is then placed in a larger bag filled with crushed
ice. The other acceptable method is to wrap the part in moist gauze and then place it in a bag
filled with ice slurry. The goal of both methods is to maintain the part at 4C. Dry ice should
not be used.
ANSWER

References:
Axelrod TS, Buchler U: Severe complex injuries to the upper extremity: Revascularization and
replantation. J Hand Surg AM 1991;16:574-584.

164. Unreamed nails are associated with an increased occurrence of what complication when
compared with reamed nails in the treatment of distal femoral shaft fractures?
12345-

Delayed union
Infection
Pulmonary dysfunction
Compartment syndrome
Shortening

Reamed nailing remains the treatment of choice for femoral shaft fracture because of (1) higher
union rate lower (2) a lower need for secondary procedures to obtain union (3) the absence of
detrimental pulmonary effects when reamed nails are used.
Clinical studies reviewing nonreamed retrograde femoral nails have revealed a rate of union of 85% to
95% after the initial procedure. The diameter of the nail must be matched to the diameter of the canal and
not undersized. Nails should be dynamized at 6 to 12 weeks postoperatively if adequate callus is not seen,
to maximize the chances of fracture union. Knee range of motion in patients without associated injuries
averaged 130, and the use of continuous passive motion during the postoperative period did not increase
the ultimate knee range of motion. There have been no intra-articular infections in patients with open
fractures, and no hardware failures. Alignment was satisfactory in all patients. Early nail dynamization
and weight bearing are important to minimize the chance of nonunion when a nonreamed nail is used.
A review of the literature concluded that reamed nailing remains the treatment of choice for femoral shaft
fractures because of (1) a higher union rate; (2) a lower need for secondary procedures to obtain union;
and (3) the absence of detrimental pulmonary effects when reamed nails are used. For fractures treated
with this method, the rates of union are at least 95%, rate of infection less than 1%, and rates of malunion
are low.
For adequately resuscitated patients, reamed intramedullary nailing remains the treatment of choice.
Coagulopathies and hypothermia must be corrected before surgery. Objective evidence of resolving shock
such as a normalizing base deficit or lactate must be present. Patients with severe pulmonary injuries who
cannot be adequately oxygenated as well as hemodynamically unstable patients who remain hypoperfused
should be treated with alternative means such as skeletal traction, a plate, or an external fixator.
Stabilization of the femoral shaft within the first 24 hours of injury in multiply injured patients (injury
severity score [ISS] > 18) has decreased the incidence of pulmonary insufficiency and adult respiratory
distress syndrome (ARDS), and decreased the length of stay and hospitalization costs for patients with
less severe associated injuries.

ANSWER

OKU 7. Rosemont IL, AAOS , 2002 pp 453-463


Clatworthy MG, et al: Reamed vs
unreamed femoral nails: A randomised, prospective trial. JBJS Br 1998; 80; 485-489
Tornetta P III, et al: Reamed vv nonreamed antegrade femoral nailing. JOT 2000; 14:15-19

165. An 18yo soccer player has acute thigh pain after the game. Initial radiographs are normal.
Sagittal MRI scans are shown in Figure 56a and 56b. What is the most likely cause of the rectus
femoris signal abnormality
1.
2.
3.
4.
5.

Myositis Ossificans
Soft-Tissue Sarcoma
Hemangioma
Muscle Contusion
Muscle Rupture

The MRI demonstrates increased signal in the rectus femoris muscle and discontinuity of
fibers. There is a history of trauma (after a game) make 2 and 3 unlikely and these are also not
consistent with the MRI images. Myositis ossificans would demonstrate some changes on the
plain films and also would not develop so acutely. Muscle contusion would result in increase
edema but not fiber discontinuity.

ANSWER

Weishaupt D et al Injuries to the distal gastroc muscle. MR findings JCAT 2001 25 677-82.

166. A 28-year-old woman undergoes ACL reconstruction using a quadruple-looped ST-Ghamstring autograft. During the first 6 weeks of therapy, the patient should refrain from:
1. prone hangs
2. passive extension
3. passive and active assisted ROM
4. isokinetic quadriceps strengthening
5. patellar mobilization

The 2 most commonly used autografts for ACL reconstruction are bone-patellar-bone and
hamstring. Potential morbidity with bone-patellar-bone grafts including PF pain, patellar
fracture, and loss of motion have prompted increased use and investigation with hamstring
autografts. Williams et al recently reported good results with use of quadruple loop hamstring
grafts.
The rehab protocol following ACL reconstruction allows prone hangs, passive extension ROM,
passive and active assisted knee flexion, patellar mobilization. amd proprioceptive training
during the first 2-3 weeks after surgery. During the second 2-3 weeks, ROM and weight-bearing
are progressed. Leg squats, wall sits and leg press exercises are started at this point. At
approximately the 6 week mark, isokinetic quadriceps and hamstring training are initiated and
progressively increased. Return to sports typically occurs at approximately 5-6 months.
In this question, choices 1,2,3, and 5 are permitted in the initial post-operative period. Only
isokinetic strengthening (choice 4) is delayed until approximately 6 weeks after surgery.

Phillips BB: Arthroscopy of the lower extremity, in Campbells Operative Orthopaedics


1998, 1470-1561.
Arendt EA: OKU: Sports Medicine 2. AAOS 1999, 307-316.
Wiliams RJ et al: Anterior cruciate ligament reconstruction with a 4-strand hamstring
tendon autograft. JBJS 2004; 86: 225-32.
Howell SM et al: Brace-free rehabilitation, with early return to activity, for knees
reconstructed with a double-looped semitendinosis and gracilis graft. JBJS 1996; 78: 81425.
Campbells Operative Orthopaedics. Volume 3. 10th edition. 2003.

ANSWER: 4

167. Figure 57 shows the radiograph of a patient who reports the sudden onset of hip pain.
What is the primary cause of the failure?
12345-

ANSWER

excessive range of motion


implant manufacturing method
patient weight
patient activity
acetabular liner dislocation

Willmann G, Ceramic femoral head retrieval data. CLin Orthop 2000; 379; 22-28.

168. Sterilization by irradiation significantly decreases the structural and mechanical properties
of an allograft at what dosage?
1. 1 Mrad
2. 2 Mrad
3. 3 Mrad
4. 4 Mrad
5. 5 Mrad

Based on proper test-taking skills, the answer would have to be 3 Mrad, since that is the middle
answer!!!!
Allograft usage is popular for multiple ligament reconstructions as there is a limitless supply of
them and there is always potential donor site morbidity associated with autograft harvest.
Allografts are usually preserved by deep-freezing or freeze-drying. Deep-freezing without drying
has little or no effect on the mechanical properties of ligament. Tissue is typically sterilized using
cobalt irradiation, because ethylene oxide gas sterilization is determinental to its biomechanical
properties. A decreasing trend in the structural and mechanical properties is significant at the 3Mrad level of dosage.
ANSWER

:3

Buckwalter JA, Einhorn TA, Simon SR (eds): Orthopaedic Basic Science: Biology and
Biomechanics of the Musculoskeletal System. Rosemont, IL, American Academy of Orthopaedic
Surgeons, 2000, pp 582-616.
Gibbons MJ, Butler DL, Grood ES, et al: Effects of gamma irradiation on the initial mechanical
and material properties of goat bone patellar tendon-bone allografts. J Orthop Res 1991; 9: 20918.

169. A 37 year old man has neck pain and stiffness following a motor vehicle accident 1 week
ago. Examination reveals decreased active range of motion in all planes of the cervical spine,
weakness of grip strength, and a bilaterl Hoffmanns sign. A lateral radiograph is shown in
Figure 58. Further evaluation should include
12345-

ANSWER

electromyogrpahy and neve conduction studies of the upper extremity


MRI of the lumbar spine
MRI of the cervical spine
flexion extension radiographs
a bone scan

Truumees E, et.al., Cervical spondylitic myelopathy and radiculopathy, ICL 2000; 49; 339-360.

170. Figures 59a through 59d show the current radiographs and biopsy specimens of a 56 year
old woman who has increasing pain 1 year after undergoing curettage and stabilization of a distal
femoral bone lesion. Chest Ct and bone scans do not show lesions elsewhere. Management
should now consist of
12345-

ANSWER

chemotherapy
radiation therapy
wide excision of the distal femur
marginal excision of the posterior exophytic mass
irrigation and debridement

Campannacci, et.al. Central chondrosarcomas in Boen and Soft Tissue Tumors; 1999, 283-318.

171. Slip progression in isthmic spondylolisthesis occurs in


1. 5% of patients with spondylolisthesis, is more common in girls, and is rare after skeletal
maturity
2. 5% of patients with spondylolisthesis, is more common in boys, and is rare after skeletal
maturity
3. 5% of patients with spondylolisthesis, is more common in girls, and is common after
skeletal maturity
4. 25% of patients with spondylolisthesis, is more common in boys, and is common after
skeletal maturity
5. 25% of patients with spondylolisthesis, is more common in girls, and is rare after skeletal
maturity

Slip progression in isthmic spondylolisthesis occurs in 5% of patients with spondylolisthesis, is


more common in girls, and is rare after skeletal maturity.
ANSWER

Frederickson, et al. The Natural History of Spondylolysis and Spondylolisthesis. JBJS Am 1984;
66: 699-707.

172. Which of the following structures is the primary stabilizer of the ankle joint under
physiologic loading conditions?
1)
2)
3)
4)
5)

Deltoid ligament
Posterior talofibular ligament
Anterior talofibular ligament
Anterior distal tibiofibular ligament
Calcaneofibular ligament

Ref: Ramsey and Hamilton: Changes in tibiotalar area of contact caused by lateral talar shift.
JBJS Am 1976;58: 356-357
This is the study which shows that a lateral displacement of the talus by 1mm decreases contact
area by 42%. So, the ligament under physiologic conditions which is stabilizes the joint in this
plane is the deltoid ligament.
ANSWER

173
When stabilizing an intertrochanteric femoral fracture, what is the most frequent
complication associated with the device shown in Figure 60?
1- Superficial infection
2- Deep infection
3- Hardware failure
4- Lag screw cutout
5- Femoral shaft fracture

You could kinda figure this one out. Deep and superficial infections dont occur that often with
nailings or DHS. Lag screw cut-out is always a concern with DHS or gamma nail constructs.
Aim for dead center, but if you arent dead on, cheat posterior and inferior, as anterior and
superior is associated with greater cut-out. Aim for less than 1cm in the AP and 1 cm in the
lateral planes.
The article compared a DHS style implant (Richards sliding hip screw) to a short gamma nail
construct. Short-type Gamma nail (203 patients) or a Richard's-type sliding hip screw and plate
(197 patients).
The requirement for revision in the Gamma nail group was twelve (6%); for Richard's
group, eight (4%). This was not statistically different (p = 0.29; odds ratio, 1.48 [0.593.7]). A subcapital femoral fracture occurred in the Richard's group. Femoral shaft
fractures occurred with four in the Gamma nail group (2%) and none in the Richard's
group (p = 0.13). Three required revision to another implant. Lag-screw cut-out occurred
in eight patients in the gamma nail group (4%) and four in the Richard's group (2%). This
was not statistically significant (p = 0.37; odds ratio, 2.29 [0.6-9.0]). The development of
other postoperative complications was the same in both groups. There was no difference
between the two groups in terms of early or long-term functional status at one year.
ANSWER

Adams CI, Robinson CM, Court-Brown CM, McQueen MM: Prospective randomized controlled
trial of an intramedullary nail versus dynamic screw and plate for intertrochanteric fractures of
the femur. J Orthop Trauma 2001;15:394-400.

174. Figures 61a and 61b show the radiographs of a 44 year old woman who has ahd great toe
pain for the past 5 years. What is the most likely diagnosis?
12345-

ANSWER

lupus arthritis
rheumatoid arthritis
gouty arthritis
psoriatic arthritis
hallux rigidus

Mann RA, et.al. Hallux rigdus; treatment by cheilectomy JBJS 1988;70; 400-406.

175. Figures 62a and 62b show the radiographs of a 3 yo boy who has a brittle bone disorder.
His parents have been told that anemia, dental anomalies, and deafness may be associated. The
pathophysiology of this condition is failure of
1. Vitamin D absorption
2. Osteoclastic and chondroclastic resorption
3. Collagen cross linking
4. Copper excretion
5. Cysteine production
------------------------------------------------------Our own ARMSTRONG DG et al. Orthopedic Management of Osteopetrosis: Results of a Survey and Review of the
Literature, 1999; 19 : 122-132.
Osteopetrosis was recognized as a distinct entity by a German radiologist, Heinrich Albers-Schonberg, who in
1904 described a 26-year-old man with sclerosis of the skeleton and multiple fractures. Thereafter the condition was
referred to as Albers-Schonberg disease and as "marble bone disease". The term osteopetrosis was coined by
Karshner in 1926. Currently it is understood to be a syndrome in which excessive density of bone occurs as a result
of abnormal function of osteoclasts. Historically distinction has been made between the "infantile malignant" type
inherited by autosomal recessive transmission and the "autosomal dominant" or "benign adult" forms.
Patients with the infantile malignant form of osteopetrosis are usually diagnosed during their first year of life. . .
Radiographs show increased density of the entire skeleton. In severe cases, there is complete absence of the
medullary canal. Lack of remodeling gives rise to metaphyseal widening or a "club-shaped" appearance of the long
bones. "Bone within a bone" phenomena or "endobones" may be seen especially in the pelvis, vertebrae, and hands
and feet. Transverse radiolucent bands may be present within the metaphyseal regions of the long bones. These have
been attributed to periodic variations in the ability to resorb bone. Longitudinal striations also may be seen in the
long bones, perhaps a result of periosteal elevation by trauma or microfractures. The skull becomes progressively
thickened and a "hair-on-end" appearance may be noted. Fractures occur commonly, especially epiphyseal
separations and femur fractures. Diaphyseal and metaphyseal fractures are usually transverse or short oblique
pattern, in keeping with their pathologic nature.
OKU 7 Osteopetrosis (Albers-Schnberg disease or marble bone disease) defines a rare group of bone metabolism
disorders in which there is decreased osteoclastic resorption of bone and cartilage, but normal bone formation. This
condition results in increased bone density and marrow space obliteration. Histologically, the bone consists of cores
of calcified cartilage surrounded by areas of new bone, usually immature woven bone. The disordered architecture
of the bones makes them more fragile and susceptible to fractures. In some forms of the disease, osteoclasts can be
present in large numbers, but are abnormal. They lack their functional ruffled borders and are not found in
Howship's lacunae. In other forms of the disease osteoclasts are decreased in number or even absent.
Both the radiographic appearance and the clinical severity can be variable. Radiographs demonstrate a diffuse
symmetric increase in bone density. Long bone metaphyses are widened with a characteristic pattern of transverse
sclerotic bands alternating with lucent bands. Vertebrae develop sclerotic bands underlying the endplates, resulting
in a "rugger jersey" appearance. At least nine forms of osteopetrosis exist as one of four phenotypes, each varying in
severity. The majority of patients present with the mild, autosomal dominant form also known as the adult (tarda)
form. Patients have a normal lifespan and may be asymptomatic or have mild anemia. Patients may also have a
history of a few fractures. The congenital form (juvenile, infantile, malignant) is an autosomal recessive disorder and
the most severe form of the disease. Patients die during childhood (younger than age 10 years) and present with
severe anemia, thrombocytopenia, hepatosplenomegaly, and a compromised immune system. They can have cranial
and optic nerve palsies. Patients often sustain multiple fractures and are prone to osteomyelitis. Another autosomal
recessive form is characterized by intermediate severity between the adult and infantile forms. An additional rare
form is associated with renal tubular acidosis and a mutation in the carbonic anhydrase II gene that results in lower
than normal levels of this enzyme available for bone resorption. These patients also can have cerebral calcifications
that often result in mental retardation.

ANSWER

176. The lengthening of a muscle during active resistance against an opposing force is classified
as what type of contraction?
12345-

Isotonic
Isokinetic
Isometric
Concentric
Eccentric

Eccentric: Muscle lengthens during contraction. Greatest potential for high muscle tension and
muscle injury. Eg. negative or elbow increasing extension during bicep curl
Concentric: Muscle shortens during contraction. Tension within the muscle is proportional to externally
applied load. Eg. biceps curl.
Isokinetic: Muscle tension is generated as the muscle maximally contracts at a constant velocity over a
full range of motion.
Isotonic: Muscle tension is constant through the range of motion. Muscle length changes through
ROM.
Isometric: Muscle tension is generated but the length of the muscle remains unchanged. Eg. pushing
against immovable object
ANSWER

Koval KJ (ed): Orthopaedic Knowledge Update 7. Rosemont, fL, American Academy of


Orthopaedic Surgeons, 2002, pp 3-18.
Skeletal Muscle: Form and Function. Champaign, IL, Alan J McComas Publisher, Human
Kinetics, 1996.

177.

What type of bone healing occurs at the contact areas when a transverse radial shaft
fracture is directly reduced and stabilized in compression with a plate?
12345-

Haversion remodeling.
Intramembranous ossification.
Endochondral ossification.
Periosteal bone formation.
Secondary callous formation.

Fracture healing can be divided into primary (osteonal) and secondary (callus) healing. Rigid
plate fixation inhibits callus formation. In regions where there is direct cortical contact, primary
healing occurs by direct haversion remodeling. Briefly, osteoclasts resorb bone and create a
channel across the fracture ends (cutting cone), enabling blood vessels and osteoblasts to follow.
This requires anatomic fracture reduction, rigid fixation, and an intact blood supply. Motion at
the fracture site will lead to secondary (callus) healing.
The majority of fractures heal through secondary callus healing which can be split into
intramembranous and endochondral ossification. Intramembranous ossification occurs in the
embryologic formation of flat bones and with distraction osteogenesis. This occurs without a
cartilagenous intermediate. Endochondral ossification occurs both in long bone development
and fracture healing. It is characterized by a cartilage intermediate and includes five stages: (1)
hematoma and inflammation; (2) angiogenesis and cartilage formation; (3) cartilage calcification
and cartilage removal; (4) bone formation; (5) bone remodeling. Do not be confused by the bone
formation and remodeling stages both intramembranous and endochondral bone first form
woven bone, and then are remodeled over several months to form lamellar compact bone.
ANSWER

Haversion remodeling

Simon SR (ed): Orthopaedic Basic Science. 1994, pp. 377-382.


Koval KJ (ed): OKU 7. 2002, pp. 19-29.

178. Based on the sagittal and axial MRI scans shown in Figures 63a and 63b, the patient most
likely has which of the following clinical findings?
1 hip flexor weakness
2 quadriceps and tibialis anterior weakness
3 extensor hallucis longus and hip abduction weakness
4 plantar flexion and eversion weakness
5 cauda equina syndrome

Review of the sagittal and axial MRI scans shows a right sided L4/5 extraforaminal disc
herniation. Based on the anatomic pathway of the L4 and L5 nerves one would expect the L4
nerve to be impinged upon by this disc herniation. This is clearly evident on the axial view. Once
you have determined what nerve root is being impinged you can then predict what symptoms this
patient may experience.

Muscle Group

Nerve

Hip flexors
Hip extension (gluteus maximus)
Hip adduction
Hip abduction (gluteus medius)
Quadriceps
Tibialis anterior
EHL
Plantar flexion and eversion

Branches
Inferior gluteal
Obturator
Superior gluteal
Femoral
Deep Peroneal
Deep Peroneal
Tibial

Nerve Root
[T12] L1, L2, L3
S1
L2, L3, L4
L5
L2,L3,L4
L4
L5
S1

Cauda equina syndrome is caused by compression of the roots of the cauda equina. This
would only be caused by a very large central disc or other space occupying lesion. L4 and L5
discs are the most common offending herniations. Obviously based on the images this is not
present. If it were present this patient could be expected to have Pain: back of thighs and legs
Numbness: perianal and loss of anal reflex or bulbocavernosus reflex. Paralysis: bowel and
bladder incontinence.
Since the affected nerve root is L4, this patient could be expected to have weakness of the
quadriceps and tibialis anterior.
ANSWER

Physical Examination of the Spine and Extremities. Hoppenfeld, S. 1976. pp. 237-263.
OKU Spine 2. AAOS pp 323-332

179.

When converting a hip arthrodesis to a total hip arthroplasty, which of the


following factors is most predictive of improved walking ability?

26- Length of time from arthrodesis to hip arthroplasty.


27- Limb-length discrepancy
28- Position of the arthrodesis
29- Abductor function
30- History of infection

This question is based on an article from Paris, France, written by Hamadouche, Kerboull L,
Meunier, Courpied and Kerboull, M. in JBJS 83A 7: 992-998. Basically they reviewed their 45
THAs (1969-1993) which were conversions from hip arthrodesis (spontaneous or surgical) and
for which they had a mean of 8.5 years follow-up. They found the only factor that was
predictive of the final functional result with regard to walking ability was the interoperative
status of the gluteal muscles (abductors). They classified the abductors on their appearanca
intraoperatively: type 1 (poor color, thin but contiguous); type 2 (fair appearance, pink and quite
thin) and type 3 (satisfactory, red, quite thin). Type 3s did the best, and type 1d might do okay.
They suggest that is contraction of the gluteals cannot be felt preoperatively, another treatment
may be better. If the gluteals are not in continuity, they do not perform the conversion to THA,
they simply fuse to an acceptable position. Great question.
ANSWER

180. A 45 y/o man w/ a hx of smoking two packs of cigarettes a day for 20 years undergoes an arthroscopic
Bankart repair under an interscalene block and general endotracheal anesthesia. There are no intraoperative
complications. Four hours after surgery, he remains in the recovery room where he continues to require
supplemental oxygenation to maintain an oxygen saturation level above 90%. The patient has bilateral breath
sounds and a normal EKG. What is the next most appropriate step in management?
1.
2.
3.
4.
5.

Observation
Discharge
Ventilation-perfusion scan
Central venous line insertion
Chest radiograph

Scalene regional anesthesia is widely used, effective and reasonably safe means of anesthesia for pts
undergoing shoulder procedures. Some studies have suggested that this technique is superior to general
anesthesia. However, the failure rate has been noted to be from 3-18% and seizure rates from 0-6%.
Temporary and even permanent neurologic injury have also been reported. While administering the block w/
the pt awake, use of blunt needles and nerve stimulators have been proposed, these techniques did not prevent
nerve injury in the referenced paper. There is also a high rate of phrenic nerve paralysis (2%) with resultant
pulmonary compromise. The main complaint is shortness of breath.
A patient who is unable to maintain his oxygen saturation is obviously unfit for discharge (2). The
incidence of PE acutely w/ upper extremity procedure is low enough not to require prophylaxis or an
immediate VQ scan (3). He appears to be hemodynamically stable so a central venous line (4) with its risk of
pneumothorax would not be indicated. Observation (1) is an option considering the pts hx of smoking and
likely poor reserves but since he has been observed for 4 hours already, something more needs to be done.
Since phrenic nerve injuries are not uncommon, an chext x-ray (5) would be appropriate.
ANSWER

5.

Chest radiograph

References:
Weber: Scalene regional anesthesia for shoulder surgery in a community setting: An assessment of risk.
JBJS Am 2002;84:775-9.

181. What normal antagonist muscle to the tibialis posterior everts the hindfoot and unlocks the
transverse tarsal joint?
1.
2.
3.
4.
5.

Anterior tibial
Flexor hallicus longus
Peroneus longus
Peroneus brevis
Gastrocnemius-soleus complex

The tibialis posterior is the primary stabilizer of the medial longitudinal arch of the foot and
primary initiator of hindfoot inversion. Hindfoot inversion allows the transverse tarsal joints to
lock during toe-off for push off. The peroneus brevis is the antagonist muscle to the TP and
everts the hindfoot. The case report referenced to this question is of a woman who had a
posterior tib transfer for posttraumatic common peroneal nerve palsy, a common procedure for
foot drop. She went on to have a flatfoot deformity secondary to this because of the imbalance
this created allowing the peroneus brevis to overpull creating hindfoot eversion.
ANSWER

Vertullo CJ, Nunley JA: Aquired Flatfoot Deformity following posterior tibial tendon transfer
for peroneal nerve injury: A Case Report. JBJS 84(A): 1214-1217

182. A 26-year-old man sustains a pelvic injury in a motor vehicle accident. AP and Judet
radiographs are shown in Figures 64a through 64c. Definitive management should consist
of
12345-

skeletal traction followed by progressive mobilization.


closed reduction and percutaneous screw fixation.
open reduction and internal fixation via the Kocher-Langenbach approach.
open reduction and internal fixation via the ilioinguinal approach.
open reduction and internal fixation via the extended iliofemoral approach.

Letournel E, Judet R (eds): Fractures of the Acetabulum, ed 2. Berlin, Germany, Springer Verlag, 1993,
pp. 417-421.
Judet R, Judet L, Letournel E: Fractures of the acetabulum: Classification and surgical
approaches for open reduction. A preliminary report. J Bone Joint Surg Am 1964;46:1615-1646.

From the radiographs provided, the radiographic markers can be identified: (1)
iliopectineal line (anterior column), (2) ilioischial line (posterior column), (3) roof of the
acetabulum, (4) anterior wall, and (5) posterior wall.
Starting with Figure 64c (the obturator oblique view), the pelvic brim, the anterior
column, and the anterior lip all appear to be intact. Now on Figure 64b (the iliac oblique view),
the ilioischial line, the posterior column, and the posterior wall all appear to be disrupted. Based
on the Letournel classification of acetabular fractures, this one appears to be an associated type
(as in associated patterns vs. simple patterns), specifically posterior column + posterior wall.
(Since there is no apparent involvement of the anterior column, there should be no transverse, ttype, or anterior components to this fracture.
With that settled (hopefully), there are two treatment options: operative and
nonoperative. The desirability of surgery is based on its ability to restore hip joint stability and
congruity of the weight-bearing acetabulum. First question: is the hip clearly subluxated on any
of the views? No, the hip appears congruent even with the patient out of traction. Second
question: is the weight bearing dome affected? (Recall the roof-arc angles: 25 anterior roofarc, 45 medial roof-arc, 70 posterior roof-arc; these indicate the weight bearing regions). Here,
there is an obvious posterior wall fracture and certainly less than 70 of the posterior roof arc is
involved. Thus, this is not amenable to skeletal traction or closed reduction (#1 and #2 are
wrong).Given that this requires open reduction and internal fixation, our final question is: what
approach to use? Here are the general guidelines:
1) Iliofemoral: anterior column and anterior wall with no extension distal to the iliopectineal
eminence.
2) Ilioinguinal: difficult fractures with anterior displacement in which access to the entire
anterior column is required; allows access to the anterior column as far as the symphysis and
includes the quadrilateral plate; can be used for both column fractures but only if the
posterior fragment is large and in one piece.
3) Kocher-Langenbach: isolated poaterior wall injuries as well as posterior column injuries;
has a higher risk of heterotopic ossification and risk to the sciatic nerve.
4) Extended iliofemoral: excellent visualization of the outer table of the ilium, the superior
dome, and the posterior column.
The approach used for this posterior wall + posterior column fracture pattern is the KocherLangenbach approach.

ANSWER

183. A 4 year old girl walks with an intoeing gait, and her parents note that she occasionally
trips. Examination reveals hip internal rotation of 90 deg, a thigh-foot angle of 10 deg
external rotation, feet with straight lateral borders and mild pronation, and a 0.5 cm
limb-length discrepancy. Neurologic examination is normal. Management should
consist of:
1. a 0.5 cm internal shoe lift
2. corrective shoes with a Thomas heel and medial arch support
3. twister cables
4. bilateral proximal femoral derotation osteotomies
5. observation

Rotational problems of the lower extremity include femoral anteversion, tibial torsion, and
metatarsus adductus. All of these problems may be a result of intrauterine positioning and
commonly present with an intoeing gate. The intrauterine position of the fetus molds the femur
by rotating it laterally and molds the tibia by rotating it medially. These molding effects usually
resolve spontaneously during infancy, and then genetically determined individual differences are
unmasked. Out-toeing in infants, medial tibial torsion in toddlers, and medial femoral torsion in
young children are extremes of a normal developmental pattern. In the vast majority, these
rotational variations fall within the broad range of normal and require no treatment.
In this case, a child presents with femoral anteversion, internal rotation of the femur seen in 3 to
6 year olds. Increased internal rotation and decreased external rotation are noted on examination
of a child with an intoeing gate and whose patellas are internally rotated. Children with this
problem classically sit in a W position. If associated with tibial torsion, femoral anteversion
may lead to patello-femoral problems. This disorder usually corrects spontaneously by age 10,
but in the older child with less than 10 degrees of external rotation, femoral derotational
osteotomy (intertrochanteric is best) may be considered for cosmesis.
This child has a thigh-foot angle of 10 degrees which falls into the range of normal values (0-20
degrees). She also has feet with straight lateral borders which indicates the absence of
metatarsus adductus. In general, projected leg-length discrepancies at maturity of <2 cm are
observed or treated with shoe lifts. Discrepancies of 2-5 cm can be treated with epiphysiodesis
of the long side, shortening of the long side (ostectomy), or lengthening. Discrepancies of >5 cm
are generally treated with lengthening.
ANSWER

184. Cellular DNA in transplanted anterior cruciate ligament allografts in dogs has been
shown to survive no longer than how many weeks?
6. 1
7. 2
8. 4
9. 8
10. 16

The answer to this question comes directly out of work done by Goertzen and collegues.
Their paper in Arch Orthop Trauma Surg. 1998; 117(4-5):208-14, describes their findings. The
purpose of the study was to determine the time dependent fibrocyte donor cell survival rate after
cryopreserved bone-ACL-bone allograft transplantation. They used DNA fingerprints to identify
and document the survival rate of the cellular DNA in transplanted ACL allografts for ACL
reconstruction in the knee joints of 10 skeletally mature dogs. At 4, 8, 26, and 52 weeks after
ACL allograft transplantation, DNA probes , H & E, Giemsa, Goldner, PAS and polarized light
staining was done to determine the time dependant changes in allograft after transplantation. At
4 weeks host fibrocytes began to grow into the graft; however, histologically, the cells could not
be distinguished as to host or donor origin. After 4 weeks the DNA pattern reflected only the
band pattern of the host. They argue that this reveals early cellular infiltration activity of the host
into the ACL allograft, which also is demonstrated in the light microscopy stainings.
They contend that the survival rate of the transplanted allogenous ACL fibrocytes had not
been documented before this study. Further, they suggest that there is no evidence that the ACL
allograft cells survive in the intra-articular environment of the hosts knee. Within 4 weeks ACL
allografts become completely repopulated with host cells. The cells that migrate early into the
ACL allografts are probably of synovial origin because they are present before revascularization
and collagen reorganization occur. From these results, the authors conclude in this paper that
viable cells in transplanted ACL allografts do not survive longer than 4 weeks after intra-articular
transplantation.
ANSWER

185. A 68-year-old woman with dementia undergoes a cemented unipolar hemiarthroplasty for a
displace femoral neck fracture via a direct lateral (Hardinge) approach to the hip. Postoperative
rehabilitation instructions should include
1) use of a high chair
2) use of an abduction pillow
3) avoidance of hip flexion, adduction, and internal rotation
4) avoidance of hip extension and external rotation
5) avoidance of elevation of the head of the bed above 60 degrees

After lateral approach to hip, external rotation and hip extension should be avoided in order to
prevent dislocation. Use of a high chair, preventing hip flexion, use of abduction pillow are
recommended postoperatively for the posterior approach to the hip. In the posterior approach,
low chairs and toilets should be avoided, in addition to the crossing of legs.
ANSWER

186: An otherwise healthy 5-month-old boy has a spinal deformity. MRI scans of the brain and
spinal cord and his neurologic examination are normal. The radiograph shown in Figure 65
reveals a left apex curve of 20 degrees and a rib vertebral angle difference of 12 degrees.
Management should consist of
1.) immediate surgery, including anterior and posterior hemiepiphyseodesis
2.) immediate surgery using a subcutaneous growth rod technique
3.) immediate bracing using a thoracolumbosacral orthosis
4.) observation, anticipating progression and early anterior and posterior fusion
5.) observation, anticipating spontaneous correction of the curve

Infantile idiopathic scoliosis is a structural, lateral curvature of the spine occurring in patients
younger than 3 years of age. James, who first used the term infantile idiopathic scoliosis, noted
that these curves occurred before 3 years of age, were more frequent in boys than in girls, and
were primarily thoracic and convex to the left.
Wynne-Davies noted plagiocephaly in 97 children in whom curves developed in the first 6
months of life; the flat side of the head was on the convex side of the curve. She also found
mental retardation in 13%, inguinal hernias in 7.4% of boys with progressive scoliosis,
congenital dislocation of the hip in 3.5%, and congenital heart disease in 2.5% of all patients.
This led her to believe that the etiological factors of infantile idiopathic scoliosis are multiple,
with a genetic tendency that is either triggered or prevented by external factors.
McMaster and Macnicol suggested that infantile idiopathic scoliosis is preventable and that the
position in which an infant is laid may be the causative factor. Brown et al. and Mau observed
that during the first few months of life all infants have a natural tendency to turn toward their
right side when laid supine. McMaster et al. postulated that when an infant is laid supine and
partially turned toward the right side, gravity causes plastic deformation of the immature thorax.
The uppermost hemithorax tends to fall backward, causing the thoracic vertebrae to rotate
backward on their long axes. Biomechanical coupling of axial rotation and lateral bending then
produces a lateral curvature of the spine with a left-sided thoracic scoliosis in infants lying on
their right sides. If laid in the prone position, a child cannot roll onto its side, and this prevents
asymmetrical molding of the thorax and promotes early maturation of the spinal extensor
muscles. McMaster et al. found that no infants in Edinburgh who were laid prone subsequently
developed infantile idiopathic scoliosis and suggested that the ideal treatment for this condition
is prevention by laying infants prone.
Infantile scoliosis may be progressive, usually increasing rapidly, or resolving (structural
resolving) spontaneously within a few years with or without treatment. The resolving type occurs
in 70% to 90% of patients with infantile idiopathic scoliosis.

ANSWER

187. Patients sustaining anterior hip dislocations, when compared to those sustaining posterior
hip dislocations, have a higher rate of
1 femoral head impaction
2 osteonecrosis
3 acetabular fracture
4 infrafoveal femoral head fracture
5 sciatic nerve injury

Items 2 through 5 are complications significantly more common in posterior dislocations due to
mechanism and directional forces. Only the first option is an outcome associated with anterior
dislocation. AVN, Acetabular fracture, Femoral Head Fractures, and sciatic nerve injury are all
significant detrimental factors which contribute to poor outcomes from posterior hip dislocations
as noted in the abstract which follows.
Dreinhofer KE. Schwarzkopf SR. Haas NP. Tscherne H.
Journal of Bone & Joint Surgery - British Volume. 76(1):6-12, 1994 Jan. From 1974 to 1989,
we treated 50 patients with a simple dislocation of the hip: 38 were posterior dislocations and 12
were anterior. All dislocations primarily treated at our hospital were reduced by closed methods
within three hours (mean 85 minutes (10 to 180)) and 43 were reviewed after an average followup of 8 years (2 to 17). It is widely held that isolated hip dislocation reduced within six hours
gives an excellent outcome, but we found a significant number of complications. There were
radiological signs of partial avascular necrosis in two, mild osteoarthritis in seven, and moderate
degeneration in two. Heterotopic ossification was seen in four patients, but 29 of 33 MRI
examinations were normal. Objective evaluation according to the Thompson and Epstein (1951)
criteria showed fair and poor results in 3 of 12 anterior dislocations, but in 16 of 30
Posterior dislocations. In six of the seven patients with no other severe injury, the hip had an
excellent or good result; in only three of the eight patients with severe multiple injuries was this
the case. The important factors in the long-term prognosis appear to be the direction of the
dislocation and the overall severity of injuries.
ANSWER

188. A 67 year old woman has a residual spastic equinovarus deformity of the left ankle
secondary to a stroke sustained 4 years ago. Use of an ankle foot orthosis has improved
her gait but is poorly tolerated because of positioning and difficulties with application
and removal. Examination shows adequate passive ankle motion, muscle strengths of
grade 4 or better, and normal sensation. The next most appropriate step in management
should consist of:
1) Botulinum toxin injections to the gastroc-soleus complex
2) ankle arthrodesis
3) split anterior tibialis tendon transfer and gastroc-soleus complex recession.
4) Posterior tibial tendon transfer to the peroneus longus
5) Peroneus longus tendon lengthening and gastroc-soleus complex recession.

Equinus deformity is caused by over activity by gastroc-soleus complex. Recession of the


gastroc complex is performed by releasing the origin of each head. Neurectomy of the medial
head may be performed in severe cases. The varus deformity can be treated by split anterior
tibialis tendon transfer or SPLATT. The tibialis anterior is split from its insertion (leaving the
medial portion attached), splitting the tendon down to the level of the myotendinous junction,
then rerouting the lateral split portion to a drill hole in the cuboid or 3rd cuneiform.
ANSWER

189. Figures 66a and 66b show the T2-weighted sagittal and axial MRI scans of a 55-year-old
woman who has low back pain. What condition is seen at L5-S1?
1- Epidural abscess
2- Annular fissure
3- Facet cyst
4- Posterolateral disk herniation
5- Foraminal disk herniation

Figures 66a and 66b demonstrate T2-weighted sagittal and axial images of the lumbar spine with
a posterolateral disk herniation at the L5-SI level. Disk pathology can be characterized as disk
protrusion, extrusion and disk sequestration depending on the degree of disruption of the annuls
and whether the nucleus material remains intact. These figures demonstrate a posterolateral disk
herniation which is the most common location due the weakness of the posterior longitudinal
ligament in this area.
Annular fissures or tears can be characterized as concentric, radial and transverse. T2-weighted
images are usually depicted as an area of high signal intensity extending into an area of
decreased signal of the annulus-ligamentous complex.
Epidural abscess should always be included within the differential diagnosis of sequestered
lumbar discs. An epidural abscess would demonstrate a high signal intensity on T2 weighted
images.
The MR images are not consistent with a foraminal disk herniation. Although neither image
shows the pedicle, a foraminal herniation occurs just below the pedicle and would be seen more
lateral on the axial image and not at the level of the disk on the sagittal image.
Facet cysts are rare and are linked to degenerative changes in the spine. They represent
enlargement of the articular synovial capsule in the spinal canal and communicate with the facet
joint. They are thought to occur by herniation of the synovium through a defective joint capsule.
Neurological deficits are unusual. On MRI synovial cysts are smooth, extradural, wellcircumscribed cystic masses arising adjacent to the facet joint. Synovial cysts without
hemorrhage appear isointense, or slightly hyperintense, relative to the cerebrospinal fluid on T1weighted images and hyperintense on T2-weighted images.
ANSWER

Radiology of the Lumbar Spine, in Herkowitz HN, Garfin SR, Balderston RA, Eismont FJ, Bell
GR, Wiesel SW (eds): The Spine, ed 4. Philadelphia, PA, WB Saunders, 1000, pp 109-134.

190. What is the primary function of the central band of the interosseous membrane of the
forearm?
1- Transfer of load from the ulna to the radius
2- Transfer of load from the radius to the ulna
3- Stabilization of the distal radioulnar joint during pronation
4- Stabilization of the distal radioulnar joint during supination
5- Stabilization of the proximal radioulnar joint

The interosseous membrane maintains the interosseous space between the radius and the ulna
through forearm rotations and actively transfers forces the radius to the ulna. The interossesous
membranes load transferring ability reduces the forces placed on the radiocapitellar joint. IOM
functions similarly to a composite material composed of stiff central fibers(longitudinal)
surrounded by a supporting fibrous matrix (transverse portion).
The reference study examined the role of the interosseous membrane (IM) in load sharing of the
forearm. The authors measure the axial loads in the distal radius and ulna and the proximal
radius and ulna with an intact and transected interosseous membrane. With an intact IM, the
proximal ulna carried more load than the distal ulna. The distal radius carried more load than the
proximal radius. After transected the IM, loads were carried equally in all forearm positions.
This suggested that the IM transferred load from the distal radius to the proximal ulna.
ANSWER

Stabile KJ, et al: The Essex-Lopresti fracture-dislocation factors in early management and
salvage alternatives. Hand Clin 2002; 18: 195-204
Birkbeck DP, et al: The interosseous membrane affects load distribution in the forearm. J Hand
Surg Am 1997; 22: 975-980

191. A father and son have aortic root widening and superior lens dislocation. The father also
has scoliosis and acetabular protrusion. No other family members have significant medical
problems. What is the genetic transmission of this syndrome.
12345-

Autosomal Recessive
Autosomal Dominant
X-linked recessive
X-linked dominant
Multifactorial

The findings described are that of Marfan syndrome. This disease is autosomal dominant
with variable penetrance. The defective gene has been mapped to fibrillin.
Unfortunately, except for eliminating X-linked from the choices, the other three choices could
not be eliminated unless you knew that.
ANSWER

Reference:
Gray JR: Marfan Syndrome. J Med genetics 1996; 33:403-8
Orthopaedic Basic Science, 2nd edition

192. A 38-year-old man sustains the injury shown in Figures 67a thru 67c. In the system of
Letournel and Judet, the acetabular fracture would be classified as what pattern?
1.anterior wall plus posterior hemitransverse
2.associated both column
3.T-shaped
4.transverse plus posterior wall
5.posterior column plus posterior wall

The Judet-Letournel classification of acetabular fractures involve 5 elementary and 5


associated fracture patterns.
The 5 elementary patterns include:
posterior wall, posterior column, anterior wall, anterior column, transverse
The 5 associated patterns include:
T-shaped, posterior column + posterior wall, transverse + posterior wall, anterior column
posterior hemitransverse, both column
Radiographic evaluation of acetabular fractures typically starts with an AP pelvis and Judet
views, which include the obturator and iliac oblique views. The iliac oblique radiograph is a 45
degree external rotation view and best demonstrates the posterior column and anterior wall. The
obturator oblique view (45 degree internal rotation) is best for evaluating the anterior column and
the posterior wall.
Review of the radiographs in this question reveal disruption of the anterior column on the
obturator oblique view and disruption of the posterior column on the iliac oblique view. Thus,
this represents a both column fracture pattern. Both columns are separated from each other and
from the axial skeleton, creating a floating acetabulum. Ideally, this is represented as the spur
sign on the obturator oblique view.
ANSWER

References:
Letournel E, Judet R: Fractures of the acetabulum, ed 2, 1993.
Judet R et al: Fractures of the acetabulum: Classification and surgical approaches for
open reduction. JBJS Am 1964; 46:1615-1646.

193.

A 40-year-old woman has a painful distal thigh mass. Plain radiographs are normal.
Figures 68a and 68b show the MRI scans, and a biopsy specimen is shown in figure
68c. What is the most likely diagnosis?
1.
2.
3.
4.
5.

Ganglion cyst
Giant cell tumor of the tendon sheath
Synovial sarcoma
Neurilemmoma
Atypical lipoma

A neurilemoma is typically a solitary encapsulated lesion that may be cystic when it is


large. Usually involves large peripheral nerves. Histologically, it contains two tissue types.
Antoni A tissue consists of spindle cells with some palisading. Antoni B tissue may have cystic
spaces and thick-walled blood vessels. Radiographically, the mass tends to be isointense with
the nerve on T1 and hyperintense on T2 (high cellular content).
Throwing out the wrong answers: Ganglion cyst obviously not. Although the MRI
looks sort of cystic, the histo shows a highly cellular tumor, something that is not characteristic
of a ganglion. Giant cell tumor of the tendon sheath occurs most commonly in the hands. Plus
there are no multinucleated giant cells on the slide. Synovial sarcoma is contiguous with bone
50% of the time (this is not). On T2 imaging, it is heterogeneous, has liquid levels, and adipose
areas, but it may be homogeneous and cystic appearing also. Histologically, cells are spindle
shaped and arranged into nests or cords. There may also be cavities or clefts filled with
eosinophilic material. We dont see that on our histo slides.
FYI: A neurolemoma grows slowly and usually does not cause symptoms unless it grows
within a confined space such as the tarsal tunnel. Clincal symptoms can include localized pain
or swelling. Arises from schwann cells.
ANSWER

Forest, M., Tomeno, B., Vanel, D.: Orthopaedic Surgical Pathology. 1998

194. Osteonecrosis occurs in association with which of the following diseases?


1. Hemachromatosis
2. Human immunodeficiency virus
3. Osteogenesis imperfecta
4. Marfan syndrome
5. Osteoporosis

A number of clinical conditions, both traumatic and nontraumatic, have been associated with
osteonecrosis of the femoral head. Some of the more common ones include history of trauma,
corticosteroid use, alcohol abuse, sickle cell anemia, coagulopathies, Lupus, radiation therapy,
and chronic pancreatitis. In a study by Ries et al., the records of 50 consecutive patients treated
surgically for osteonecrosis of the femoral head were reviewed. Twelve patients had a history of
trauma and were excluded from the study. Of the remaning 38, 7 were HIV positive and 31 were
HIV negative. Of the 7 who were positive, 4 (57%) had none of the other known risk factors. Of
the 31 who were HIV negative, only 4 (13%) had none of the other known risk factors. The
authors concluded that HIV infection is a risk factor for the development of osteonecrosis of the
femoral head. HIV has also been known to be associated with decreased bone mineral density or
osteopenia.
ANSWER

Ries MD, Barcohana B, Davidson A, Jergesen HE, Paiement GD: Association between human
immunodeficiency virus and osteonecrosis of the femoral head. J Arthroplasty 2002; 17: 135-9.
Lavernia CJ, Sierra RJ, Grieco FRL: Osteonecrosis of the femoral head. J Am Acad Orthop Surg
1999; 7(4): 250-61.

195. The application of 149 degree heat to collagen tissue immediately results in
12345-

ANSWER

increased tissue length


increased collagen synthesis
decreased tissue stiffness
cellular proliferation
no cellular changes

ARNOCZKY SP, et.al., Thermal modiification of connective tissue, ICL 2001: 50: 3-11.

196. What is the most common complication reported in the surgical treatment of olecranon
fractures in adults?
12345-

ANSWER

infection
ulnar nerve neuritis
hardware failure
heterotopic bone
painful hardwaree

HAk DJ, et.al. Olecranon fractures: treatmetn options JAAOS 2000; 8; 266-275.

197. A 20 yo man who sustained a twisting injury to his knee 3 weeks ago now reports a
persistent limp, is unable to fully extend his knee, and has difficulty doing deep knee bends.
Radiographs are normal. What component of the physical examination is most likely to indicate
the presence of a meniscus tear?
1.
2.
3.
4.
5.

Lachmans test
McMurrays test
pivot shift test
Joint line tenderness
medial or lateral joint line opening

Answer: Test characteristics of the physical examination for meniscal or ligamentous injuries of
the knee* (from Solomen, et al. JAMA 2001;286:1610-1620- represents data from 24 studies).
Likelihood ratio= sensitivity/(1-specificity).
Summary Likelihood Ratios
Injury

Examination Maneuver Number of Studies Sensitivity (%) Specificity (%) +LR

-LR

ACL

General examination
Anterior drawer test
Lachman test
Lateral pivot-shift test
General examination
Posterior drawer test
Abduction stress test
General examination
Joint-line tenderness
McMurray test

0.04
0.3
0.1
0.05
0.1
0.4
1.1
0.8

PCL
Meniscus

ANSWER

3
9
7
5
2
2
1
5
4
4

Joint line tenderness

62-100
9-93
60-100
27-95
91-100
51-86
94
64-82
76-85
29-63

56-100
23-100
100
NR
80-99
NR
100
78-84
11-43
29-100

25.0
3.8
42.0
21.0
94.0
2.7
0.9
1.3

198. A 13 year old girl sustains an injury shown in Figures 69a and 69b in a fall. Exam reveals
this to be an isolated injury, and the patient is neurovascularly intact. Base on these findings,
management should consist of:
1)
2)
3)
4)
5)

a long arm cast


closed reduction in E.D. and long arm cast
closed reduction under general anesthesia and long arm cast
closed reduction and percutaneous pinning
open reduction and internal fixation

Ref: Rockwood Peds Fracture, Skeletal Trauma


Kid has a displaced rotated lateral condyle fracture. Non operative treatment is not going to flip
that thing around heal it. So 1 is wrong. Theoretically, you could attempt a closed reduction and
some sort of stabilization, but this has been tried in the past and there have been poor results.
With this particular fracture, the best thing to do is open it up and reduce under direct
visualization and then fix it.
ANSWER

199 The sagittal oblique MRI scan shown in Figure 70 reveals a lesion in the shoulder that
typically affects what neurologic structure?
1- Axillary nerve
2- Musculocutaneous nerve
3- Long thoracic nerve
4- Suprascapular nerve to the infraspinatus muscle
5- Suprascapular nerve to the supraspinatus muscle

Clearly, what you are looking at in the MRI is the worlds worst image of a shoulder. The
question does say that it is a sagittal oblique image, so that helps a little. They also tell you its a
shoulder, cause chances are you wouldnt be able to tell from the picture. What you are looking
at is sort of a Scapular Y type of view. The large globular structure is a cyst, and its in the
spinoglenoid notch impinging upon the suprascapular nerve.
The suprascapular nerve orginates from the upper trunk of the brachial plexus and is contributed
to by the C5 and C6 cervical roots. It courses under the body of the omohyoid and anterior
border of the trapezius muscle to the scapular notch and is fixed there by a fibroosseous tunnel.
The nerve passes through this notch, under the transverse scapular ligament and first innervates
the supraspinatus muscle and also gives off sensory branches to the shoulder and
acromioclavicualar joint. The scapular notch is the most common location of impingement. The
nerve continues on to innervate the infraspinatus by passing through the spinoglenoid notch.
Entrapment here would cause infraspinatus atrophy and weakness. Entrapment of the nerve can
be exacerbated by activities such as weight lifting, backpacking, and volleyball. Clinical findings
include supra/infraspinatus weakness and poorly localized pain, often with associated loss of
abduction and external rotation. Xrays, electromyographic and nerve conduction studies help pin
point the lesion.

ANSWER

Fehrman DA, Orwin IF, Iennings RM: Suprascapular nerve entrapment by ganglion cysts: A
report of six cases with arthroscopic findings and review of the literature. Arthroscopy
1995;11:727-734.
Black KP, Lombardo IA: Suprascapular nerve injuries with isolate paralysis of the infraspinatus.
Am I Sports Moo 1996;18:225-228.

200. Figures 71a and 71b show the radiographs of a 38 year old woman who sustained an
isolated right hip injury in a motor vehicle accident. Treatment should consist of
12345-

ANSWER

balanced skin traction


open reduction internal fixation with a 135 degree sliding hip screw
antegrade femoral nail using a piriformis entry point
open reduction internal fixation with a 95 degree fixed angle lateral implant
a calcar replacement bipolar hemiarthroplasty

Haidukewych GJ, et.al., Reverse obliquity fractures of the intertrochanteric region of the femur
JBJS 2001; 83; 643-650.

201. Figure 72 shows the radiograph of a 4 year old girl who has a painless right Trendelenburg
limp. Management should consist of
12345-

ANSWER

closed reduction
open reduction with femoral shortening and pelvic osteotomy
traction followed by closed reduction
medial soft tissue release, traction, and closed reduction
pelvic osteotomy without reduction of the joint

OKU 7 p 387-394.

202. When using the ilioinguinal approach to the acetabulum, the middle (second) window uses
the interval between which of the following structures?
12345-

Iliac wing and iliopsoas muscle


Iliopsoas muscle and femoral nerve
Femoral nerve and external iliac artery
External iliac artery and external iliac vein
External iliac vein and pectineus muscle

The ilioinguinal approach was developed in 1965 as an anterior approach to the pelvis and
acetabulum. Before this date, the Smith-Petersen incision or a modification of it called the
iliofemoral approach provided the only access to the upper part of the anterior column of the
acetabulum. The ilioinguinal approach provides total and complete access to the anterior column
from the sacroiliac joint to the pubic symphysis. It has a lower incidence of external iliac fossa
heterotopic ossification. It does not, however, provide direct visualization of the joint surface.
The ilioinguinal approach is divided into three windows by the lacuna musculorum (iliopsoas
and femoral nerve) and the lacuna vascularum (external iliac/femoral vessels). The lateral (first)
window provides access to the iliac fossa from the anterior sacroiliac joint to the ASIS. It is
lateral to the lacuna musculorum (iliopsoas and femoral nerve). The middle (second) window
allows exposure from the ASIS to the pectineal eminence, including the quadrilateral plate. It is
between the lacuna musculorum (iliopsoas and femoral nerve) and the lacuna vascularum
(external iliac/femoral vessels). The medial (third) window provides access to the symphysis
pubis by removing recutus abdominis from its insertion onto the symphysis. It is medial to the
lacuna vascularum (external iliac/femoral vessels).
ANSWER

Letournel E, Judet R (eds): Fractures of the Acetabulum, ed 2. Berlin, Germany, Springer Verlag,
1993, pp 417-421.
Letournel E: The treatment of acetabular fractures through the ilioinguinal approach. Clin Orthop
1993;292:62-76.

203.

A patient is diagnosed with a disk herniation at the L4-L5 level. At the time of lumbar
diskectomy, the most effective method of ensuring surgery at the proper level is with
12345-

palpation of the spinous processes.


palpation of the iliac crests.
an intraoperative lateral radiograph with a marker in place.
an intraoperative AP radiograph with a marker in place.
use of a computer-guided navigation system.

Although all of these techniques can be used, the most effective method is a lateral spine
radiograph with a marker. We always do this during spine surgery for good reason.

ANSWER:

An intraoperative lateral radiograph with a marker in place.

AAOS Advisory Statement: Wrong Site Surgery. 1998.


AAOS Council on Education: Report of the Task Force on Wrong Site Surgery, 1998.

204. A 19 year old track athlete who has aching and burning pain in her right leg reports that
her symptoms usually begin about 10 minutes into the run and slowl resolve 30 to 40 minutes
after the run. She has decreased her running distance in half, with no change in her
symptoms. She notes tingling occasionally over the dorswal aspect of the right foot while
running. Examination reveals mild diffuse tenderness over the anterolateral leg but no point
of focal tenderness over the tibia. Distal pulses are normal. Plain radioagraphs and a bone
scan of the legs are normal. Stretching and strengthening exercises have failed to provide
releif. What is the next step in evaluation.?
12345-

ANSWER

three phase bone scan


electromyography and nerve conduction studies
measurrement of compartment pressures before and after exercise
arteriography
MRI with contrast

Schepsis AA, et.al.; Surgical managemntt of exertional compartment syndromes of the lower
leg: long term followup. Am J sports Med. 1993; 21;811-817.

205. A 34-year-old man reports vague right upper extremity discomfort following a
recreation sports injury. Figures 73a through 73d show a radiograph, a T1-weighted coronal
MRI scan, and low- and high-power views of the biopsy specimen. What genetic defect is
associated with this disease?

31- Type I collagen mutation


32- FGF receptor mutation
33- Gs-alpha protein
34- Ring chromosome 12
35- Translocation (12;16).

The Xray (supposed to be ground glass, expansile lesion with cortical thinning?), MRI
(intracortical, well-circumscribed, lobular, low-signal intensity) and biopsy specimen (spindle
cells which are whorled or storiform, with interspersed trabeculae of immature woven bone
without surface osteoblasts) are consistent with a fibrous dysplasia. Fibrous dysplasia is best
explained as a dysplastic anopmaly of bone-forming tissue.it is unable to produce (focally or
generally) mature lamellar bone. It is thus proposed that mutation of the G proteins (the genes
that modulate cellular differentiation into mature osteocytes capable of producing matue lamellar
bone) are the culprit. The disorder is a broad spectrum from solitary lesions to a severe
generalized condition.the thought is that G protein mutations alter the tissue differentiation
process.
ANSWER

Type 1 collagen mutation=osteogenesis imperfecta


FGF receptor mutation= achondroplasia
Ring chromosome 12=no clue
Translocation 12;16 =liposarcoma

206. Compared to reamed nailing of tibial shaft fractures, an unreamed technique is associated with
1.
2.
3.
4.
5.

more rapid restoration of endosteal blood flow


more rapid fracture healing
decreased cortical perfusion
decreased new bone formation
increased periosteal perfusion

Intramedullary nailing is the treatment of choice with many closed and open diaphyseal fractures.
Reamed nails have been the standard because of increased stability at the fracture site, improved ease w/ nail
insertion, reduced risk of nail incarceration, nail perforation and causing comminution. Cortical reaming and
nail insertion injury the medullary vascular system and results in avascularity for portions of the cortex.
The authors of the referenced articles performed in vivo studies on a sheep tibia model in which they
compared reamed vs. unreamed techniques. They demonstrated that unreamed nails (when compared to
reamed) have less cortical porosity, increased cortical profusion (not decreased as in 3) There was no
difference in the amount of new bone formation (eliminates 4) and all of the fractures in the series healed in
the same time (eliminates 2). Although not specifically noted, the periosteum is not disrupted in either
technique but some believe it increases w/ reamed nails (eliminates 5). The cortical revascularization and
return of endosteal blood flow returned sooner in the unreamed nail (1).
ANSWER

1.

more rapid restoration of endosteal blood flow

References:
Schemitsch: Cortical bone blood flow in reamed and undreamed locked intramedullary nailing: A fractured
tibia model in sheep. JOT 1994; 8:373-82.
Schemitsch: Quantitative assessment of bone injury and repair after reamed and undreamed locked
intramedullary nailing. J Trauma 1998; 45:250-5.

207. A 15yo boy who has been undergoing tibial lengthening for the past 2 months now reports
pain for the past 5 days while distracting the frame. Examination reveals pin sites are clean and
the pins are well tensioned. Radiographs show well-formed regenerate bone; however, the pins
appear bent. What is the reason for his pain?
1.
2.
3.
4.
5.

Pin tract infection


Pin placement near a cutaneous nerve
Fracture through regenerate bone
Premature consolidation of the regenerate bone
Poor tensile strength of the regenerate bone

First of all, you can eliminate answers 1 and 2. There is no evidence of infection as you are told
the pin sites are clean. If the pins were placed near a cutaneous nerve, it is unlikely that the
patient would have sxs starting 2 months into the lengthening. Answer 3 is a possibility, but then
the pt would have pain when weightbearing, not just when distracting. Answer 5 doesnt explain
pain. So that leaves you with the correct answer of premature consolidation which is a common
comlplication in lengthening in children because they heal fractures so quickly, and the
osteotomy is no different. The bending pins are another clue that the frame is moving, but not
the bone.
ANSWER

The references given are not easily obtainable.

208. What percent of patients who receive a steroid injection for carpal tunnel syndrome will
experience at least transient relief of their symptoms?
12345-

0%.
20%.
50%.
80%.
100%.

References:
Koval KJ (ed): Orthopaedic Knowledge Update 7. Rosemont, IL, American Academy of
Orthopaedic Surgeons, 2002, pp. 359-377.
Green DP: Diagnostic and therapeutic value of carpal tunnel injection. J Hand Surg Am
1984;9:850-854.
Explanation:
In his retrospective study of 233 patients, Green examined the diagnostic and therapeutic
value of steroid injections in patients with carpal tunnel syndrome. He administered 228
injections. His follow-up ranged from 6 to 45 months. 81% of the patients obtained good or
complete relief lasting from 1 day to 45 months. In most of the patients, symptoms began to
recur after about 2 to 4 months (average 3.3 months), but in only 46% were recurrent symptoms
severe enough to warrant surgical treatment. (According to Green, use a 1-1/2 inch 25-gauge
needle just ulnar to the palmaris longus, slightly dorsiflex the wrist, 30 degrees to the skin and in
a slightly radial direction. Direct injection of steroid solution into the median nerve is to be
avoided.). Thus the preferred response is 4.
ANSWER

209. A 44-year-old man has had aching right shoulder pain for the past several months.
Figures 74a through 74c show the plain radiograph, sagittal T2-weighted MRI scan, and
biopsy specimen. Management should consist of
1. extended intralesional curettage
2. observation
3. wide intra-articular excision
4. wide extra-articular excision
5. forequarter amputation

These images represent a giant-cell tumor of the proximal humerus. Surgical resection is the
universal standard of care for treatment of giant-cell tumor of bone. The common objective of
all surgical procedures in the treatment of these tumors is to minimize the incidence of local
recurrence. Caution must be exercised to avoid overly extensive resection with subsequent
damage to the adjacent joint surface. The cellular activity of giant-cell tumors can be aggressive
and these tumors have been found to recur with a high frequency within the first 2 years after
resection. A number of surgical techniques are described. These include intralesional curretage
and bone grafting or cementation and wide resection with allograft or vascularized bone
reconstruction. Adjunctive therapies used to extend the zone of tumor cell ablation include high
speed burring, phenol and alcohol chemical cauterization, heat cauterization with electrocautery
or polymethylmethacrylate polymerization, and liquid nitrogen cryotherapy.
Ghert, et al studied 75 patients with giant-cell tumor to determine whether type of surgical
resection correlated with rate of local recurrence. This study found that regardless of type of
resection, there was no statistically significant difference detected for risk of recurrence between
the lesions treated with intralesional, wide, marginal, or radical resection. The standard
treatment at their institution is to spare the joint surface as often as possible and to do
intralesional curettage using a high-speed burr. Adjuvant therapy to extend the zone of tumor
necrosis is used in all cases with electrocautery or phenol. Reconstruction of the lesion is done
with cancellous bone grafting packed under the joint surface, augmented with PMMA, or with
PMMA alone. Phenol, which causes protein coagulation, damages DNA and causes necrosis.
ANSWER

210. The incidence of postoperative nerve palsy is highest after total hip arthoplasty for which of
the following conditions?
1. previous hip arthodesis
2. previous acetabular fracture
3. hip dysplasia in women
4. protrusio in men
5. two-stage exchange for infection

Schalzried et al. describe the incidence of nerve palsy after total hip arthroplasty in the
article Update on Nerve Palsy Associated with Total Hip Arthoplasty, Clinical Orthopaedics
and Related Research.344:188-206. In this article they suggest that the incidence of nerve palsy
after total hip arthoplasty is generally about 1%. This can be further broken down into primary
arthoplasty (0.9%) and revision arthoplasty (2.6%).
The nerve palsy noted after total hip arthoplasty primarily involve the sciatic and
peroneal nerves (79%). This appears to be the case for several reasons. One, the common
peroneal division of the sciatic is composed of fewer and larger funiculi with less connective
tissue than the tibial division, leading to less displacement and dissipation of mechanical force
after a deforming force than smaller, more widely spaced funiculi in the tibial division. Second,
the common peroneal is more tethered at the sciatic notch and the fibular neck and thus may be
more intolerate of stretch. Finally, its more lateral position in the gluteal region may predispose
it to injury.
In addition to revision surgery as an increased risk for nerve palsy after THA, female
gender and developmental hip dysplasia (number 3 in this question), also predispose to nerve
palsy during THA. Compared to men, whose incidence is approximately 0.77%, women have an
overall prevalence of 1.5% of nerve palsy after THAa two-fold increase in risk. Similarly,
patients with developmental dysplasia of the hip have an increased risk. The prevalence of nerve
palsy in this group is approximately 5.2%. For this reason, in patients with dysplasia, a
prophylactic, limited sciatic neurolysis can be helpful. This allows visualization of the nerve
and reduces the risk of iatrogenic injury.
The etiology of these nerve palsies are varied, and include direct trauma to the nerve,
excessive tension because of increase in limb length, offset, or both, bleeding or compression by
a developing hematoma, or unknown. As for limb lengthening, there are no guidelines as to the
amount of limb lengthening that can safely be achieved without running the risk of a nerve palsy.
In most cases, the origin of the palsy is unknown.
As for outcomes of these nerve palsies, what can be expected? Approximately 41% of
reported cases note complete recovery, 44% have some residual, mild deficit, and some 15% go
on to have poor outcome including weakness that prevents the patient from ambulating well
and/or persistent dysesthesia. Patients who have some motor function immediately and those
who recover some function within the first 2 weeks after surgery have the best long term
prognosis.
ANSWER

211. A 15-year-old cross-country runner has had dull, achy right groin pain for the past 3 weeks.
He began his training 16 weeks prior to the development of his symptoms. He reports that he
has not changed his running habits, shoes, or mileage during this time. He denies sharp pain,
numbness, or tingling. Examination reveals an antalgic gait. Initial hip and pelvis radiographs
are normal. What is the next most appropriate step in evaluation?
1) CBC and erythrocyte sedimentation rate
2) Repeat radiographs
3) CT
4) MRI
5) Femoral arteriography

The patient in this question suffers from a femoral neck stress fracture. Although femoral neck
stress fractures are unusual, they are not rare, they typically occur in two groups of patients:
young, healthy active adults and elderly patients with osteoporosis. Although rare in children
and adolescents, they should be considered in the differential of hip and groin pain.
A complete history and physical should be the first step in diagnosis. physical exam may reveal
the hip to be painful at the extremities of motion. The patient may ambulate with an antalgic gait
and have pain with percussion of the greater trochanter or palpation of the inguinal area
overlying the hip joint. Single legged hop test on the affected side will commonly reproduce the
pain.
If plain films do not reveal the stress fracture, then MRI would be the next step. Technitium
bone scans have long been considered the standard test for suspected stress fractures because of
their high sensitivity, however their specificity is lower than MRI. MRI has proved to be more
specific than bone scans for the early detection of stress fractures and other hip disorders in the
young endurance athlete.
ANSWER

212: A 42-year-old man has severe low back pain, urinary retention, and saddle anesthesia. His
medical history is unremarkable. What is the most likely diagnosis?
1.) Spondylolisthesis
2.) Cauda equine syndrome
3.) Peripheral neuropathy
4.) Herpes zoster infection
5.) Cervical myelopathy

Cauda equina syndrome, or injury between the conus and the lumbosacral nerve roots within the
spinal canal, also results in areflexic bladder, bowel, and lower limbs. With a complete cauda
equina injury, all peripheral nerves to the bowel, bladder, perianal area, and lower extremities are
lost, and the bulbocavernosus reflex, anal wink, and all reflex activity in the lower extremities
are absent, indicating absence of any function in the cauda equina. It is important to remember
that the cauda equina functions as the peripheral nervous system, and there is a possibility of
return of function of the nerve rootlets if they have not been completely transected or destroyed.
Most often the cauda equina syndrome presents as a neurologically incomplete lesion.

ANSWER

213. Figure 75 shows the T1-weighted MRI scan of a patient with osteonecrosis. The area
depicted by the arrow is supplied by which of the following arteries?
1 Subscapular
2 Suprascapular
3 Thoracoacromial
4 Posterior Circumflex
5 Anterior Circumflex

The humeral head blood circulation is supplied primarily by the arcuate artery which is a
terminal branch of the anterior circumflex humeral artery. A small area of the posterior inferior
aspect of the humeral head is supplied by the posterior circumflex humeral artery. The
Suprascapular, Subscapular, and thoracoacromial trunk do not supply hematalogic nourishment
to the humeral head. In our example with the anterior medial aspect of the humeral head with
osteonecrosis, the responsible arterial insufficiency comes from that of the anterior circumflex
humeral artery.
ANSWER

214. The iliac oblique radiograph shown in figure 76 shows which of the following acetabular
fracture patterns?
1.
2.
3.
4.
5.

Anterior Column
Anterior Wall
Posterior Column
Posterior Wall
Associated both column

Anterior column, please see OKU 7 pg 399 for a diagram of the various patterns.

ANSWER

215. Figure 77 show a T1-weighted MRI scan. The structure at the arrow is supplied by
which of the following nerves?
12345-

Axillary
Subscapular
Suprascapular
Long thoracic
Spinal accessory

The image is an axial MRI of the shoulder with the arrow pointed to the subscapularis muscle
which is innervated by the both the upper and lower subscapular nerves. Both the upper and
lower subscapular nerves are branches of the posterior cord of the brachial plexus.
The suprascapular nerve is a branch of the superior trunk of the brachial plexus and
innervates the supraspinatus and infraspinatus muscles.
The axillary nerve is a large terminal branch of the posterior cord of the brachial plexus. It
passes through the quadrangular space and supplies the teres minor and deltoid muscles.
The long thoracic nerve arises from the posterior aspect of the ventral rami of C5, C6 and C7
and supplies the serratus anterior muscle.
The spinal accessory nerve is also known as cranial nerve XI and supplies the
sternocleidomastoid and trapezius muscles.
ANSWER

References:
Moore, KL, Clinically Oriented Anatomy 3rd Ed. 1992

216. A 9yo boy who has SCFE is in the 40th percentile for weight. Which of the following studies
should be obtained?
1- MRI pelvis
2- CT of pelvis
3- Bone scan
4- Serum BUN and creatinine levels
5- CBC and ESR

Atypical SCFE are those associated with endocrine disorders, with metabolic disorders such as
renal failure osteodystrophy or with radiation to the pelvis. They fall into 3 subgroups: renal
failure associated, radiation therapy associated, SCFE associated with endocrine disorders.
For two patient of equal weight, those younger than 10 yrs old or than 16yrs are 4.2 times more
likely to have an atypical SCFE; for two patients of equal age, those < 50th percentile for weight
were 8.4 times more likely.
SCFE in children is classified as idiopathic or atypical. Causes of atypical SCFE include
endocrinopathies like hypothyroidism, radiation therapy, and metabolic disorders like renal
failure osteodystrophy. With idiopathic SCFE, there is a strongly associated with being
overweight and female.
The authors looked at 433 children with 612 SCFE in attempt determine a way to predict if a
SCFE is from an atypical cause. They described an age-weight test to stratify children into
groups that are likely to have an atypical SCFE. A positive age-weight test suggests that there
may be an atypical cause for the slip. The age-weight test is positive for any child < 50th
percentile in weight or for children older than 16 years old who are > 50th percentile in weight
The age-weight test is negative if the child under the age of 16 but > 50th percentile 93 % of
children in the study who were had an idiopathic SCFE. In other words, fat kids under the age of
16 are likely to have an idiopathic slip. The sensitivity is 55% but the negative predictive value
is 93% percent. This indicates that a negative age-weight test means that there is a 93% chance
that the SCFE is idiopathic rather than from an atypical cause. Therefore, children with a
positive age-weight test warrant investigation for an atypical cause of their SCFE.
The child in the question has a positive age-weight test and should be tested for an atypical
cause. Answer 4 screens for renal failure osteodystrophy.
ANSWER

Loder RT, et al: Clinical characteristics of children with atypical and idiopathic SCFE:
Description of the age-wt test and implications for further diagnostic investigation. J Pediatr
Orthop 2001; 21: 481-487

217. A 12yo boy has a flexible pes planovalgus deformity of the right foot. Orthotic
management should consist of
12345-

A semirigid orthotic with medial longitudinal arch support and medial heel posting
A semirigid orthotic with medial longitudinal arch support and lateral heel posting
An extra-depth shoe with a SACH heel
An articulated ankle-foot orthosis
A plastizote insert

This question deals with conservative management of conservative management of a


pediatric flat-foot deformity. Out of the choices above, choice two is the only one that addresses
both the flatfoot as well as the developing valgus deformity.
ANSWER

Reference:
Bordelon RL: Surgical and conservative foot care. Thorofare, NJ, Slack Inc 1988.

218. ITEM DELETED

219.

Following a subtrochanteric femoral fracture, the proximal fragment assumes a


position of
1.
2.
3.
4.
5.

extension, abduction, and internal rotation


extension, adduction, and external rotation
flexion, adduction, and internal rotation
flexion, abduction, and external rotation
flexion, abduction, and internal rotation

This question is just straight up anatomy. They have a really good picture of the
muscular deforming forces in Skeletal Trauma. But as the article states, the proximal fragment
assumes a position of flexion, abduction, and external rotation. Close your eyes and visualize
the muscular attachments to the proximal femur. First you have the lesser trochanter which
serves as the attachment of the iliopsoas, resulting in flexion of the proximal fragment. Next,
you have the greater trochanter which is an insertion for the hip ABductors gluteus medius,
minimus, resulting in abduction of the proximal fragment. Finally, you have the external rotators
which insert medial to the posterior border of the greater trochanter (or near by) piriformis,
inferior gemellus, obturator internus, superior gemellus, and quadratus femoris. These are your
short external rotators which oddly enough cause external rotation of that proximal fragment. By
thinking through things anatomically this question is pretty straight-forward. And remember that
the fracture line is proximal to the gluteus maximus or ADductor insertions so these muscles do
not act on the PROXIMAL fragment. Or you could just memorize that picture from skeletal
trauma.
ANSWER

220. Figures 78a and 78b show the MRI scans of a 59-year-old woman who has plantar foot pain
and a mass. A biopsy specimen is shown in Figure 78c. Management should consist of
1. custom foot inserts
2. local steroid injection
3. marginal excision
4. wide excision
5. Symes amputation

The MRI shows a superficial soft tissue mass on the plantar surface of the forefoot that measures
approximately 3cm by 1cm. The mass appears to be homogeneous. The histologic section is
consistent with cells with glandular morphology. Most soft-tissue tumors about the foot and
ankle are benign. Because there are no distinct fascial boundaries in the foot, one must assume
that a foot and ankle soft-tissue tumor is malignant until proven otherwise. Plantar fibromatosis
or Lederhosen disease is the most common soft-tissue neoplasm in the foot. Resection of the
overlying skin in continuity with the nodule and plantar fascia is recommended (wide excision).
The most common primary soft-tissue malignant tumor in the foot and ankle region is synovial
sarcoma. Other soft-tissue tumors of the foot and ankle include malignant melanoma, clear cell
sarcoma, and epitheloid sarcoma. The treatment for all of these is wide excision or radical
excision. In an analysis of 1,041 patients with localized soft tissue sarcomas of the extremies,
significant prognostic factors for local recurrence include age greater than 50, microscopically
positive surgical margins, and recurrent disease at presentation.
ANSWER

Cany L, Stoeckle E, Coindre JM, Kantor G, Bonichon F, Bui BN: Prognostic factors in
superficial adult soft tissue sarcoma: Analysis of a series of 105 patients. J Surg Oncol 1999; 71:
4-9.
Pisters PW, Leung DH, Woodruff J, Shi W, Brennan MF: Analysis of prognostic factors in 1,041
patients with localized soft tissue sarcomas of the extremities. J Clin Oncol 1996; 14: 1679-89.
Bos GD, Esther RJ, Woll RS. Foot tumors: diagnosis and treatment. J Am Acad Orthop Surg
2002; 10(4): 259-70.

221. When a short cephalomedullary nail is compared to a sliding hip screw and side plate for
stabilization of intertrochanteric femoral fractures, the intramedullary device has been associated
with a significantly higher
12345-

ANSWER

amount of blood loss


surgical time
rate of lag screw cutout
rate of poor functional outcomes
rate of femoral shaft fracture

Hardy DC, et.al., use of an intramedullary hip-screw compared with a compressin hip screw
with a plate for intertrochanteric femoral fractures: A prospective randomized study of one
hundred patients. JBJS 1998: 80; 618-630.

222. At age 5 years, the spinal canal has grown to approximately what proportion of the
expected adult cross sectional area?
12345-

ANSWER

25%
33%
50%
75%
95%

Lovell and Winters, 2000, pp 33-62


Weinstein, The pediatric spine, 2001, pp 203-218.

223. What factor is necessary in the development of osteomalacia?


1. insufficient dietary vitamin D
2. insufficient dietary calcium
3. failure to maintain serum calcium and phosphate levels
4. enteric absorption defects
5. renal insufficiency

Buckwalter, et al, ed. Orthopaedic Basic Science. Pg 357:


Deficient or impaired mineralization of bone matrix (osteoid) is the diagnostic feature of all
osteomalacic syndromes. The single most important causative factor in the development of
osteomalacia is the failure to maintain a serum calcium-phosphorous level sufficient to promote
mineralization of newly-formed osteoid. The causes are many and include renal
osteodystrophy, dietary deficiency (most common vitamin D), GI absorption defects, renal
tubular defects, renal tubular acidosis, some soft-tissue tumors (fibrous dysplasia,
neurofibromatosis), and anticonvulsant medications such as Dilantin (induction of P450 causes
increase in vitamin D metabolites (a favorite pimpage of Dr Marcus at VA conference))

ANSWER

224. Figures 79a through 79c show the radiograph, CT scan, and biopsy specimen of a 42 year
old woman who has a painful shoulder mass. What is the most likely diagnosis?
1)
2)
3)
4)
5)

Extraskeletal osteosarcoma
Myositis ossificans
Synovial sarcoma
Tumoral calcinosis
Calcific myonecrosis

So on xray we see s ort of globular ill defined mass. On CT scan we see a peripherally
mineralizing mass in the soft tissue. Paraphrasing Getty, if you have to look the path slide,
youre hosed.
In extraskeletal osteosarcoma, the pattern of mineralization is more from the center to the
periphery.
Myositis ossificans can develop either in response to soft tissue injury (eg, blunt trauma, stab
wound, fracture/dislocation, surgical incision) or can occur without known injury. Proposed
mechanisms for atraumatic myositis ossificans include nondocumented trauma, repeated small
mechanical injuries, and nonmechanical injuries caused by ischemia or inflammation.
Obtain serial radiographs to distinguish between myositis ossificans and osteosarcoma. In
osteosarcoma, calcification extends from center to periphery. In myositis ossificans, calcification
first occurs in the periphery of the soft tissue mass. Myositis ossificans' calcification occurs in
association with the bone's diaphysis, unlike osteogenic sarcoma's association with the
metaphysis.
Histologic Findings: Ossification has 3 distinct zones: the central undifferentiated zone, the
surrounding zone of immature osteoid formation, and the peripheral zone with mature bone. At
least 10 days are required following onset of symptoms for these zones to become apparent. If
the biopsy is performed before 10 days have elapsed, or if a biopsy sample is obtained from the
central region, the specimen yields undifferentiated tissue resembling an osteosarcoma. In
contrast to osteosarcoma, myositis ossificans exhibits a zonal pattern, the lesion has viable
muscle fibers, and myositis ossificans does not invade surrounding tissue. Biopsy performed
after ossification maturation reveals primarily mature lamellar bone. Biopsy of lesions from
patients with myositis ossificans atraumatica may lack the typical histological appearance of
myositis ossificans.

ANSWER

225 When comparing the results of tibial shaft fractures treated with intramedullary nails to
those treated in casts, patients treated with intramedullary nails can be expected to have an
increased incidence of

1- Nonunion
2- Malunion
3- Delayed union
4- Restricted knee motion
5- Knee pain

A classic complaint after IM nailing of the tibia is knee pain. In the second article, 12 of 27
patients had anterior knee pain after nailing. This was not a significant complaint for the 26
patients in the casted group. Paratendonous or insertion through the patellar tendon does not
seem alter the incidence of knee pain. Dissection in the patellar region, trauma in the patellar
tendon during nail insertion, and iatrogenic damage of the infrapatellar nerve are proposed
mechanisms.

ANSWER

Hooper GJ, Keddell RG, Penny ID: Conservative management or closed nailing for tibial shaft
fractures: A randomised prospective trial. J Bone Joint Surg Br 1991 ;73:83-85.
Karladani AH, Granhed H, Edshage B, Jerre R, Styf J: Displaced tibial shaft fractures: A
prospective randomized study of closed intramedullary nailing versus cast treatment in 53
patients. Acta Orthop Scand 2000;71: 160-167.
Koval Kl (ed): Orthopaedic Knowledge Update 7. Rosemont,IL, American Academy of
Orthopaedic Surgeons, 2002,..pp 479-488.

226. What method most effectively identifies musculoskeletal problems during the
preparticipation physical examination.
12345-

ANSWER

flexibility testing
radiographic evaluation
complete orthopaedic history and questionnaire
observation of athlete during sport specific activities
skinfold examination

OKU 7, pp 99-110.

227. Delayed onst or postexercise muscle soreness is best characterized by


12345-

ANSWER

muscle cytoskeletal hypertrophy


inflammation
reduced serum creatine kinase levels
increased serum calcium
local hematoma

Orthopaedic Basic Science, 2000, pp 684-716


Lieber RL; et.al. Morphologic and mechanical basis of delayed onset muscle soreness. JAAOS
2002; 10; 67-73

228. Which of the following is a characteristic of the flow phase of systemic metabolic response
to trauma?
12345-

Increased angiotensin levels


Increased gluconeogenesis
Increased myocontractility
Neuroendocrine release of epinephrine
Interstitial fluid shift

The systemic metabolic response to trauma has been divided into the early ebb phase followed
by the flow phase. Ebb phase is characterized by cardiovascular instability, impaired oxygen
transport, and heightened autonomic activity (e.g. hypovolemic shock). Flow phase is
characterized by hyperdynamic circulatory changes, fever, glucose intolerance, and muscle
wasting. The liver uses lactate from anaerobic metabolism and aminoacids from muscle
breakdown to form glucose (gluconeogenesis).
ANSWER

Browner BD, Jupiter JB, Levine AM, Trafton PG (eds): Skeletal Trauma. ed 2. Philadelphia. PA.
WB Saunders. 1998. pp 2327-2404.
Beaty JH (ed): Orthopaedic Knowledge Update 6. Rosemont. IL. American Academy of
Orthopaedic Surgeons. 1999. pp 123-137.

229.

A 43-year-old man reports a snapping sensation in the left shoulder with overhead
activities. Figures 80a through 80e show the radiographs, MRI scan, bone scan, and
biopsy specimen. What is the most likely diagnosis?
12345-

Osteochondroma
Chondrosarcoma
Osteoblastoma
Osteogenic sarcoma
Myositis ossificans

The history gives you no previous trauma or reason to suspect myositis ossificans. The
radiograph shows a mineralized lesion primarily outside the cortex of the proximal humerus
(also known as juxtacortical). The bone scan is not really useful other than to demonstrate that
the lesion is single or monostotic (rather than polyostotic). The fact that the lesion itself
demonstrates increased uptake is useless. The MRI demonstrates the lesion to be homogenous
and without medullary continuity. The path confirms the diagnosis. There are no cartilagenous
looking cells (so rule out chondrosarcoma). There are many, many osteoblasts, and there is
pinkish material throughout which is osteoid. So this aggressive looking lesion is bone
forming. That indicates the diagnosis is osteogenic sarcoma.
The lesion is a parosteal osteosarcoma. The proximal humerus is the second most common
location, the posterior distal femur is the most common. It often presents as this case shows
with mechanical symptoms of musculotendon irritation. The radiographic appearance is that of
an ossified oval mass attached to the surface of the injvolved bone. The lesion may show a
lucent region which separates it from the underlying cortex. The MRI can in late stages only
show medullary continuity, which is a poor prognosticator (recall that osteochondromas always
have medullary continuity). The treatment of choice is wide resection. Common to other
osteosarcomas, pre- and post-op radiation therapy is utilized, but chemotherapy has shown little
benefit.
ANSWER

Osteogenic sarcoma

Schajowicz F, et.al.; Osteosarcoma arising on the surfaces of long bones. JBJS 1988; 70:555-64.
Temple HT, et.al.; Clinical Outcome of 38 patients with juxtacortical osteosarcoma. CORR
2000; 373: 208-217.

230. During surgical treatment of a bicondylar tibial plateau fracture, the surgeon elects to
perform a posteromedial second approach to address condylar comminution. Surgical dissection
should proceed between what two structures?
12345-

ANSWER

gracilis and semitendinosus


semitendinosus and semimebranosus
semimebranosus and medial gastrocnemius
medial gastrocnemius and tibial nerve
medial collateral ligament and gracilis

Georgiadis GM; combined andterior oand posterior approaches for complex tibial plateau
fractures JBJS Br 1994; 76; 285-289.

231.

Which of the following events occurs during healing of meniscal tears in the
peripheral vascular zone?

36- A plasmin clot forms after injury


37- Inflammatory cells infiltrate the injured area
38- Differentiated meniscal cells cross into the injured area.
39- Newly synthesized acellular collagen matrix bonds the wound edges together.
40- Healing occurs without a fibrovascular scar.

The only chance a meniscal tear has of healing is if it occurs in the peripheral vascular zone. It
goes something like this: a fibrin clot forms that is rich in inflammatory cells. Vessels from the
premeniscal capillary plexus proliferate through the fibrin scaffold, accompanied by the
proliferation of undifferentiated mesenchymal cells. Eventually the lesion is filled with cellular,
fibrovascular scar tissue that bonds the wound edges together abd appears continuous with the
adjacent normal meniscal fibrocartilage. Vessels from the perimeniscal capillary plexus and
from the synovial fringe penetrat the fibrous scar to provide a marked inflammatory response.
ANSWER

232. A 35 y/o carpenter who sustains an injury to his elbow in a fall undergoes emergency surgery. At his 6
week follow up examination, he reports elbow pain and loss of motion. A radiograph is shown in Figures 81.
Management should now consist of
1.
2.
3.
4.
5.

intensive physical therapy


closed reduction and casting for 4 weeks.
radial head replacement.
open reduction and hinged external fixation
total elbow arthroplasty

The question states that the patient is a young carpenter (physical laborer) w/ an elbow injury of some
sort. His post operative x-rays demonstrate that his emergency surgery entailed a radial head excision. The xrays also demonstrate that his ulnohumeral joint is essentially subluxing and perched on the coronoid (some
would say it is dislocated). Elbow stability depends on bony congruity (radial head, coronoid process,
olecrenon) and the soft tissue structure (MCL and LCL). This patients instability is due to his radial head
being excised and ligamentous disruption. In the acute period, radial head excision for radial head fractures is
often an appropriate course provided that there is not instability present. If the elbow is found to be unstable
or the DRUJ or PRUF has been disrupted after the radial head has been excised, a radial head replacement (3)
is usually the treatment of choice. This strategy also prevents proximal migration of the radius. However,
since this patient has been subluxed for up to six weeks, this strategy would force the patient to be
immobilized for some time creating a very stiff elbow. A similar stiff elbow would result if the elbow was
able to be closed reduced (unlikely at this point) and casted (2).
Given the propensity of the elbow to develop a contracture after injury, one of the objectives of
treatment should be to provide sufficient stability to allow post op mobilization. An external fixation device
must provide stability and allow movement through the anatomical center of rotation of the elbow. McKee
and Jupiter recommend the use of hinged elbow external fixator (4) in the treatment of recurrent, complex
elbow instability. It allows early ROM, after fixation or repair of soft tissues. First an open reduction would
be performed to repair any loose ligament and remove soft tissue that would prevent reduction.
Obviously this elbow is quite unstable and needs to be treated with more than just physical therapy (1).
Also, in this young patient, a total elbow (5) would be the last choice and not indicated without arthrosis.
Also, the ROM would likely not improve w/ this strategy.
ANSWER

4.

open reduction and hinged external fixation

References:
McKee, Jupiter: Management of recurrent complex instability of the elbow w/ a hinged external fixator. JBJS
Br 1998;80:1031-6.

233. What is the most common complication reported in the surgical treatment of intraarticular
distal humeral fractures in adults?
1.
2.
3.
4.
5.

Infection
Ulnar nerve injury
Malunion
Distal humeral nonunion
brachial artery injury

Complications include nonunion (1-11%), heterotopic ossification (3-30%), infection (3-7%) and
ulnar nerve injury (7-15%). While the range of incidence is greater for HO, the incidence is
skewed by high energy injuries and head injuries which increase the risk. The answer is ulnar
nerve injury simply due to the proximity of the nerve. Most authors advocate locating and
transposing the ulnar nerve during ORIF to reduce complications.
ANSWER

Kuntz DG, Baratz ME: Fractures of the elbow. OCNA 1999, 30:37-61

234. A 12-year-old boy sustains a 100% displaced closed Salter-Harris type II fracture through
the distal femoral physis while playing football. Neurovascular examination of the
extremity is intact. Management should consist of
12345-

closed reduction and a knee immobilizer.


closed reduction under anesthesia and percutaneous skeletal fixation.
open reduction and transphyseal screw insertion.
skeletal traction for 2 weeks, followed by a spica cast.
a retrograde rigid intramedullary nail.

Thomson JD, Stricker SJ, Williams MM: Fractures of the distal femoral epiphyseal plate.
J Pediatr Orthop 1995;15:474-478.
Lombardo SJ, Harvey JP Jr: Fractures of the distal femoral epiphysis: Factors
influencing prognosis. A review of thirty-four cases. J Bone Joint Surg Am 1977;59:742-751.
First, recall the Salter-Harris classification. This young man has a Type II fracture. A
Type II fracture (the most frequent) extends from the periphery along the epiphyeal plate for a
distance and then turns course through the metaphysis. This type usually occurs in children more
than 10 years old, and the prognosis is considered excellent if the circulation to the epiphysis
remains intact.

Lombardo and his colleagues in 1977 examined 34 fractures of the distal femoral
epiphyseal plate of different types. When they considered all types of fractures together, they
found that the development of a limb-length discrepancy was more closely related to the (1)
amount of displacement and (2) quality of reduction than to the classification (i.e., prognoses
made on the basis of the Salter-Harris classification alone were not reliable).
In a follow-up study by Thomson and his colleagues in 1995, they retrospectively looked
at 30 of these fractures. They concluded that displaced distal femoral physeal separations are
best treated with early, gentle anatomic reduction under general anesthetic (i.e., not in the ER).
Open reduction should follow unsuccessful closed reduction. After reduction, internal fixation
should be used first with percutaneous smooth Steinmann pins. The surgeon may consider screw
fixation provided the immature physis is not violated. Long leg cast and traction immobilization
are inadequate if no internal fixation is used.
ANSWER

235. To avoid wrong site surgery in a patient who is scheduled to undergo knee surgery,
the American Academy of Orthopaedic Surgeons advocates which of the following
actions?
1. Have the surgeon check the side on the radiographs
2. Have the surgeon put an X on the opposite leg
3. Have the surgeon initial the proper leg
4. Have the circulating nurse confirm the site
5. Keep a sock on the opposite leg

The AAOS advisory statement regarding wrong site surgery states the following: Wrong-site
surgery is preventable by having the surgeon, in consultation with the patient when possible,
place her initials on the operative site using a permanent marking pen and then operating through
or adjacent to his or her initials. Spinal surgery done at the wrong level can be prevented with an
intraoperative X-ray that marks the exact vertebral level (site) of surgery. Similarly, institutional
protocols should include these recommendations and involve operating room nurses and
technicians, hospital room committees, anesthesiologists, residents and other preoperative allied
health personnel.
The intended site should be marked such that the mark will be visible after the patient has been
prepped and draped. Once the patient has been moved into the operating room, the surgical team
should pause to take a time-out to communicate about the specific patient and procedure.
ANSWER

http://www.aaos.org/wordhtml/papers/advistmt/1015.htm

236. A 5-year-old boy who sustained a traumatic transtibial amputation 3 years ago now reports
pain and tenderness over the lateral aspect of the residual limb. Radiographs reveal terminal
overgrowth of the fibula and prosthetic modifications have failed to provide relief. Treatment
should now consist of
1. proximal fibular epiphysiodesis
2. shortening of the residual fibula
3. shortening of the residual fibula and tibia
6. resection of the entire fibula
7. conversion to a knee disarticulation

In their book, Handbook of Physical Medicine and Rehabilitation, Krusen, Kottke and
Ellwood, suggest that the juvenille amputee can present challenging management. Below the
knee amputation occurring before the age of 10 years, as in the present question, will inevitably
show evidence of bowing and kyphosis of the stump after some time. Above-knee amputations
is often accompanied by hemiatrophy of the pelvis on the same side as the amputation. The
typical complications of amputation seen in adults, namely painful spurs, symptomatic
neuromata, bursitis and phantom pain, are relatively rare in children. However, what often
complicates the amputation is stump overgrowth.
Between the ages of six and ten there is a disproportionate growth between the skin and
bone, as shown by a long thin spindle of bone growing from the end of the amputated stump.
This is visulaized by a red, swollen and tender stump. Unless managed surgically, this bony end
protrudes form the stump. This most commonly occurs with the fibula, as in the present case. It
can also be seen in the tibia and humerus, however. Surgical shortening of the residual fibula
corrects the problem promptly.
It should be noted, however, that despite the answer preferred in this question, a recent
article published by Pinzur in Chir Narzadow Ruchu Ortop Pol. 1999; 64(5): 571-81, suggests
that when it comes to pediatric amputation, disarticulation of the limbs is the method of choice
because it retains the growth potential of the bone and prevents bony overgrowth as seen in this
case.
ANSWER

237. Which of the following factors is associated with an increased risk for hip fracture?
1) Age less than 50 years
2) Smoking
3) Caucasian male
4) Rural dwelling
5) Obesity

According to OKU, in addition to smoking, other risk factors include: a doubling of the rate for
every decade over 50 years of age, being a Caucasian female, urban dwelling, excessive alcohol
and caffeine intake, physical inactivity, previous hip fracture, use of psychotropic medication,
and senile dementia.
ANSWER

238: The threshold of slowly adapting skin receptors is best assessed with
1.) static two-point discrimination
2.) moving two-point discrimination
3.) Semmes-Weinstein monofilaments
4.) vibrometry
5.) electromyography

The purpose of this study was to determine the validity of tests or a combination of tests for the
diagnosis of carpal tunnel syndrome. Three groups of 50 subjects each were studied: group 1 had
definite carpal tunnel syndrome as defined by history, clinical presentation, and improvement of
symptoms following carpal tunnel release; group 2 had a variety of nontraumatic upper extremity
disorders other than carpal tunnel syndrome; and group 3 subjects were asymptomatic healthy
volunteers. Subjects submitted a self-administered hand diagram, and were queried about night
pain, symptom duration, and coexistent medical conditions. Phalen's test, Tinel's sign, Durkan's
compression test, and Semmes-Weinstein monofilament testing both before and after a Phalen's
maneuver for 5 minutes were performed on each subject. Grip and pinch strengths were
measured. Univariate analysis of groups 1 and 2 showed that the tests with the highest sensitivity
were Durkan's compression test (89%), Semmes-Weinstein testing after Phalen's maneuver
(83%), and hand diagram scores (76%). Night pain was a sensitive symptom predictor (96%).
The most specific tests were the hand diagram (76%) and Tinel's sign (71%). Analysis of groups
1 and 3 without group 2 increased the specificity and predictive value of a positive test. A
regression model was used to develop a multivariate equation with 4 variables. If a patient has an
abnormal hand diagram, abnormal sensibility by Semmes-Weinstein testing in wrist-neutral
position, a positive Durkan's test, and night pain, the probability that carpal tunnel syndrome will
be correctly diagnosed is 0.86. If all 4 of these conditions are normal, the probability that the
patient has carpal tunnel syndrome is 0.0068. We found that the addition of electrodiagnostic
tests did not increase the diagnostic power of the combination of 4 clinical tests.
ANSWER

Szabo RM, Slater RR, Farver TB, Stanton DB, Sharman WK: The value of diagnostic testing in
carpal tunnel syndrome. J Hand Surg Am 1999;24:704-714.

239. Which of the following designs of cementless femoral components has been associated
with an unacceptable incidence of distal osteolysis following total hip arthoplasty?
1 Circumferential proximal coating only
2 Proximal noncircumferential porous coating
3 Uncoated stems
4 Fluted Stems
5 Fully coated stems

A study out of RUSH in 1996 JBJS conducted a histiological study examining 15 titanium alloy
femoral stems obtained from cadaver donors without circumferential porous coating which were
1 to 89 months status Total Hip Arthoplasty. The authors determined that while the percentage of
ingrowth increased with time, the average per cent in growth was only 64%. Four of fifteen had
no continuity with the trabecular bed. Furthermore, periprosthetic membranes were discovered
at the interfaces of the nonporous coating of the implant, and those membranes which had been
in place for more than eight months showed extensive poly debris. In conclusion the authors
state that noncircumferential porous coatings allow for membranous material to form along
points of no coating which in turn provide an avenue for polyethylene debris to enter and disrupt
the diaphyseal canal and lead to distal osteolysis. Therefore, noncircumferential porous coating
is associated with an unacceptable rate of distal osteolysis.
ANSWER

240. What is the most common location for skeletal metastasis from adenocarcinoma?
1.
2.
3.
4.
5.

Femur
Humerus
Hand
Spine
Pelvis

The spine is a frequent site for metastatic disease. These are the most common spine tumors
and only show on plain films after 30% of the body is destructed. Breast, lung, and prostate
mets are common.
ANSWER

spine

241. A 32-year-old man who fell from a second story balcony is hemodynamically stable and
neurologically intact, but he reports foot pain. Pre- and postreduction radiographs are shown in
Figures 82a through 82d. Management should consist of
12345-

immediate application of a short leg cast.


splinting, followed by a short leg cast.
percutaneous pinning
percutaneous screw fixation
open reduction and internal fixation.

The radiographs demonstrate pre and post reduction of a Lisfranc ligament disruption without
obvious fracture. The post reduction radiographs demonstrate residual tarsometatarsal widening.
The treatment of Lisfranc injuries has progressed from closed reduction and percutaneous K-wire
fixation or casting to open reduction and screw fixation. Good results require an accurate and
stable reduction and there is a high incidence of post injury arthritis with poor reduction.
Because of the need for an anatomic and very stable fixation most people now recommend open
reduction with screw fixation. K-wire fixation has been associated with a high rate of failure.
The authors of the Skeletal Trauma conceded that there may be residual stiffness after screw
fixation, but argue that the medial three tarsometatarsal joints have very little motion to begin
with. The paper by Kuo and colleagues relates the long-term follow up of 48 patients who
underwent open reduction and screw fixation for Lisfranc injuries and stresses the importance of
stable reduction. They also state that purely ligamentous Lisfranc injuries, as in this case, are
more prone to failure.
ANSWER

References:
Kuo et al. Outcome after open reduction and internal fixation of Lisfranc joint injuries. JBJS
2000;82;1609-1618
Skeletal Trauma 3rd Ed. 2441-2451.

242. A 21yo woman lacerated the peroneal nerve at the level of the fibular head in a skiing
accident 24 months ago. Because she does not want to continue to use an AFO, treatment should
now consist of
12345-

transfer of the achilles tendon to the peroneus longus


transder of the FHL to the medial cuneiform
interosseous transfer to Pos. Tib. to the lateral foot
EHL transfer
Split anterior tib tendon transfer

The outlook for significant recovery of motor function following transection and repair of the
peroneal nerve is poor and it is difficult to predict recovery. Despite uncertainty, however,
correction of the foot drop can initially be obtained by AFO. If a patient desires a posterior tibial
tendon transfer may be considered as a mean of eliminating the need for an AFO.
Injuries at the level of fibular head causes a foot drop and a supination deformity due to loss of
the anterior tibial muscle (dorsiflexion) and the peroneus longus and brevis (eversion). The most
common procedure used to treat a foot drop in the face of a peroneal nerve injury is a posterior
tibial tendon transfer through the interosseous membrane to dorsum of the foot. This is an outof phase transfer because the PT is inactive in the swing phase but active during the stance
phase.
The author of the reference indicated that a pronation deformity can occur with this procedure.
To alleviate this problem, he recommends transferring the FDL to the navicular at the same time
to counteract the pull.
ANSWER

Dehne R: Congenital and acquired neurologic disorders, Coughlin MJ, Mann RA; Surgery of the
Foot and Ankle ed 7, St louis MO, Mosby 1999, pp552-557

243-

A 20 yo man who is an avid surfer has had left medial knee pain and numbness over
the anteromedial leg for the past 3 weeks. Examination reveals no effusion, no joint line
tenderness, active ROM 0-135, and marked tenderness about the medial calf proximally.
McMurrays test worsens the pain. What anatomic structure is most likely responsible
for these symptoms?

12345-

Medial Meniscus
Medial Collateral Ligament
Saphenous Nerve
Peroneal Nerve
Popliteal artery

This patient suffers from saphenous neuritis, a painful condition caused by compression
of the nerve at the adductor canal or elsewhere along its course. This process is also known as
surfers neuropathy. It is recognized by H&P with most patients presenting with complaints of
medial thigh, knee or calf pain (in the distribution of the saphenous nerve). Symptoms of nerve
irritation may also be apparent such as tingling, shooting pain or numbness. On exam, the
patient will have tenderness with light palpation along the course of the nerve (most specifically
at Hunters canal or along the proximal third of the leg). Motor exam/reflexes will be normal (it
is a purely sensory nerve). McMurrays test places the nerve on stretch and may replicate or
worsen the symptoms. To confirm the diagnosis, a nerve block can be performed at Hunters
canal. Etiologies include compression anywhere along its course (surfers may get it due to
hypertrophy of the adductor musculature) or trauma.
Nonsurgical treatment includes activity modification, TCA, neurontin and injections
(although the success has been limited). Surgical options include decompression if the site of
compression is known, or in extreme cases, neurectomy although permanent sensory deficit will
result.
ANSWER

Morganti CM et al. saphenous neuritis JAAOS 2002 10 130-137.

244. A 42-year-old man has severe shoulder pain and is unable to raise his arm after playing
softball. He reports no history of shoulder problems. Examination reveals dense weakness of
abduction, external rotation, and elbow extension. Which of the following studies would best aid
in diagnosis?
1. MRI with gadolinium
2. Bone scan
3. double contrast arthrography
4. electromyography
5. cervical myelogram

The symptoms described in this particular case can not be explained by an isolated shoulder
problem. Also, the exam is not in any specific dermatomal or nerve root pattern, which lowers
the suspicion for a cervical spine as a cause.
This pattern of a sudden onset of severe pain without specific trauma and with sensory deficits
and muscle weakness in no specific nerve distribution is consistent with acute brachial neuritis or
Parsonage-Turner syndrome. The diagnosis is made by symptoms and confirmed with EMG
testing. Treatment is supportive, consisting of analgesics and physical therapy
Misamore et al followed 7 patients with acute brachial neuritis for an average of 6 years. With
supportive management, the pain decreased spontaneously and resolved completely in all
patients. Weakness decreased very gradually in all patients with 3/7 patients having persistent
mild weakness at 6 years following onset of symptoms.
References:
Misamore GW et al: Parsonage-Turner syndrome. JBJS Am 1996; 78:1405-08.
Aymond JK et al: Neuralgic amyotrophy. Orthop Rev 1989; 18:1275-79.
ANSWER: 4

245.

Which of the following changes takes place in a skeletal muscle as the results of
immobilization or disuse?
1.
2.
3.
4.
5.

Hypertrophy
Increased weight
Increased fatigability
Increased ability to use fats in aerobic metabolic pathways
Decreased strength per cross-sectional area

Explanation:
Disuse can happen with more than just immobilization. It occurs as a voluntary response
to painful conditions, force deprivation by suspension, bed rest, or hypogravity states. Among
the first changes with immobilization is muscle atrophy (not hypertrophy as pointed out in
answer choice 1). The weight of the muscle declines (answer choice 2 is thus wrong), but not in
linear fashion. Near exponential with more weight loss in the initial days than in subsequent
days. Concomitant with loss of mass is loss of strength. Remember, strength is a function of the
muscles cross-sectional area. The chapter points out that when force is normalized for strength
per cross-sectional area, disuse and atrophy produce no change or decrease in strength (answer
choice 5 is wrong). Next they point out that with increased immobilization, the capacity of
muscle to do prolonged work also decreases, or the fatigability increases (answer choice 3 is
right). Lastly, they point out that the increased fatigability is associated with a diminished ability
to use fats in aerobic metabolic pathways (this is why answer 4 is wrong).
Further FYI: Changes that accompany immobilization are related to the lengths at
which the muscles are immobilized. Atrophy and strength loss are more pronounced in muscles
immobilized w/o tension as opposed to those under tension. You still get decreased strength and
cross-sectional area under tension, but there is less of a change in mass, because stretched
myocytes synthesize new contractile proteins, and sarcomeres are added to the ends of existing
fibrils. So changes in cross-sectional area are partially offset by increasing sarcomeres along the
length of the muscle. On the molecular level, the rate of protein synthesis declines within hours
of the initiation of immobilization. Insulin sensitivity declines rapidly. Increases level of
corticosteroids.
ANSWER

Buckwalter J.A., Einhorn, T.A., Simon, S.R., (eds): Orthpaedic Basic Science: Biology and
Biomechanics of the Musculoskeletal System, ed 2. Rosemont, IL, Ameracan Academy of
Orthopaedic Surgeons, 2000, pp 684-716.

246. Control of refractory pain in diffuse blastic metastatic disease can be accomplished with
systemic IV administration of
1. iodine
2. strontium
3. tamoxifen
4. gold
5. selenium

Sorry guys, reference cited is a random article in an obscure journal. Not much I can add to this
other than: you either know it or you dont. Move on to the next question!!! But, I did find a
review article in the tan journal from Jan/Feb 2004 (after the 2003 OITE). Intravenously
administered radiotracers promptly diffuse from the intravascular to the extravascular space and
come into contact with newly produced calcium phosphates, including hydroxyapatite. The
radiopharmaceuticals currently approved by the US Food and Drug Administration for therapy of
bone pain are Sr-89 (strontium) and Sm-153 (samarium). Strontium is chemically similar to
calcium, and is quickly taken up into the mineral matrix of bone. Its uptake occurs preferentially
at sites of active osteogenesis. This allows primary bone tumors and areas of metastatic
involvement to accumulate markely higher concentrations than the surrounding normal bone.
Fairly significant improvement in quality of life and relief of pain have been reported in phase III
clinical trials completed in Canada and the United Kingdom.
ANSWER

Reddy EK, Robinson RG, Mansfield CM: Strontium 89 for palliation of bone metastases. J Natl
Med Assoc 1986; 78: 27-32.
Frassica DA, Frassica FJ, Simon MA, Springfield D (eds): Surgery for Bone and Soft-Tissue
Tumors. Philadelphia, PA, Lippincott-Raven 1998, pp 633-8.
Advances in radionuclide therapeutics in orthopaedics. J Am Acad Orthop Surg 2004; 12(1): 5564.

247. Osteomyelitis is a unique disorder in the neonate because of which of the following
findings?

1epiphyseal extension from the metaphysis, multiple site involvement, and the presence of
staphylococcus aureus.
2epiphyseal extension from the metaphysis, multiple site involvement, and the presence of
group B streptococcus
3epiphyseal extension from the metaphysis, single site involvement, and the presence of
group B streptococcus
4-

metaphyseal and single site involvement and the presence of staphylococcus aureus

5-

metaphyseal and single site involvement and the presence of group B streptococcus

ANSWER

BLyth MJ, et.al, The changing epidemiology of acute and subacute osteomylitis in children
JBJS Br 2001; 83;99-102.

248. Item deleted

249. Stabilization of femoral shaft fractures in the first 24 hours after injury has been shown to
decrease complications in which of the following patient groups?
1. patients with diabetes mellitus
2. patients with an ISS > 18
3. patients with a head injury and elevated intracranial pressure
4. pediatric patients
5. all patients

Bone, et al. JBJS Am 1989;71:336-340 looked at 178 adults with femur fxs
When stabilization of the fracture was delayed in the patients who had multiple injuries, the
incidence of pulmonary complications (adult respiratory-distress syndrome, fat embolism, and
pneumonia) was higher, the hospital stay was longer, and the number of days in the intensivecare unit was increased. The cost of hospital care showed a statistically significant increase for all
patients who had delayed treatment of the fracture compared with those who had early
stabilization.

ANSWER

250. After finger replantation surgery, use of the medical grade leeches is sometimes indicated
to:
1)
2)
3)
4)
5)

prevent digital vessel thrombosis


debride avascular tissue
increase arterial inflow
treat venous congestion
diminish ischemic pain

Ref CORR 1989; 245: 133-137


Leeches, when applied, infect hirudin enzyme. It is an inhibitor of prothrombin to thrombin
conversion and fibrinogen to fibrin conversion. Often in the face of replantation, the venous
vessels are ill defined and are unable to be well repaired. As a result, venous congestion can
occur and can adversely affect the replantation. So leeches can be used to treat the venous
congestion.
ANSWER

251 The process of thrombosis (as indicated by elevation of markers of thrombotic generation
and fibrin formation) is initiated at what point during total hip arthroplasty?
1- Hip dislocation
2- Hip reduction
3- Acetabular reaming
4- Femoral head removal
5- Femoral canal preparation

The following is an excerpt from the abstract of the first article. It summarizes the answer
nicely. However, according to the article, activation starts at the insertion of the femoral
component and not at femoral canal preparation. So, like many other OITE questions, this
question is poorly written and designed to fuck you over. If you know the article you can
eliminate answers 1,2,3, and 4 so you can arrive at the correct choice through the process of
elimination, but you wont feel good about it.
In 3 separate studies, the following observations were made. Circulating indices of
thrombosis and fibrinolysis: prothrombin F1.2, thrombin-antithrombin complexes,
fibrinopeptide A, and D-dimer, did not increase during osteotomy of the neck of the
femur or during insertion of the acetabular component, but rose significantly during
insertion of the femoral component. Thrombin-antithrombin complexes, fibrinopeptide
A, and D-dimer were higher after insertion of a cemented component than insertion of
a noncemented femoral component. A significant decline in central venous oxygen
tension was observed after relocation of the hip joint and after insertions of cemented
and noncemented femoral components, providing evidence of femoral venous
occlusion during insertion of the femoral component. In patients receiving a cemented
femoral component, mean pulmonary artery pressure increased after relocation of the
hip joint, indicating intraoperative pulmonary embolism. No changes in mean
pulmonary artery pressure were noted with noncemented total hip arthroplasty.
Administration of 1000 units of unfractionated heparin before insertion of a cemented
femoral component blunted the rise of fibrinopeptide A. The results of these studies
suggest that (1) the greatest risk of activation of the clotting cascade during total hip
arthroplasty occurs during insertion of the femoral component; (2) femoral venous
occlusion and use of cemented components are factors in thrombogenesis during total
hip arthroplasty; and (3) measures to prevent deep venous thrombosis during total hip
arthroplasty (such as intraoperative anticoagulation) should begin during surgery rather
than during the postoperative period and be applied during insertion of the femoral
component

ANSWER

Sharrock NE, Go G, Harpel PC, Ranawat CS, Sculco TP, Salvati EA: The John Charnley
Award: Thrombogenesis during total hip arthroplasty. Clin Orthop 1995;319:16-27.
Pellici PM, Tria AI, Garvin KL (eds): Orthopaedic.Knowledge Update: Hip and Knee
Reconstruction 2. Rosemont, IL, American Academy of Orthopaedic Surgeons, 2000, pp 13-24.

252. What is the most common initial manifestation of ankylosing spondylitis?


12345-

ANSWER

sacroilitis
aortitis
uveitis
peripheral joint synovitis
decreased chest expansion (less than 2 cm)

Booth RE, et.al. In Herkowitz et, al,; The Spine, 1999, p 429-453.

253. Which of the following is the best predictor of an increased risk for foot complications in a
patient with diabetes mellitus?
1. Diabetes type (I vs II)
2. Elevated level of hemoglobin-A-1-C
3. Use of insulin
4. Associated renal dysfunction (dialysis or transplantation)
5. History of a foot ulcer

---------------------------------------------------------------Pinzur MS et al. Guidelines for Diabetic Foot Care. F&A Int. 1999; 20: 695 702.
RISK FACTORS for developing diabetic foot ulcers include the following:
1. Peripheral neuropathy as measured by semmes-weinstein 5.07 monofilament. Peripheral
neuropathy is the primary risk factor for development of diabetic foot ulcers.
2. Absent pedal pulses DP and/or PT
3. One absent pedal pulse and 3 advanced trophic skin changes including increased hair
growth, abnormal toenails, skin discoloration, and/or atrophy of skin
4. Clinical claudication as determined by a history of exertional calf or leg pain
5. Non-traumatic unilateral partial or total foot amputation
6. History of diabetic foot ulcer
7. Previous hospital admission for a diabetic foot infection
8. Bony (structural) deformity of the foot or ankle
9. Physical findings of peripheral edema or abnormal skin temperatures
ANSWER

254. What fibers in the menisci resist tensile hoop stresses that occur with axial compressive
loading of the knee joint?
1- Oblique
2- Radial
3- Crossing
4- Circumferential
5- Vertical

Menisci are composed of collagen fibers that are arranged radially and circumferentially
(longitudinally). The circumferential fibers resist tensile hoop stresses.
ANSWER

Simon SR (ed): Orthopedic Basic Science. Rosemont, IL, American Academy of Orthopaedic
Surgeons, 1994, pp 74-81.
Bullough PO, Munueral L, Murphy J, Weinstein AM: The strength of the menisci of the knee as
it is related to their fine structure. J Bone Joint Surg Br 1970;52:564-570.

255.

A 42-year-old woman with a 20-year history of type I diabetes mellitus has had pain and
stiffness in her dominant shoulder for the past 3 months. Initial management should
consist of
12345-

a subacromial corticosteroid injection.


manipulation under anesthesia.
physical therapy.
arthroscopic capsular release.
open capsular release.

Although the term frozen shoulder is commonly used, there are no strict guidelines as to what the
diagnostic criteria are. The patient has no inciting factor for shoulder stiffness such as a fracture,
and thus falls into the group of primary frozen shoulder. Diabetes is a known risk factor. Most
often this is a self limiting condition which will resolve after 2-3 years. During this time the
appropriate treatment is phsysical therapy and NSAIDS. Cortisone injections have not shown
any benefit, and may in fact be harmful since they weaken the rotator cuff. The correct answer
for this patient at the three month timepoint is physical therapy.
The cited references deal with arthroscopic release and manipulation under anesthesia. Both
studies represent attempts to shorten the 2 year cycle of frozen shoulder. Both manipulation and
arthroscopic release were successful in >90% of patients after a 3 month period. However, both
methods were used after a mean of at least 6 months of physical therapy.
ANSWER

physical therapy.

Watson L, et.al; Frozen Shoulder: A 12-month clinical outcome trial. J Should Elbow Surg
2000; 9: 16-22.
Dodenhoff RM, et.al.; Manipulation under anesthesia for primary frozen shoulder: Effect on
early recovery and return to activity. J Should Elbow Surg 2000; 9: 23-26.

256. What type of study is used to determine the prevalence rates and risk factors for
homologous transfusion after total knee arthroplasty in consecutive series of patients treated at
one institution for the past 10 years?
1 experimental
2 observational
3 interventional
4 descriptive
5 case-controlled

Experimental: Broad term describing studies in which the researcher exercises control over
the independent variables, deliberately manipulating them. These are sometimes called
intervention studies. For example, in studying the effectiveness of a drug for a certain disease,
the administration of the drug is the independent variable, whereas the resulting disease severity
is the dependent variable.
Observational: Also called nonexperimental studies, this is a broad term describing studies in
which the researcher does not exercise control over the independent variables. Nature is simply
allowed to take its course. From the example above, the researcher would simply observe the
different patients who had or had not taken the drug and seen its effect.
Interventional: Same as experimental studies
Descriptive: Type of observational study which aims to describe the occurrence and distribution
of diseases or other phenomena. They do not try to offer explanations or test a theory or a
hypothesis; they merely generate a description of the frequency of the disease or other
phenomenon of interest according to the places, times, and people involved. They are often the
first method used to study a particular disease.
Case-controlled: Compare people who do have a disease (the case) with otherwise similar
people who do not have the disease (the controls). These studies start with an outcome, or
dependent variable (the presence or absence of disease), and then look back into the past for
possible independent variables that may have caused the disease to see if a possible or suspected
cause or risk factor was present more frequently in people with the disease than in those without.
Also called retrospective studies.
Based on the description of the proposed study, the researcher will simply see what happened
over a 10 year period as it relates to blood transfusions. There are no controls, no manipulation of
independent variables. The description of answer 4 sounds correct but the preferred answer is
number 2 because it is more broad in its application to type of study.
ANSWER

:2

Orthopaedic Basic Science 2nd Ed. AAOS pp 2 17.


High Yield Biostatistics
Janssen: Experimental design and data evaluation in orthopaedic research. J Orth Res 1986;4:
504.

257. The parents of a 7-year-old girl report the development of multiple non-tender
subcutaneous nodules over both distal tibiae for the past few months. Incisional biopsy reveals
granuloma annulare. What is the best course of action?

41- Reassure the family that the nodules are self-limiting


42- Surgically remove all nodules
43- Administer steroids
44- Administer low-dose irradiation
45- Administer low-dose methotrexate.

Granuloma Annulaire is a benign inflammatory dermatosis found in adults and children. It


presents with a rapidly growing, painless soft-tissue mass of the extremities or scalp. Parental
concerns for malignancy and lack of recognition lead to expensive, extensive, and excessive
diagnostic testingand on occasion inappropriate therapy.
Definitve diagnosis requires biopsy, but once confirmed no further intervention is necessary.
The lesions spontaneously resolve over months to years with no long-term sequelae.
Local recurrence and sunsequent lesions are expected in 75%.
ANSWER

258. A 14 y/o boy who plays football reports that he is unable to complete the required laps during the second
week of practice because of right knee pain. Examination reveals reveals a positive patellar grind test and
tenderness and swelling over the tibial tuberosity. What is the best course of action?
1.
2.
3.
4.
5.
6.

Discontinuation of all sports for the season


Crutches with no weight bearing on the right side
Knee immobilizer
NSAIDs and quadriceps stretches
Arthroscopy
Berate the kid for being a wimp

I am quite sure that everyone is very familiar with the suggest references in the Primary Care journal and
American Family Physician. This is a fairly classic story of an adolescent with anterior knee pain. His pain
has been brought on by repetitive activity and there is no history of trauma or a twisting injury that may
indicate a fracture, a ligament tear or meniscus tear. While they do not mention if he has joint line tenderness
or a Lachmans, etc, he have to assume that these are negative and there would not be an indication for an
arthroscopy (5). The patient appears to have a combination of patellar femoral disease (as evidenced by the
+patellar grind) and Osgood Schlatters disease with pain and swelling over his tibial tuberosity. OsgoodSchlatters disease or osteochondrosis of the tibial tuberosity occurs at the time of most rapid growth. Patello
femoral pain if the result of increased joint reactive forces put through the patella when it articulates with the
femur. Weakness of the quad increases these joint reactive forces. The best strategy for these conditions has
been quad stretches and strengthening and NSAIDs (4) to relieve the inflammation.
While a knee immobilizer (3), crutches and NWB (2) or discontinuation of activities (1) are strategies
they would not be indicated in this case. In fact, since this anterior knee pain may be caused by quad
weakness, these techniques may allow his knee to feel better in the short term but when he resumes his
activity, more quad weakness may have occurred and he would likely have continued knee pain. When
anterior knee pain is refractory to PT and NSAIDs, limitation of activity in order to cool down the condition
followed by slowly resuming activities may need to be the course of therapy.
ANSWER

: 4.

NSAIDs and quad stretches

References:
Kaeding: Musculoskeletal injuries in adolescents. Prim Care 1998; 25:211-23.
Peck: Apophyseal injuries in the young athlete. Am Fam Physician 1995; 51: 1891-8.

259. A new scoring system to predict the need for amputation following high energy lower
extremity trauma is being developed. When this system is applied, a score above a threshold
level will result in the decision to perform an amputation. To avoid unnecessary amputations, it
is most important this scoring system have
1.
2.
3.
4.
5.

High sensitivity
low sensitivity
high specificity
low specificity
low negative predictive value

Sensitivity: the number of true positive tests out of all affected. High sensitivity means few
false negatives and a good ability to test presence of a disease. In this case, all those who need
and amputation would get one.
=
true positive
true positive + false negatives
Specificity: the number of true negatives out of all unaffected. High specificity means can
better identify the absence of disease. In this case all those who do not need an amputation do
not get one.
=
true negatives
true negatives + false positives
In this case, you want to eliminate the false positives as much as possible, as this would result in
unnecessary amputation. Thus you want a high specificity.
ANSWER

260. An otherwise healthy 76-year-old woman who sustains a two-part intertrochanteric hip
fracture in a fall was treated with reduction and placement of a 135-degree sliding hip
screw with a two-hole side plate. Examination following the procedure reveals an
excellent reduction, and the tip of the lag screw is 1 cm from the apex of the femoral head
on both the AP and lateral views. These findings will most likely result in
12345-

varus deformity with screw cutout.


failure at the two-hole side plate.
nonunion.
aseptic necrosis.
uneventful healing.

Koval KJ (ed): Orthopaedic Knowledge Update 7. Rosemont, IL, American Academy of Orthopaedic
Surgeons, 2002, pp. 407-416.
Baumgaertner MR, Solberg DB: Awareness of tip-apex distance reduces failure of fixation of trochanteric
fractures of the hip. J Bone Joint Surg Br 1997;79:969-971.
Bolhofner BR, Russo PR, Carmen B: Results of intertrochanteric femur fractures treated with a 125-degree
sliding screw with a two-hole side plate. J Orthop Trauma 1999;13:5-8.

This question has two parts: (1) the location of the tip of the lag screw and (2) the use of a twohole plate.
A sliding hip screw with a fixed angle is commonly used to fix trochanteric fractures of the hip,
but has a reported failure rate of 7% to 13%. Mechanical failure is usually due to cut-out through the
femoral head when the fracture collapses into varus. There is known to be an increased risk of cut-out in
older or osteoporotic patients, those with unstable fractures, and after poor reduction or fixation.
Baumgaertner and Solberg in 1997 devised the use of the tip-apex distance (TAD) from AP and
lateral radiographs to describe the position of the screw within the femoral head and have shown it to be
highly predictive of failure by cutout.

The authors recommend a TAD of under 25 mm. The measurement does not differentiate
between peripheral or shallow placement, but the smaller values do confirm that the implant is more
central and deeply located. In their control group, the cutout was 8% with a TAD of 25 mm. In their
study group, the cutout was 0% with a TAD of 20 mm. In our vignette, the TAD is 20 mm (ideal).
The second part addresses the use of a two-hole side plate. Bolhofner and his colleagues examined
this in 1999. It is unclear from the literature how a 4 hole side plate came into being and the authors
feel that it may have resulted from tradition without actual scientific justification. With regard to
healing time, complication rate, operating room time, blood loss, and collapse (without showing
increased side plate failure), their results compared favorably using a 2 hole side plate. They
conclude that the advantage of a 2 hole side plate, especially in an older more frail patient population,
may be shorter operating room times and less blood loss.

ANSWER

Which of the following factors is LEAST important in determining a patients


perception of the results after a lower extremity limb amputation?
1. residual limb comfort
2. residual limb length
3. contralateral limb condition
4. participation in recreational activities
5. social factors
261.

Matsen, et al studied 148 patients who had had a major lower extremity amputation to
determine which factors are most important in determining a patients perception of the results.
They found the most important factors to be: the comfort of the residual limb; the condition of
the contralateral limb; the comfort, function, and appearance of the prosthesis; social factors; and
the ability to exercise regularly. The level of satisfaction was not strongly correlated with the
level of amputation or even with the unilateral or bilateral nature of the amputation. The
perceived result of bilateral amputation in this study was not significantly worse than that of
unilateral amputation.
ANSWER

262. An 18 year-old female musician cuts off the tip of her nondominant index finger,
transversely removing about 25% of the distal nail bed and the distal tuft of the phalanx. The
patient brings along the tip. Management should consist of
1. defatting the tip and suturing it in place as a composite graft
2. a cross finger flap to the tip
3.a volar V-Y advancement flap
8. microvascular replantation
9. bone shortening and primary closure

In the article in JBJS. 1970:52:921-926, Atasoy et al. describe what they call a new
procedure for the management of distal phalanx injuries. They begin by delineating the various
varieties of amputated finger tip injuries. They categorize them into transverse, oblique dorsal
and oblique palmer. The procedure they go on to describe, known as the triangular volar flap, or
the V-Y flap, is best suited for those patients who have either a transverse amputation or an
oblique dorsal amputation of the finger tip.
The V-Y procedure can be used successfully, they argue, to cover soft tissue and bone
related to finger tip amputation that is not associated with either extensive soft tissue damage or
volarward amputation (so-called oblique palmar). In this article they stress the benefits of this
procedure in providing good cosmetic closure as well as maintaining finger-tip sensation. As in
the present question, fingertip sensation in musicians is of utmost importance.
The procedure involves making a V type incision in which the base of the triangle is the
cut edge of skin where the amputation has occurred. If the amputation is of the distal phalanx,
the tip of the triangle should be placed at the distal flexion crease, since it is easier to advance a
longer flap. The distally based triangular flap is developed by cutting only through full thickness
of skin. The nerves and blood vessels of the flap are preserved. Separation of the fibrofatty
subcutaneous tissue from periosteum and flexor tendon sheath aids in mobilization of the flap.
After debridement of the stump, the flap is then advanced over the bone and the base of the
triangle is sutured to the nail bed with 6.0 nylon sutures. The V incision on the palmar aspect of
the distal phalanx is then closed by converting it to a Y.
The authors preformed this procedure in a series of sixty-four fingers on sixty-one
patients and had exceptional results, with normal sensation in all but two patients who initially
lost sensation but then regained normal sensation later on. Post-operatively, only a simple finger
dressing and guard is required without immoblization.
ANSWER

263. Patients with metal-on-metal total hip replacements, when compared to patients with metalon-polyethylene total hip replacements, have been shown to have
1) Increased serum and urine metal concentrations.
2) Increased amount of osteolysis.
3) Greater range of motion.
4) A lower rate of loosening
5) An increased incidence of cancer

See OKU 7 for more details. The effects of such serum and metal concentrations on the body are
being further studied.
ANSWER

264: A 27-year-old man injures his neck in a motor vehicle accident. Neurologic examination
reveals intact light touch and pinprick sensation. His deltoids and biceps are 5/5, and he has 3/5
strength in his left hand and 2/5 strength in the right hand. He has 4/5 strength throughout his
left lower extremity and 4+/5 on the right side. His neurologic status could most accurately be
classified as
1.) complete cord injury
2.) posterior cord syndrome
3.) anterior cord syndrome
4.) central cord syndrome
5.) Brown-Sequard syndrome
A complete spinal cord injury is manifested by total motor and sensory loss distal to the injury.
When the bulbocavernosus reflex is positive and no sacral sensation or motor function has
returned, the paralysis will be permanent and complete in most patients. An incomplete spinal
cord syndrome may be a Brown-Squard syndrome, central cord syndrome, anterior cord
syndrome, posterior cord syndrome, or rarely monoparesis of the upper extremity. Ninety percent
of incomplete lesions produce either a central cord syndrome, a Brown-Squard syndrome, or an
anterior cervical cord syndrome. Central cord syndrome is the most common. It consists of
destruction of the central area of the spinal cord, including both gray and white matter. The
centrally located arm tracts in the cortical spinal area are the most severely affected, and the leg
tracts are affected to a lesser extent. Generally patients have a quadriparesis involving the upper
extremities to a greater degree than the lower. Sensory sparing is variable, but usually sacral
pinprick sensation is preserved. Frequently these patients show immediate partial recovery after
being placed in skeletal traction through skull tongs. Prognosis is variable, but more than 50% of
patients have return of bowel and bladder control, become ambulatory, and have improved hand
function. This syndrome usually results from a hyperextension injury in an older person with
preexisting osteoarthritis of the spine. The spinal cord is pinched between the vertebral body
anteriorly and the buckling ligamentum flavum posteriorly. It also may occur in younger
patients with flexion injuries. Brown-Squard syndrome is an injury to either half of the spinal
cord and usually is the result of a unilateral laminar or pedicle fracture, penetrating injury, or a
rotational injury resulting in a subluxation. It is characterized by motor weakness on the side of
the lesion and the contralateral loss of pain and temperature sensation. Prognosis for recovery is
good, with significant neurological improvement often occurring.Anterior cord syndrome usually
is caused by a hyperflexion injury in which bone or disc fragments compress the anterior spinal
artery and cord. It is characterized by complete motor loss and loss of pain and temperature
discrimination below the level of injury. The posterior columns are spared to varying degrees,
resulting in preservation of deep touch, position sense, and vibratory sensation. Prognosis for
significant recovery in this injury is poor.Posterior cord syndrome involves the dorsal columns of
the spinal cord and produces loss of proprioception vibrating sense while preserving other
sensory and motor functions. This syndrome is rare and usually is caused by an extension injury.
Canale: Campbell's Operative Orthopaedics, 10th ed., Copyright 2003 Mosby, Inc. p1603.

ANSWER

265. What is the most common complication associated with open repair of an Achilles tendon
rupture?
1 Infection
2 Neurologic injury
3 Loss of motion
4 Rerupture
5 Skin Healing Problems

In the article Quantitative Review of Operative and Nonoperative of Achilles Tendon Ruptures
comparisons were made throughout 125 articles between closed and operative management of
Achilles tendon rupture. In general complication rates were lowest with open repair and early
mobilization. However, the largest dichotomy and in fact the most common complication in
open repair is delay in skin healing. The open groups had skin complication rates of 14.6%,
while the closed group was 0.5%. Of note, rerupture rates were highest in conservatively
managed patients (6.7%) and lowest in external fixation groups (0%). Therefore the most
common complication following open repair of Achilles tendon rupture is Skin healing
problems.
ANSWER

266. An 18 year old sustains a moderate inversion ankle sprain. Early functional recovery is
best achieved with:
1.
2.
3.
4.
5.

a prolonged period of no weight bearing


immediate muscle stimulation
immobilization followed by strengthening
early use of ultrasound and electrical stimulation
strength and proprioceptive training

External rotation is associated with syndesmotic, peroneal tendon, and deltoid injuries.
Inversion is associated with CFL injury. Sprains are classified I, II, and III. Treatment of
rest, ice, compression, elevation are important early treatments. Early weight bearing and
range of motion (alphabet) exercises are the mainstay of early rehab. Braces can be used for
grade I and II. Grade III sprains can be casted or treated with a removeable cast.
FUNCTIONAL REHAB CONSISTS OF ISOMETRIC AND RESISTANCE EXERCISES
AND PROPRIOCEPTIVE TRAINING. OKU 7 pg 557.
ANSWER

267.

A 14-year-old female volleyball player reports that her shoulder dislocated and
spontaneously reduced while she was serving a volleyball. In the following 6 months,
she had three similar episodes. Which of the following conditions is most likely present?

12345-

Capsular laxity
Hill-Sachs lesion
Anterior inferior labral tear
Superior labral anterior posterior tear
Rotator cuff tear

The scenario describes a young athlete with recurrent shoulder dislocation during overhead
activity without a history of a specific trauma. Remember the pneumonic, AMBRI: Atraumatic
Multidirectional Bilateral shoulder dislocation/subluxation which often responds to
Rehabilitation or sometimes requires an Inferior capsular shift. This is in contrast to, TUBS:
Traumatic Unilateral dislocations with a Bankart lesion that often require Surgery. Capsular
laxity may be congenital or acquired. Congenital laxity is usually bilateral while acquired laxity
may be secondary to repetitive motion such as in serving a volleyball. The conversion from a
functionally stable ligamentously lax shoulder to one with instability is usually secondary to a
relatively nontraumatic event.
Hill-Sachs lesion is a compression fracture of the posterolateral aspect of the humeral head
secondary to an anterior glenohumeral dislocation. It is the result of instability, not the cause of
it.
Anterior inferior labral tears are relatively uncommon and patients usually present with pain or
mechanical catching.
The mechanism of SLAP injuries remains unclear, but falls on an outstretched hand and
overhead throwing activities have been implicated. Patients often present with pain during
overhead activities, mechanical catching, popping or grinding, depending on the type of SLAP
lesion present. SLAP lesions are associated with shoulder instability.
Rotator cuff tears are often related to overhead motion, but usually present with pain with
activity. Muscle weakness, catching and grinding may also be present.
ANSWER

Burkhead WZ Jr. Rockwood CA Jr.: Treatment of instability of the shoulder with an exercise
program. JBJS 1992;74;890-896
Schenck TJ, Brems JJ: Multidirectional instability of the shoulder: Pathophysiology, diagnosis,
and management. JAAOS 1998;6:65-72.

268. The apical ectodermal ridge is an embryonic structure necessary for growth and
development of the
12345-

anterior neuropore
neural tube
limbs
rostral sclerotomes
ventral gut

At the limb of the limbbud lies a thickened ridge of the ectoderm, the apical ectodermal ridge
(AER), which has a role in patterning the structures that form within the limb.
The upper limb buds appears from the lower cervical region at 24 days. The lower limb buds
appear at 28 days. Each limb bud consists of an outer ectodermal cap and an inner mesodermal
core. There is a ridge-like thickening along the apex of the ectoderm called the apical
ectodermal ridge. This is ridge is essential for normal limb differentiation.
ANSWER

Lovell and Winters Pediatric Orthopaedics ed 5. Philadelphia PA, Lippincott Williams and
Wilkins, 2000 1-31

269. A patient with ankylosing spondylitis is scheduled to undergo THA. Which of the
following is an important preoperative consideration?
12345-

Flexion-Extension cervical spine radiographs


Radiation therapy consultation
MRI of the Spine
Urology Consultation
Discontinuation of methotrexate.

Patients with AS have involvement of the hip in as high as 50% of patients, with 90% of
these patients being bilateral. The reference summarizes the results of 186 hips treated in 103
patients with an average of 10 year follow-up. 96% of patients achieved excellent pain relief but
only 69% of patients achieved excellent hip function, mostly secondary to limitations in ROM.
There was a 19 complication rate with 2 deaths (Aortic dissection in one, PE in another), 7
infections, 4 dislocations, 5 nonunion of the greater trochanter and 1 sciatic nerve palsy. 21
patients developed heterotopic ossification (type I and II mostly) and no cases of reankylosis.
The discussion quotes rates of HO as high as 77% in some studies.
Interestingly enough, there is no mention in the reference of radiation for prophylaxis.
The authors briefly mention the use of NSAIDs and ASA for prophylaxis and actually
recommend against it, stating that the HO did not significantly impact their patients or their
function.
ANSWER

Reference:
Joshi AB et al. THA in AS. J Arthroplasty 2002; 17 (427-33).

270. Orthotripsy (shock-wave therapy) has been approved for the treatment of:
1.plantar fibromatosis
2.sinus tarsi syndrome
3.rheumatoid enthesopathy of the Achilles tendon
4.calcific tendonitis of the heel
5.chronic proximal plantar fasciitis

Extracorporeal shock wave therapy is routinely used for treatment of urolithiasis. Recently,
shock wave therapy has gained increasing acceptance for musculoskeletal problems due to its
noninvasive nature and minimal associated complications.
Ogden et al reported a series of 320 patients with chronic proximal plantar fasciitis treated with
shock wave therapy vs. placebo. 56% more of patients treated with shock wave therapy had
improved results . The authors conclude that electro-hydraulic shock wave therapy applied
directly to the plantar fascia insertion on the calcaneus is a safe and effective treatment for
recalcitrant refractory proximal plantar fasciitis,
Alvarez et al conducted a meta-analysis of shock wave therapy on chronic plantar fasciitis.
Success rates as high as 88% were noted in patients with chronic proximal plantar fasciitis.
In summary, shock wave therapy represents a safe and effective means of treating chronic
proximal plantar fasciitis. It offers an alternative to cortisone injection which may be associated
with rupture, and it should be considered prior to any surgical intervention

Ogden JA et al: Shock wave therapy in musculoskeletal disorders. Clin Orthop 2001;
387: 22-40.
Alvarez R: Preliminary results on the safety and efficacy of the OssaTron for treatment of
plantar fasciitis. Foot Ankle Int 2002; 23:197-203.

ANSWER: 5

271. Which of the following factors has been shown to contribute to poor results after anatomic
reduction of posterior wall acetabulum fractures.?
12345-

ANSWER

marginal articular impaction


involvement of the wieght bearing dome
delay in reduction of hip dislocation greater than 12 hours
size of the wall fragment
patient age of less than 40 years

Moed BR, et.al, results of operative treatment of fractures of the posterior wall of the
acetabulum JBJS 2002; 84; 752-758.

272. During total knee arthroplasty with trial components in position, the knee can be fully
extended but is tight in flexion. Which of the following actions would improve the flexionextension mismatch?
1. Resect more distal femur
2. Resect more proximal tibia
3. Downsize the femoral component
4. Downsize the tibial insert
5. Increase flexion of the femoral component.
The best reference for this is the chart in the Miller textbook on page 259. Key is understanding
it so that no memorization would be necessary. Distal femur cut/ length affects your extension.
While the AP size (femoral component size) and the posterior tibial slope affects flexion. Goal is
to convert asymmetric gap to symmetric gap.
Extension tight
Flexion tight
Extension good
Flexion loose

Symmetric gap

1) cut more proximal tibia

Asymmetric gap
Cut too much posterior
femur

Extension tight
(contracture)
Flexion good

Extension good
Flexion tight (will
not bend fully)

Asymmetric gap
Did not cut enough
distal femur or release
enough posterior
capsule/osteophytes
asymmetric gap
did not cut enough
posterior bone or PCL
scarred/ too tight
not enough posterior
slope in tibial bone
asymmetric gap
cut too much distal
femur

1) increase size of femoral


component
2) fill posterior gap with cement
or metal augmentation
1) release posterior capsule
2) take off more distal femur (1-2
mm at a time

Extension loose
(recurvatum)
Flexion good

ANSWER

1) decrease size of femoral


component
2) recess PCL
3) check posterior slope of tibia
(may need more
1) Distal femur augmentation
2) Use smaller size femur to
convert to symmetric gap
3) Use thicker polyethylene
insert, then address tight
flexion gap

:3

Ries MD: Anterposterior positioning of the femoral component in total knee arthroplasty.
Seminars in Arthroplasty1999; 10: 219-24.

273. In an athlete who has full, painless range of motion and a normal neurologic examination,
which of the following is considered an absolute contraindication to participation in a collision
sport such as football?

1-

a history of C45 anterior diskectomy and fusion

2-

a history of spinal cord neurapraxia with transiet quadriparesis that has now resolved.

3-

os odontoideum

4-

congenital C56 fusion

5-

congenital stenosis

ANSWER

Thomas BE, et.al. Cervical spine injuries in football players JAAOS 1999; 7; 338-347.
TOrg JS; et.al. Neurapraxia of the cervical spine with transient quadriplegia. JBJS 1986; 68;
1354-1370.

274. A 22 year old professional skier is unconscious after hitting a retrain ing fence during a
race. The patients helmet and face shield are intact. Appropriate initial management should
consist of

1-

removal of the helmet to access the airway

2-

removal of the helmet with immediate transportation

3-

removal of the helmet followed by cervical spine imjmobilization

4-

removal of face shield to access the airway without immobilization

5-

immobilization fo the cervical spine, leaving the helmet in place and removing the face
shield.

ANSWER

Warren WL, et.al; On the field evaluation of athletic neck injuries. JAAOS 1999; 17; 99-110.

275. What is the most common site for metastases from synovial sarcoma?
1. liver
2. lung
3. lymph nodes
4. bone
5. synovium

Campbells and Robbins Pathologic Basis of Disease: Synovial sarcoma typically arises near a
joint. It does not arise from synoviocytes; rather, it is so named because of its histologic
appearance that is similar to synovial tissue. Tissue elements are biphasic with predominantly
spindle cells with interspersed pseudoglandular tissue. The typical age is a young adult, though
it has been reported in patients from 10 weeks to 92 yrs. Presentation is pain or mass or both.
Xrays may show soft tissue calcification. Current recommended treatment is wide resection
followed by local XRT and adjuvant chemotherapy. It can metastasize via lymphatics to
regional nodes, but the most common route of metastasis is hematogenous to the lungs.
ANSWER

Você também pode gostar